Sunteți pe pagina 1din 83

LEUNG YEE, plaintiff-appellant, vs. FRANK L.

STRONG MACHINERY COMPANY ISSUE: W/N the defendant has a better title to the building subject of the action?
and J. G. WILLIAMSON, defendants-appellees.
HELD:
FACTS:
Defendant. The Court ruled that the defendant shall have a better title than
Compania Agricola Filipina (Compania) bought a considerable quantity of rice- the plaintiff on account of the third paragraph of Art. 1473 and not due to the lower
cleaning machinery from the defendant machinery company and executed a chattel court’s rationale.
mortgage as security for the payment of the purchase price. It included in its mortgage
deed the building in which the machinery was installed but with no reference to the land The court propounded that the “registry” referred to in the 2 nd paragraph of Art
where it stands. When the debt secured was not paid when it became due, the 1473 is of course the registry of the real property and it must be apparent that the
mortgage was foreclosed and the mortgage was sold to the defendant. The mortgage annotation or inscription of a deed of sale of real property in a chattel mortgage registry
was registered in the chattel mortgage registry and the sale was annotated on cannot be given the legal effect of an inscription in the registry of real property.
December 29, 1913. The machinery was also delivered to the possession of the
defendant on the same date. By its express terms, the Chattel Mortgage Law contemplates and makes
provision for mortgages of personal property; and the sole purpose and object of the
On January 14, 1914, Compania executed an unregistered deed of sale on chattel mortgage registry is to provide for the registry of "Chattel mortgages," that is to
the land where the building stands in favor of the defendant in order to cure any defect say, mortgages of personal property. The building of strong materials in which the rice-
on the defendant’s title to the building under the sheriff’s certificate of title. cleaning machinery was installed is a real property and the mere fact that the parties
seem to have dealt with it separate and apart from the land on which it stood in no wise
At or about the same time the chattel mortgage was issued by Compania in changed its character as real property. It follows that neither the original registry in the
favor of the defendant, Compania has also executed another mortgage upon the same chattel mortgage registry of the instrument purporting to be a chattel mortgage of the
building to the plaintiff separate from the land it stands for the payment on the building and the machinery installed therein, nor the annotation in that registry of the
construction of the building. Also, upon Compania’s failure to pay its debt from the sale of the mortgaged property, had any effect whatever so far as the building was
plaintiff, the plaintiff has secured a favorable judgement and has thereafter managed to concerned.
levy the building. The plaintiff bought the building upon execution sale on December
18, 1914 and had the sheriff’s certificate of sale registered in the registry. However, the Court is of opinion that the plaintiff is not a purchaser in good
faith.
At the time of the execution of the levy on the building, the defendant who was
in possession of the same filed a sworn statement to the sheriff setting forth its claim The agreed statement of facts clearly discloses that the plaintiff, when he
and demanded its concomitant release from the levy. Thereafter, upon demand of bought the building at the sheriff's sale and inscribed his title in the land registry, was
the sheriff, the plaintiff executed an indemnity bond in favor of the sheriff in the duly notified that the machinery company had bought the building from plaintiff's
sum of P12,000, in reliance upon which the sheriff sold the property at public judgment debtor; that it had gone into possession long prior to the sheriff's sale; and
auction to the plaintiff, who was the highest bidder at the sheriff's sale. that it was in possession at the time when the sheriff executed his levy. The execution
of an indemnity bond by the plaintiff in favor of the sheriff, after the machinery company
This action was executed by the plaintiff for recovery of possession of the had filed its sworn claim of ownership, leaves no room for doubt in this regard. As such,
building. He took the risk and must stand by the consequences; and it is in this sense that we
find that he was not a purchaser in good faith.
RTC: The RTC ruled in favor of the defendant applying Art 1473 of the Civil Code and
on the ground that the defendant had its title to the building registered prior to the date One who purchases real estate with knowledge of a defect or lack of title in
of registry of the plaintiff's certificate. his vendor cannot claim that he has acquired title thereto in good faith as against the
true owner of the land or of an interest therein; and the same rule must be applied to
Art. 1473. "If the same thing should have been sold to different vendees, the one who has knowledge of facts which should have put him upon such inquiry and
ownership shall be transferred to the person who may have first taken possession investigation as might be necessary to acquaint him with the defects in the title of his
thereof in good faith, if it should be personal property. vendor.

"Should it be real property, it shall belong to the person acquiring it who first Hence, the Court sustained the title of the defendant under the third paragraph
recorded it in the registry. of the above cited article of the code, it appearing that the company first took
possession of the property; and further, that the building and the land were sold to the
"Should there be no entry, the property shall belong to the person who first machinery company long prior to the date of the sheriff's sale to the plaintiff.
took possession of it in good faith, and, in the absence thereof, to the person who
presents the oldest title, provided there is good faith."
DEVELOPMENT BANK OF THE PHILIPPINES, petitioner, vs. COURT OF CA: upon appeal, CA uphold the RTC decision.
APPEALS, MYLO O. QUINTO and JESUSA CHRISTINE S. CHUPUICO,
respondents. Hence this petition. Petitioner contends that respondents fraudulently
obtained the property in litigation.
FACTS:
ISSUE: W/N the land in dispute could have been validly mortgaged while
Petitioner extend a loan of php94,000 to Sps. Olidiana. To secure the loan the still the subject of a Free Patent Application with the government?
sps. executed a real estate mortgage on several properties among which was Lot 2029
which at the time of mortgage was still the subject of a Free Patent application filed by HELD:
the Olidianas with the Bureau of Lands but registered under their name in the Office of
the Municipal Assessor of Molave for taxation purposes. No. The court upheld the decision of the court a quo. It held that petitioner
bank did not acquire valid title over the land in dispute because it was public land when
On Nov. 2, 1978, the Sps. Olidiana filed with the Bureau of Lands request for mortgaged to the bank. The court cannot accept petitioner's contention that the lot in
amendment of their application for free patent over several parcels of land including Lot dispute was no longer public land when mortgaged to it since the Olidiana spouses had
2029. In the said request, they renounced, relinquished and waived their rights and been in open, continuous, adverse and public possession thereof for more than thirty
interests over Lot 2029 to the private respondents. The same was approved and (30) years.
concomitant free patents were issued in favor of the private respondents covering each
of them ½ of the total area of the Lot 2029. Thereafter, the private respondents obtained As held in Visayan Realty, Inc. v. Meer, The Court ruled that he approval of a
certificate of titles thereof to their respective share to Lot 2029. sales application merely authorized the applicant to take possession of the land so that
he could comply with the requirements prescribed by law before a final patent could be
On April 20, 1979, the spouses have again obtained another loan from the issued in his favor. Meanwhile the government still remained the owner thereof, as in
petitioner executing once more a mortgage upon lot 2029. fact the
application could still be canceled and the land awarded to another applicant should it
Thereafter, when the spouses failed to meet its obligation, the petitioner be shown that the legal requirements had not been complied with. What divests the
extrajudicially foreclosed all their mortgaged properties. Consequently, a public auction government of title to the land is the issuance of the sales patent and its subsequent
was held on April 14, 1983 on which the petitioner emerged as the highest bidder. A registration with the Register of Deeds. It is the registration and issuance of the
certificate of sale was issued to the petitioner and an Affidavit of Consolidation of certificate of title that segregate public lands from the mass of public domain and
Ownership was registered in its name. However, upon registration and consolidation, convert it into private property.
the petitioner found that the Lot 2029 was already registered and divided under the
name of the private respondents. Since the disputed lot in the case before us was still the subject of a Free
Patent Application when mortgaged to petitioner and no patent was granted to the
In view thereof, the petitioner filed an action for quieting title and cancellation Olidiana spouses, Lot No. 2029 remained part of the public domain.
or annulment of certificate of title against private respondents.
With regard to the validity of the mortgage contracts entered into by the
RTC: the RTC ruled in favor of the private respondents holding that the contracts of parties, Art. 2085, par. 2, of the New Civil Code specifically requires that the pledgor or
mortgage entered into by petitioner and the subsequent foreclosure of subject property mortgagor be the absolute owner of the thing pledged or mortgaged. Thus, since the
could not have vested valid title to petitioner bank because the mortgagors were not disputed property was not owned by the Olidiana spouses when they mortgaged it to
the owners in fee simple of the property mortgaged. The court also found the mortgages petitioner the contracts of mortgage and all their subsequent legal consequences as
over Lot No. 2029 regards Lot No. 2029 are null and void.
of no legal consequence because they were executed in violation of Art. 2085, par. 2,
of the New Civil Code which requires that the mortgagor be the absolute owner of the Finally, anent the contention of petitioner that respondents fraudulently
thing mortgaged. According to the court a quo there was no evidence to prove that the obtained the property in litigation, we also find for the latter. As correctly found by the
mortgagors of the land in dispute were its absolute owners at the time of the mortgage lower courts, no evidence existed to show that respondents had prior knowledge
to petitioner. of the real estate mortgages executed by the Olidiana spouses in favor of
petitioner. The act of respondents in securing the patents cannot therefore be
Resultantly, the trial court declared the following as null and void insofar as categorized as having been tainted with fraud.
they related to Lot No. 2029 (Pls-61) being a public land: the real estate mortgage dated
4 April 1978, the second mortgage dated 23 April 1979, the foreclosure sale on 14 April
1983, the certificate of sale registered with the Register of Deeds of Zamboanga del
Sur on 1 September 1983, and the affidavit of consolidation of ownership registered
with the Register of Deeds on 2 August 1985.
HEIRS OF LEOPOLDO DELFIN and SOLEDAD DELFIN, namely EMELITA D. Petitioner: Petitioner contends that they and their predecessors-in-interests' open,
FABRIGAR and LEONILO C. DELFIN, petitioners, vs. NATIONAL HOUSING continuous, exclusive, and notorious possession of the Iligan Property for more than 30
AUTHORITY, respondent. years converted the property from public to private.

FACTS: Petitioners also assert that the Court of Appeals disregarded certifications and
letters from government agencies, which support their claims, particularly, their and
In a complaint for payment for parcels of land and improvement, the Delfin their predecessors-in-interest's possession since June 12, 1945.
spouses claimed that they were the owners of a 28,800 square meter parcel of land in
Townsite, Suarez, Iligan City (the "Iligan Property"). They allegedly bought the property Respondent: NHA counters, citing the Court of Appeals Decision, that petitioners
in 1951 from Felix Natingo and Carlos Carbonay, who, allegedly, had been in actual cannot rely on Section 14 (2) of Presidential Decree No. 1529 because the property
possession of the property since time immemorial. The Delfin Spouses had been was not yet declared private land when they filed their Complaint. And the property is
declaring the Iligan Property in their names for tax purposes since 1952, and had been part of military reservation area pursuant to Proclamation 2151 (which should be 2143).
planting it with mangoes, coconuts, corn, seasonal crops, and vegetables. ISSUE: W/N the petitioners and their predecessors-in-interests have been in
possession of the Iligan Property for such duration and under such circumstances as
They further alleged that, sometime in 1982, respondent National Housing will enable them to claim ownership which entitles them for just compensation?
Authority forcibly took possession of a 10,798 square meter portion of the property.
Despite their repeated demands for compensation, the National Housing Authority HELD:
failed to pay the value of the property. The Delfin Spouses thus, filed their Complaint.
Yes. Petitioners are erroneously claiming title based on acquisitive
In its answer, the NHA alleged that the property was part of a military prescription under sec. 14(2) of PD No. 1529 but may claim title by prescription
reservation area. It cited Proclamation No. 2151 (which should be Proclamation No. under Sec. 48(b) of Commonwealth Act No. 141 (the Public Land Act)
2143). According to the According to the National Housing Authority, Proclamation No.
2151 also mandated it to determine the improvements' valuation. Based on the study I. The period to acquire public land by acquisitive prescription under
of the committee it created, the value of the property was supposedly only P4.00 per Presidential Decree No. 1529 begins to run only after the promulgation of
square meter, regardless of the nature of the improvements on it. a law or a proclamation by the President stating that the land is no longer
intended for public use or the development of national wealth.
It emphasized that among all claimants, only the Delfin Spouses and two
others remained unpaid because of their disagreement on the property's valuation. Petitioners are erroneously claiming title based on acquisitive prescription
under Section 14 (2) of Presidential Decree No. 1529 which provides: “Those who have
The National Housing Authority failed to appear during the pre-trial conference acquired ownership of private lands by prescription under the provision of existing
and was declared in default, hence case was set ex parte. laws.”
RTC: RTC ruled in favor of the sps. Delfin stating that it had no reason to doubt the
evidence presented by the Delfin Spouses. For acquisitive prescription to set in pursuant to Section 14 (2) of Presidential
Decree No. 1529, two (2) requirements must be satisfied: first, the property is
The NHA filed for motion for reconsideration but was denied. Hence, it established to be private in character; and second the applicable prescriptive
appealed. period under existing laws had passed.

CA: On appeal, the CA reversed the RTC decision, stating that the characterization of Property — such as land — is either of public dominion or private
the property is no longer an issue because the National Housing Authority already ownership.
conceded that the property is disposable public land by citing Proclamation No. 2151,
which characterized the property as "a certain disposable parcel of public land." 1. Land is public dominion if it either: (a) is intended for public use; or (b)
However, the Delfin Spouses supposedly failed to establish their possession of belongs to the State, without being for public use, and is intended for
the property since June 12, 1945, as required in Section 48 (b) of the Public Land some public service or for the development of the national wealth."
Act.
2. Private property "consists of all property belonging to private persons,
During the pendency, the sps. Delfin died, the hers filed for motion to be either individually or collectively," as well as "the patrimonial property of
substituted but was remained unacted by the CA. the State, provinces, cities, and municipalities.

Note that Land that belongs to the state but which is not or is no longer
Hence, this petition. intended for public use, for some public service or for the development of the national
wealth, is patrimonial property; it is property owned by the State in its private capacity.
For land of the public domain to be converted into patrimonial property, there Section 48 (b) of the Public Land Act therefore requires that two (2) requisites
must be an express declaration — "in the form of a law duly enacted by Congress or a be satisfied before claims of title to public domain lands may be confirmed: first, that
Presidential Proclamation in cases where the President is duly authorized by law"— the land subject of the claim is agricultural land; and second, open, continuous,
that "the public dominion property is no longer intended for public service or the notorious, and exclusive possession of the land since June 12, 1945.
development of the national wealth or that the property has been converted into
patrimonial." The need for the land subject of the claim to have been classified as
agricultural is in conformity with the constitutional precept that "alienable lands of the
Accordingly, only publicly owned lands which are patrimonial in character are public domain shall be limited to agricultural lands."
susceptible to prescription under Section 14 (2) of Presidential Decree No. 1529 in
relation to Article 1113 of Civil Code which states: “all things which are within the As explained in Heirs of Malabanan v. Republic,
commerce of men are susceptible of prescription, unless provided. Property of the State
or any of its subdivisions not patrimonial in character shall not be the object of Under the 1935 Constitution, lands of the public domain were classified into
prescription.” three, namely, agricultural, timber and mineral.

Contrary to petitioners' theory then, for prescription to be viable, the publicly- The 1987 Constitution adopted the classification under the 1935 Constitution
owned land must be patrimonial or private in character at the onset. Possession for into agricultural, forest or timber, and mineral, but added national parks. Agricultural
thirty (30) years does not convert it into patrimonial property. lands may be further classified by law according to the uses to which they may be
devoted. The identification of lands according to their legal classification is done
It is disclosed that attached to the present Petition was a copy of a May 18, exclusively by and through a positive act of the Executive Department.
1988 supplemental letter to the Director of the Land Management Bureau which
embodied: Based on the foregoing, the Constitution places a limit on the type of public
land that may be alienated. Under Section 2, Article XII of the 1987
“It is very clear in the 4th Indorsement of the Executive Secretary dated Constitution, only agricultural lands of the public domain may be alienated; all other
April 24, 1954 the portion thereof that will not be needed for any public or quasi-public natural resources may not be.
purposes, be disposed in favor of the actual occupants under the administration of the
Bureau of Lands (copy of the Executive Order is herewith attached for ready Alienable and disposable lands of the State fall into two categories, to wit:
reference).”
(a) patrimonial lands of the State, or those classified as lands of private ownership
However, a mere indorsement of the executive secretary is not the law or under Article 425 of the Civil Code, without limitation; and
presidential proclamation required for converting land of the public domain into (b) lands of the public domain, or the public lands as provided by the Constitution,
patrimonial property and rendering it susceptible to prescription. There then was no but with the limitation that the lands must only be agricultural.
viable declaration rendering the Iligan property to have been patrimonial property at the
onset. Accordingly, regardless of the length of petitioners' possession, no title could As the Court of Appeals emphasized, respondent has conceded that the Iligan
vest on them by way of prescription. property was alienable and disposable land public land. This characterization of the
property is conceded by [respondent] who cites Proclamation No. 2151 as declaring
II. Under Commonwealth Act No. 141, a claimant may acquire alienable and that the disputed property was "a certain disposable parcel of public land."
disposable public land upon evidence of exclusive and notorious
possession of the land since June 12, 1945. Thus, the fact that the Iligan property was alienable and disposable,
agricultural land, has been admitted. What is claimed instead is that petitioners'
Section 48 enabled the confirmation of claims and issuance of titles in favor possession is debunked by how the Iligan Property was supposedly part of a military
of citizens occupying or claiming to own lands of the public domain or an interest reservation area.
therein.
Indeed, by virtue of Proclamation No. 2143 (erroneously referred to by
Section 48 (b) specifically pertained to: those who "have been in open, respondent as Proclamation No. 2151) certain parcels of land in Barrio Suarez, Iligan
continuous, exclusive, and notorious possession and, occupation of agricultural lands City were reserved for slum-improvement and resettlement program purposes.
of the public domain, under a bona fide claim of acquisition or ownership, since June However, even if the Iligan Property was subsumed by Proclamation No. 2143, the
12, 1945 immediately preceding the filing of the application for confirmation of title, same proclamation recognized private rights, which may have already attached, and
except when prevented by war or force majeure. These shall be conclusively presumed the rights of qualified free patent applicants.
to have performed all the conditions essential to a government grant and shall be
entitled to a certificate of title under the provisions of this chapter. (As amended by PD Whatever rights petitioners (and their predecessors-in-interest) may have had
1073.) over the Iligan property was, thus, not obliterated by Proclamation No. 2143. On the
contrary, the Proclamation itself facilitated compensation.
More importantly, there is documentary evidence to the effect that the Iligan
Property was not even within the area claimed by respondent.

Deputy Public Land Inspector Pio Lucero, Jr. noted that: That this land known
as Lot No. 5258, Cad. 292, Iligan Cadastre which portion was claimed also by the
Human Settlement and/or National Housing Authority; but the area applied for by
Leopoldo Delfin is outside the claim of the said agency as per certification issued dated
June 10, 1988.

The same letter likewise indicated that the Iligan Property was already
occupied by June 1945 and that it had even been released for agricultural purposes in
favor of its occupants. Accordingly, the Deputy Public Land Inspector recommended
the issuance of a patent in favor of petitioner Leopoldo Delfin.

Clearly then, petitioners acquired title over the Iligan Property pursuant to
Section 48 (b) of the Public Land Act. First, there is no issue that the Iligan Property
had already been declared to be alienable and disposable land. Respondent has
admitted this and Deputy Public Land Inspector Pio Lucero, Jr.'s letters to the Director
of Land attest to this. Second, although the Delfin Spouses' testimonial evidence and
tax declarations showed that their possession went only as far back as 1952, Deputy
Public Land Inspector Pio Lucero, Jr.'s letters to the Director of Land nevertheless attest
to a previous finding that the property had already been occupied as early as June
1945.

Having shown that the requisites of Section 48 (b) of the Public Land Act have
been satisfied and having established their rights to the Iligan Property, it follows that
petitioners must be compensated for its taking.

The right to be justly compensated whenever private property is taken for


public use cannot be disputed. Article III, Section 9 of the 1987 Constitution states that:
“Private property shall not be taken for public use without just compensation.”

Wherefore, petition is hereby granted.


HEIRS OF MARIO MALABANAN, (Represented by Sally A. Malabanan), SC: denied the petition because Malabanan failed to establish by sufficient evidence
petitioners, vs. REPUBLIC OF THE PHILIPPINES, respondent. possession and occupation of the property on his part and on the part of his
predecessors-in interest since June 12, 1945, or earlier.
FACTS:
Petitioner and the Republic filed their respective motion for reconsideration.
The property subject of the application of land registration is a land situated in
Cavite denominated as lot 9864-A. On February 20, 1998, Malabanan who had Petitioners: petitioners submit that the mere classification of the land as alienable or
purchased the property from Eduardo Velazco, filed an application for land registration disposable should be deemed sufficient to convert it into patrimonial property of the
covering the property in the RTC Cavite claiming that the property formed part of the State. Relying on the rulings in Spouses de Ocampo v. Arlos, Menguito v. Republic and
alienable and disposable land of the public domain, and that he and his predecessors- Republic v. T.A.N. Properties, Inc., they argue that the reclassification of the land as
in-interest had been in open, continuous, uninterrupted, public and adverse possession alienable or disposable opened it to acquisitive prescription under the Civil Code; that
and occupation of the land for more than 30 years, thereby entitling him to the judicial Malabanan had purchased the property from Eduardo Velazco believing in good faith
confirmation of his title. that Velazco and his predecessors-in-interest had been the real owners of the land with
the right to validly transmit title and ownership thereof; that consequently, the ten-year
To prove his claim Malabanan presented a certification issued by the CENRO period prescribed by Article 1134 of the Civil Code, in relation to Section 14 (2) of the
(Community Environment and Natural Resources Office) of the DENR that the subject Property Registration Decree, applied in their favor; and that when Malabanan filed the
parcel of land is verified to be within the Alienable or Disposable land per Land application for registration on February 20, 1998, he had already been in possession of
Classification Map No. 3013 established under Project No. 20-A and approved as such the land for almost 16 years reckoned from 1982, the time when the land was
under FAO 4-1656 on March 15, 1982. declared alienable and disposable by the State.

RTC: After trial, RTC rendered decision granting Malabanan’s application. Republic: The Republic seeks the partial reconsideration in order to obtain a
clarification with reference to the application of the rulings in Naguit and Herbieto. It
The OSG appealed to the CA arguing that petitioner had failed to prove that contends that the decision has enlarged, by implication, the interpretation of Section 14
the property belonged to the alienable and disposable land of the public domain. (1) of the Property Registration Decree through judicial legislation. It reiterates its view
that an applicant is entitled to registration only when the land subject of the application
CA: CA reversed the RTC decision citing the ruling in Republic v. Herbieto (Herbieto), had been declared alienable and disposable since June 12, 1945 or earlier.
the CA declared that under Section 14 (1) of the Property Registration Decree, any
period of possession prior to the classification of the land as alienable and disposable ISSUE: W/N the respective motions for reconsideration should be given merit?
was inconsequential and should be excluded from the computation of the period of
possession. Noting that the CENRO-DENR certification stated that the property had HELD:
been declared alienable and disposable only on March 15, 1982, Velazco's possession
prior to March 15, 1982 could not be tacked for purposes of computing Malabanan's No. Land which is immovable property may be classified either as public
period of possession. dominion or of private ownership.

During pendency of the appeal, Malabanan died and his heirs filed this 1. Land is public dominion if it either: (a) is intended for public use; or (b)
petition. belongs to the State, without being for public use, and is intended for
some public service or for the development of the national wealth."
Petitioners: Petitioners assert that the ruling in Republic v. Court of Appeals and
Corazon Naguit (Naguit) remains the controlling doctrine especially if the property Land that belongs to the state but which is not or is no longer intended for
involved is agricultural land. In Naguit, Court ruled that any possession of agricultural public use, for some public service or for the development of the national
land prior to its declaration as alienable and disposable could be counted in the wealth, is patrimonial property; it is property owned by the State in its
reckoning of the period of possession to perfect title under the Public Land Act private capacity.
(Commonwealth Act No. 141) and the Property Registration Decree.
2. Land that is other than part of the patrimonial property of the State,
They relied also on the case of Republic v. Tan to support their argument that provinces, cities and municipalities is of private ownership if it belongs
the property had been ipso jure converted into private property by reason of the open, to a private individual.
continuous, exclusive and notorious possession by their predecessors-in-interest of an
alienable land of the public domain for more than 30 years. According to them, what
was essential was that the property had been "converted" into private property through Pursuant to Regalian doctrine (jura regalia) all lands of the public domain
prescription at the time of the application without regard to whether the property sought belong to the State. This means that the State is the source of any asserted right to
to be registered was previously classified as agricultural land of the public domain. ownership of land, and is charged with the conservation of such patrimony. All lands
not appearing to be clearly under private ownership are presumed to belong to the
State. Also, public lands remain part of the inalienable land of the public domain unless preceding the filing of the application for confirmation of title, except when
the State is shown to have reclassified or alienated them to private persons. prevented by war or force majeure. These shall be conclusively presumed to
have performed all the conditions essential to a government grant and shall
A. Whether or not land of the public domain is alienable and disposable primarily be entitled to a certificate of title under the provisions of this chapter. (As
rests on the classification of public lands made under the Constitution. amended by PD 1073.)

Under the 1935 Constitution, lands of the public domain were classified into Note that Section 48 (b) of the Public Land Act used the words "lands of the
three, namely, agricultural, timber and mineral. public domain" or "alienable and disposable lands of the public domain" to clearly
signify that lands otherwise classified are outside the coverage of the Public Land Act.
The 1987 Constitution adopted the classification under the 1935 Constitution
into agricultural, forest or timber, and mineral, but added national parks. Agricultural The use of the descriptive phrase "alienable and disposable" further limits the
lands may be further classified by law according to the uses to which they may be coverage of Section 48 (b) to only the agricultural lands of the public domain as set
devoted. The identification of lands according to their legal classification is done forth in Article XII, Section 2 of the 1987 Constitution.
exclusively by and through a positive act of the Executive Department.
Bearing in mind such limitations under the Public Land Act, the applicant must
Based on the foregoing, the Constitution places a limit on the type of public satisfy the following requirements in order for his application to come under Section 14
land that may be alienated. Under Section 2, Article XII of the 1987 (1) of the Property Registration:
Constitution, only agricultural lands of the public domain may be alienated; all other
natural resources may not be.
1. The applicant, by himself or through his predecessor-in-interest, has been in
Alienable and disposable lands of the State fall into two categories, to wit: possession and occupation of the property subject of the application;
2. The possession and occupation must be open, continuous, exclusive, and
(a) patrimonial lands of the State, or those classified as lands of private ownership notorious;
under Article 425 of the Civil Code, without limitation; and 3. The possession and occupation must be under a bona fide claim of
(b) lands of the public domain, or the public lands as provided by the Constitution, acquisition of ownership;
but with the limitation that the lands must only be agricultural. 4. The possession and occupation must have taken place since June 12, 1945,
or earlier; and
B. Disposition of Alienable Public Lands 5. The property subject of the application must be an agricultural land of the
public domain.
Under Sec. 11 of CA 141 (Public Land Act), Public lands suitable for
agricultural purposes can be disposed of only as follows, and not otherwise: Taking into consideration that the Executive Department is vested with the
authority to classify lands of the public domain, Section 48 (b) of the Public Land Act,
(1) For homestead settlement; in relation to Section 14 (1) of the Property Registration Decree, presupposes that the
(2) By sale; land subject of the application for registration must have been already classified as
(3) By lease; and agricultural land of the public domain in order for the provision to apply.
(4) By confirmation of imperfect or incomplete titles:
(a) By judicial legalization; or Thus, absent proof that the land is already classified as agricultural land of the
(b) By administrative legalization (free patent). public domain, the Regalian Doctrine applies, and overcomes the presumption that the
land is alienable and disposable as laid down in Section 48 (b) of the Public Land Act.
And on Sec. 48 (b) of the said Act provides that:
If a public land is classified as no longer intended for public use or for the
Sec. 48. The following-described citizens of the Philippines, occupying lands development of national wealth by declaration of Congress or the President, thereby
of the public domain or claiming to own any such lands or an interest converting such land into patrimonial or private land of the State, the applicable
therein, but whose titles have not been perfected or completed, may apply to provision concerning disposition and registration is no longer Section 48 (b) of the
the Court of First Instance of the province where the land is located for Public Land Act but the Civil Code, in conjunction with Section 14 (2) of the Property
confirmation of their claims and the issuance of a certificate of title thereafter, Registration Decree. As such, prescription can now run against the State.
under the Land Registration. expressly requires possession by a Filipino
citizen of the land since June 12, 1945, or earlier refers to wit: To reiterate, then, the petitioners failed to present sufficient evidence to
establish that they and their predecessors-in-interest had been in possession of the
(b). those who "have been in open, continuous, exclusive, and notorious land since June 12, 1945. Without satisfying the requisite character and period of
possession and, occupation of agricultural lands of the public domain, under possession — possession and occupation that is open, continuous, exclusive, and
a bona fide claim of acquisition or ownership, since June 12, 1945 immediately
notorious since June 12, 1945, or earlier — the land cannot be considered ipso jure
converted to private property even upon the subsequent declaration of it as alienable
and disposable. Prescription never began to run against the State, such that the land
has remained ineligible for registration under Section 14 (1) of the Property Registration
Decree. Likewise, the land continues to be ineligible for land registration under Section
14 (2) of the Property Registration Decree unless Congress enacts a law or the
President issues a proclamation declaring the land as no longer intended for public
service or for the development of the national wealth.

WHEREFORE, the Court DENIES the petitioners' Motion for Reconsideration


and the respondent's Partial Motion for Reconsideration for their lack of merit.

In case asked:

To Sum Up:

1. As a general rule and pursuant to the Regalian Doctrine, all lands of the public
domain belong to the State and are inalienable. Lands that are not clearly
under private ownership are also presumed to belong to the State and,
therefore, may not be alienated or disposed;
2. The following are excepted from the general rule, to wit:
a. Agricultural lands of the public domain are rendered alienable and
disposable through any of the exclusive modes enumerated under Section 11
of the Public Land Act. If the mode is judicial confirmation of imperfect title
under Section 48 (b) of the Public Land Act, the agricultural land subject of the
application needs only to be classified as alienable and disposable as of the
time of the application, provided the applicant's possession and occupation of
the land dated back to June 12, 1945, or earlier. Thereby, a conclusive
presumption that the applicant has performed all the conditions essential to a
government grant arises, and the applicant becomes the owner of the land by
virtue of an imperfect or incomplete title. By legal fiction, the land has already
ceased to be part of the public domain and has become private property.
(b) Lands of the public domain subsequently classified or declared as no
longer intended for public use or for the development of national wealth are
removed from the sphere of public dominion and are considered converted
into patrimonial lands or lands of private ownership that may be alienated or
disposed through any of the modes of acquiring ownership under the Civil
Code. If the mode of acquisition is prescription, whether ordinary or
extraordinary, proof that the land has been already converted to private
ownership prior to the requisite acquisitive prescriptive period is a condition
sine qua non in observance of the law (Article 1113, Civil Code) that property
of the State not patrimonial in character shall not be the object of prescription.
REPUBLIC OF THE PHILIPPINES, petitioner, vs. DAMIAN ERMITAÑO DE year possession requirement. This is in accord with the ruling in Almeda vs. Court of
GUZMAN, et. al, respondents. Appeals.

FACTS: While the Court acknowledge the CA’s finding that private respondents and
their predecessors-in-interest have been in possession of the subject land for 63 years
Conflicting applications for confirmation of imperfect title were filed by Norma at the time of the application, the Court is tied by the applicable laws and jurisprudence
Almanzor and private respondent Salvador De Guzman over parcels of land located in in giving practical relief to them. The fact remains that from the time the subject land
Silang, Cavite. was declared alienable until the time of their application, private respondents'
occupation thereof was only 26 yrs. and the 37 yrs. of possession prior to the release
The Following are undisputed facts: of the land as alienable absent the fact of declassification prior to the possession and
cultivation in good faith by petitioner, the property occupied by him remained classified
The subject parcels of land were forest lands released as agricultural land in as forest or timberland, which he could not have acquired by prescription.
1965 while the petition for confirmation of imperfect title was filed by private
respondents only in 1991. Thus, the period of occupancy of the subject parcels of land
from 1965 until the time the application was filed in 1991 was only twenty-six (26) years,
four (4) years short of the required thirty (30) year period possession requirement under
Sec. 14, P.D. 29 and R.A. No. 6940.

RTC: The RTC rendered judgment in favor of private respondents.

CA: It was affirmed on appeal by the Court of Appeals. It reasoned out that while it is
true that the land became alienable and disposable only in December, 1965, however,
records indicate that as early as 1928, Pedro Ermitaño, respondents’ predecessor-in-
interest, was already in possession of the property, cultivating it and planting various
crops thereon. It follows that respondents’ possession as of the time of the filing of the
petition in 1991 when tacked to Pedro Ermitaño's possession is 63 years or more than
the required years period of possession. The land, which is agricultural, has been
converted to private property.

Hence, this recourse of the Republic.

ISSUE: W/N the respondent the respondent’s possession of the subject property
complied with the law?

HELD:

No. The Court of Appeals' consideration of the period of possession prior to


the time the subject land was released as agricultural is in direct contravention of the
pronouncement in Almeda vs. Court of Appeals:

“… possession of the land while it was still inalienable forest land, or before it was
declared alienable and disposable land of the public domain on January 13, 1968, could
not ripen into private ownership, and should be excluded from the computation of the
30-year open and continuous possession in concept of owner required under Section
48(b) of Com. Act 141…

Unless and until the land classified as forest is released in an official


proclamation to that effect so that it may form part of the disposable lands of the public
domain, the rules on confirmation of imperfect title do not apply.”

In summary, therefore, prior to its declaration as alienable land in 1965, any


occupation or possession thereon cannot be considered in the counting of the thirty-
REPUBLIC OF THE PHILIPPINES, petitioner, vs. DIOSDADA I. GIELCZYK,
respondent. A. The respondent failed to completely prove that there was an expressed State
declaration that the properties in question are no longer intended for public
FACTS: use, public service, the development of the national wealth and have been
converted into patrimonial property, and to meet the period of possession
The respondent sought registration of two parcels of land located in Cebu and occupation required by law.
denominated s Lot Nos. 3135-A and 3136-A.
Under Sec. 14 of Property Registration Decree the following may apply for
She claims that she is the owner of the said lands by virtue of deeds of registration of title to land:
absolute sale and that that she had been in open, complete, continuous, and peaceful
possession in the concept of an owner over said parcels of land up to the present time 1. Those who by themselves or through their predecessors-in-interest have
for more than 30 years, including the possession of her predecessors-in-interest. been in open, continuous, exclusive and notorious possession and
occupation of alienable and disposable lands of the public domain under a
On her testimony she avers that she acquired the subject lot 3136-A and 3135- bona fide claim of ownership since June 12, 1945, or earlier.
A from Constancio and Luisa Ceniza respectively, she was never delinquent to pay the 2. Those who have acquired ownership of private lands by prescription under
real property taxes from 1948 to present, the said lots are alienable and disposable and the provision of existing laws.
are not covered by subsisting public land application and the respondent and her 3. Those who have acquired ownership of private lands or abandoned river
respective predecessors-in-interest had been in possession of Lot No. 3135-A and Lot beds by right of accession or accretion under the existing laws.
No. 3136-A for more than 40 years in the concept of an owner, exclusively, completely, 4. Those who have acquired ownership of land in any other manner provided
continuously, publicly, peacefully, notoriously and adversely, and no other person has for by law.
claimed ownership over the same land.
A closer scrutiny will show that the questioned decision was based on PD No.
The petitioner filed its opposition stating that neither the respondent nor her 1529, Section 14(2).
predecessors-in-interest have been in open, continuous, exclusive, and notorious
possession and occupation of the land in question since June 12, 1945 or prior thereto In Heirs of Mario Malabanan v. Republic, the Court further clarified the
and muniments of title and/or the tax declarations and tax payment receipts of the difference between Section 14 (1) and Section 14 (2) of P.D. No. 1529:
respondent attached to or alleged in the application do not constitute competent and
sufficient evidence of a bona fide acquisition of the land or of their open, continuous, 1. The former refers to registration of title on the basis of possession, while
exclusive and notorious possession and occupation thereof in the concept of an owner the latter entitles the applicant to the registration of his property on the
since June 12, 1945, or prior thereto and the parcel of land applied for is a portion of basis of prescription.
the public domain belonging to the petitioner and that the said parcel is not subject to
private appropriation. 2. Registration under the first mode is extended under the aegis of the P.D.
No. 1529 and the Public Land Act (PLA) while under the second mode is
RTC: RTC rendered decision in favor of the respondents. made available both by P.D. No. 1529 and the Civil Code.

CA: On appeal, the same is affirmed by the CA. 3. Moreover, under Section 48 (b) of the PLA, as amended by Republic Act
No. 1472, the 30-year period is in relation to possession without regard
Hence this petition. to the Civil Code, while under Section 14 (2) of P.D. No. 1529, the 30-
year period involves extraordinary prescription under the Civil Code,
Petitioner: particularly Article 1113 in relation to Article 1137.

1. The petitioner argued that the respondent failed to show proof of an expressed In the instant case the respondent has filed certification from CENRO that the
State declaration that the properties in question are no longer intended for subject lots were found to be within Alienable and Disposable Block I of Land
public use, public service, the development of the national wealth or have Classification Project No. 28 per L. C. Map No. 2545 of Consolacion, Cebu certified
been converted into patrimonial property. under Forestry Administrative Order No. 4-1063 dated September 1, 1965.
2. petitioner contends that the respondent failed to present specific acts of
ownership to substantiate the latter's claim of open, continuous, exclusive, However, following our ruling in Republic of the Philippines v. T.A.N.
notorious and adverse possession in the concept of an owner. Properties, Inc., this CENRO Certification by itself is insufficient to establish that a
public land is alienable and disposable. While the certification refers to Forestry
ISSUE: W/N the respondent may cause the registration of the said parcels of land? Administrative Order No. 4-1063 dated September 1, 1965, the respondent should
have submitted a certified true copy thereof to substantiate the alienable character of
HELD: No. the land.
2. On Lot 3136-A: As to Lot No. 3136-A, the deed of absolute sale showed that
Indeed, the respondent failed to meet the required period of possession and there were 14 coconut trees, eight (8) jackfruit trees, and a residential building,
occupation for purposes of prescription. From the time of the declaration on September which was actually possessed by the vendor Constancio Ceniza. Moreover, it
1, 1965 that the properties in question are purportedly alienable and disposable up to was only in Tax Declaration Nos. 29200, 04210 and 13275 where it was
the filing of the application of the respondent on July 17, 1995, the respondent and her declared that a residential building has been built in Lot No. 3136-A. 58 And
predecessors-in-interest had possessed and occupied the said properties for only 29 based on the records, Tax Declaration No. 29200, where the residential
years and 10 months, short of two months to complete the whole 30-year possession building was first indicated, is dated 1981. It may be said then that it was only
period. in 1981 when the respondent's predecessors-in-interest exercised specific
acts of dominion over Lot No. 3136-A, the period of which consists barely of
B. The respondent failed to present specific acts of ownership to substantiate 14 years. Thus, the respondent has not completed the required 30 years of
her claim of open, continuous, exclusive, notorious and adverse possession "open, continuous, exclusive and notorious possession and occupation."
in the concept of an owner.
Clearly, from the pieces of documentary and testimonial evidence, and
considering that the respondent did not present any other witness to support her claim,
the Court has no other recourse but to declare that she has not presented the premium
In Roman Catholic Bishop of Kalibo, Aklan v. Municipality of Buruanga, Aklan,
of evidence needed to award her title over the two parcels of land.
the court clarified what it meant when it said "open, continuous, exclusive and notorious
possession and occupation”

Actual possession of land consists in the manifestation of acts of dominion


over it of such a nature as a party would naturally exercise over his own property.

Possession is open when it is patent, visible, apparent, notorious and not


clandestine. It is continuous when uninterrupted, unbroken and not intermittent or
occasional; exclusive when the adverse possessor can show exclusive dominion over
the land and an appropriation of it to his own use and benefit; and notorious when it is
so conspicuous that it is generally known and talked of by the public or the people in
the neighborhood.

In the instant case, the respondent failed to show that she or her
predecessors-in-interest have exercised acts of dominion over the said parcels of land.
In fact, it was only the respondent who testified to substantiate her allegations in the
application. She did not present anyone else to support her claim of "open, continuous,
exclusive and notorious possession and occupation." Unfortunately, her testimony
simply made general declarations without further proof

From here testimony the Court can deduce that:

1. On lot 3135-A: besides intermittently paying the tax dues on Lot No. 3135-A,
the respndent did not exercise acts of dominion over it. Neither can the Court
give credence to the respondent's claim that her predecessors-in-interest had
exercised dominion over the property since the respondent failed to present
any witness who would substantiate her allegation. The pieces of
documentary evidence, specifically the tax declarations and the deeds of
absolute sale, can neither be relied upon because the same revealed no
indication of any improvement that would have the Court conclude that the
respondent exercised specific acts of dominion. For instance, the deed of
absolute sale simply said that the improvements on Lot No. 3135-A consisted
of two (2) coconut trees, one (1) mango tree, one (1) caimito tree and one (1)
jackfruit tree. The tax declarations have not shown any indication supporting
the respondent's claim that she exercised specific acts of dominion.
FLORENTINO W. LEONG AND ELENA LEONG, ET AL., Petitioners, vs. EDNA C. On April 1, 1997,21 Edna filed a complaint22 for recovery of possession against Elena
SEE, Respondent. G.R. No. 194077, December 3, 2014 and the other relatives of the Leong ex-spouses.23

Doctrine: Factual findings of lower courts are generally deemed conclusive and The complaint alleged that in 1995 after the fire had razed the building on the land,
binding upon this court.1 In any event, "even if the procurement of title was tainted with Elena erected makeshift houses on the land without Carmelita’s knowledge or
fraud and misrepresentation, such defective title may be the source of a completely consent.24
legal and valid title in the hands of an innocent purchaser for value."

FACTS: This case originated from two civil complaints involving the sale of a parcel of In response, Elena alleged the title’s legal infirmity for lack of Florentino's conformity to
land in favor of respondent Edna C. See (Edna). its sale.25 She argued that Carmelita's noncompliance with the proviso in the property
agreement — that the Quiapo property "may not be alienated without Florentino first
Petitioners assails the Decision of the Court of Appeals’ (a) May 19, 2010 decision obtaining a clean title over the Malabon property" 26 — annulled the transfer to Edna.
affirming in toto the trial court's July 9, 2008 decision granting Edna possession and
ownership over the land upon finding her to be a buyer in good faith and for value, and RTC: Edna See is granted possession and ownership over the subject property
(b) August 25, 2010 resolution denying reconsideration.
CA: affirmed in toto the trial court’s decision

The spouses Florentino Leong (Florentino) and Carmelita Leong (Carmelita) used to ISUUE: Whether respondent Edna C. See is a buyer in good faith and for value.
own the property located at No. 539–41 Z.P. De Guzman Street, Quiapo, Manila.6
RULING: YES. The Torrens system was adopted to "obviate possible conflicts of title
Petitioner Elena Leong (Elena) is Florentino's sister-in-law.7 She had stayed with her by giving the public the right to rely upon the face of the Torrens certificate and to
in-laws on the property rental-free for over two decades until the building they lived in dispense, as a rule, with the necessity of inquiring further." 57
was razed by fire.8 They then constructed makeshift houses, and the rental-free
arrangement continued.9 Florentino and Carmelita immigrated to the United States and
eventually had their marriage dissolved in Illinois.10 A provision in their marital One need not inquire beyond the four corners of the certificate of title when dealing with
settlement agreement states that"Florentino shall convey and quitclaim all of his right, registered property.58Section 44 of Presidential Decree No. 1529 known as the
title and interest in and to 540 De Guzman Street, Manila, Philippines . . . to Property Registration Decree recognizes innocent purchasers in good faith for value
Carmelita."11 and their right to rely on a clean title:

The Court of Appeals found that "[a]pparently intercalated in the lower margin of page Section 44. Statutory liens affecting title. - Every registered owner receiving a certificate
12 of the instrument was a long-hand scribbling of a proviso, purporting to be a footnote of title in pursuance of a decree of registration, and every subsequent purchaser of
remark":12 Neither party shall evict or charge rent to relatives of the parties, or convey registered land taking a certificate of title for value and in good faith, shall hold the same
title, until it has been established that Florentino has clear title to the Malabon property. free from all encumbrances except those noted in said certificate and any of the
following encumbrances which may be subsisting, namely:
Clear title to be established by the attorneys for the parties or the ruling of a court of
competent jurisdiction. In the event Florentino does not obtain clear title, this court
reserves jurisdiction to reapportion the properties or their values to effect a 50-50 First. Liens, claims or rights arising or existing under the laws and Constitution of the
division of the value of the 2 remaining Philippine properties. 13 Philippines which are not by law required to appear of record in the Registry of Deeds
in order to be valid against subsequent purchasers or encumbrances of record.
On November 14, 1996,14 Carmelita sold the land to Edna.15 In lieu of Florentino's
signature of conformity in the deed of absolute sale, Carmelita presented to Edna and Second. Unpaid real estate taxes levied and assessed within two years immediately
her father, witness Ernesto See, a waiver of interest notarized on March 11, 1996 in preceding the acquisition of any right over the land by an innocent purchaser for value,
Illinois.16 In this waiver, Florentino reiterated his quitclaim over his right, title, and without prejudice to the right of the government to collect taxes payable before that
interest to the land.17 Consequently, the land’s title, covered by TCT No. 231105, was period from the delinquent taxpayer alone.
transferred to Edna's name.
Third. Any public highway or private way established or recognized by law, or any
Edna was aware of the Leong relatives staying in the makeshift houses on the government irrigation canal or lateral thereof, if the certificate of title does not state that
land.19 Carmelita assured her that her nieces and nephews would move out, but the boundaries of such highway or irrigation canalor lateral thereof have been
demands to vacate were unheeded.20 determined.
Fourth. Any disposition of the property or limitation on the use thereof by virtue of, or Even assuming the procurement of title was tainted with fraud and misrepresentation,
pursuant to, Presidential Decree No. 27 or any other law or regulations on agrarian "such defective title may still be the source of a completely legal and valid title in the
reform.59 (Emphasis supplied) hands of an innocent purchaser for value."80

An innocent purchaser for value refers to someone who "buys the property of another Respondent, an innocent purchaser ingood faith and for value with title in her name,
without notice that some other person has a right to or interest in it, and who paysa full has a better right to the property than Elena. Elena’s possession was neither adverse
and fair price at the time of the purchase or before receiving any notice of another to nor in the concept of owner.81
person’s claim."60 One claiming to be an innocent purchaser for value has the burden
of proving such status.61 Article 428 of the Civil Code provides:

The protection of innocent purchasers in good faith for value grounds on the social Art. 428. The owner has the right toenjoy and dispose of a thing, without other
interest embedded in the legal concept granting indefeasibility of titles. Between the limitations than those established by law. The owner has also a right of action against
third party and the owner, the latter would be more familiar with the history and status the holder and possessor of the thing inorder to recover it. 82
of the titled property. Consequently, an owner would incur less costs to discover alleged
invalidities relating to the property compared to a third party. Such costs are, thus,
better borne by the owner to mitigate costs for the economy, lessen delays in Thus, respondent had every right to pursue her claims as she did.
transactions, and achieve a less optimal welfare level for the entire society.

The question of whether Florentino and Carmelita were already American citizens at
the time of the property’s sale to Edna — thus no longer covered by our laws relating
to family rights and duties77 — involves a factual question outside the ambit of a petition
for review on certiorari. In any event, respondent exerted due diligence when she
ascertained the authenticity of the documents attached to the deed of sale such as the
marital settlement agreement with Florentino’s waiver of interest over the property. She
did not rely solely on the title. She even went to the Registry of Deeds to verify the
authenticity of the title.78 These further inquiries were considered by the lower courts in
finding respondent to be an innocent purchaser in good faith and for value.

Lastly, an allegation of fraud must be substantiated. Rule 8, Section 5 of the Rules of


Court provides:

SEC. 5. Fraud, mistake, condition of the mind. – In all averments of fraud or mistake,
the circumstances constituting fraud or mistake must be stated with particularity.Malice
intent, knowledge or other condition of the mind of a person may be averred generally.
(Emphasis supplied)

In petitioners’ memorandum before this court, they mentioned the rule of fraud as an
exception to the indefeasibility of title principle, but failed to substantiate their allegation
by immediately concluding as follows:

Petitioners beg to disagree with the ruling of the Honorable Trial Court and the
Honorable Court of Appeals.1âwphi1Respondent Edna See is not a buyer in good faith.
The ruling that every person can rely on the correctness of the certificate of title and
that the buyer need not go beyond the four corners of the title to determine the condition
of the property is not absolute and admits of exception. As held in the case of Remegia
Feliciano vs. Sps. Zaldivar, G.R. No. 162593, 2006 Sep 26 the principle of indefeasibilty
of a Torrens title does not apply where fraud attended the issuance of the title. The
Torrens title does not furnish a shield for fraud. As such, a title issued based on void
documents may be annulled.79 (Emphasis in the original removed)
HEIRS OF BIENVENIDO AND ARACELI TANYAG, namely: ARTURO TANYAG, As to Lot 2, petitioners averred that it was sold by Agueda Dinguinbayan to Araceli
AIDA T. JOCSON AND ZENAIDA T. VELOSO, Petitioners, Tanyag under Deed of Sale executed on October 22, 1968. Thereupon, petitioners took
vs. SALOME E. GABRIEL, NESTOR R. GABRIEL, LUZ GABRIEL-ARNEDO married possession of said property and declared the same for tax purposes as shown by TD
to ARTURO ARNEDO, NORA GABRIEL-CALINGO married to FELIX CALINGO, Nos. 11361, 3395, 120-014-00482, 120-00-014-20-002-000, C-014-00180 and D-014-
PILAR M. MENDIOLA, MINERVA GABRIEL-NATIVIDAD married to EUSTAQUIO 00182 issued for the years 1969, 1974, 1979, 1985, 1991 and 1994. 8 Petitioners
NATIVIDAD, and ERLINDA VELASQUEZ married to HERMINIO VELASQUEZ, claimed to have continuously, publicly, notoriously and adversely occupied both Lots 1
and 2 through their caretaker Juana Quinones 9 ; they fenced the premises and
Respondents. G.R. No. 175763, April 11, 2012
introduced improvements on the land.10

Doctrine: Registration of a piece of land under the Torrens System does not create or vest title,
because it is not a mode of acquiring ownership. A certificate of title is merely an evidence of Sometime in 1979, Jose Gabriel, father of respondents, secured TD No. 120-014-
ownership or title over the particular property described therein.33 Thus, notwithstanding the 01013 in his name over Lot 1 indicating therein an increased area of 1,763 square
indefeasibility of the Torrens title, the registered owner may still be compelled to reconvey the meters. Said tax declaration supposedly cancelled TD No. 6425 over Lot 1 and
registered property to its true owners. The rationale for the rule is that reconveyance does not set contained the following inscription11 :
aside or re-subject to review the findings of fact of the Bureau of Lands. In an action for
reconveyance, the decree of registration is respected as incontrovertible. What is sought instead
is the transfer of the property or its title which has been wrongfully or erroneously registered in Note: Portions of this Property is Also Declared
another person’s name, to its rightful or legal owner, or to the one with a better right. in the name of Araceli C. Tanyag under
T.D.#120-014-00858 686 sq. m.
An action for annulment of title or reconveyance based on fraud is imprescriptible where
the plaintiff is in possession of the property subject of the acts. The totality of the Also inscribed on TD No. 120-014-0085812 (1979) in the name of Araceli Tanyag
evidence on record established that it was petitioners who are in actual possession of covering Lot 1 are the following:
the subject property; respondents merely insinuated at occasional visits to the land.
However, for an action for reconveyance based on fraud to prosper, this Court has held This property is also covered by T.D. #120-014-01013
that the party seeking reconveyance must prove by clear and convincing evidence his in the name of Jose P. Gabriel
title to the property and the fact of fraud. 1-8-80
which notation was carried into the 1985, 1990 and 1991 tax declarations, all in the
FACTS: Subject of controversy are two adjacent parcels of land located at Ruhale, name of Araceli Tanyag.
Barangay Calzada, Municipality of Taguig (now part of Pasig City, Metro Manila). The
first parcel ("Lot 1") was originally declared in the name of Jose Gabriel under Tax On March 20, 2000, petitioners instituted Civil Case No. 67846 alleging that
Declaration (TD) Nos. 1603 and 6425 issued for the years 1949 and 1966, while the respondents never occupied the whole 686 square meters of Lot 1 and fraudulently
second parcel ("Lot 2") was originally declared in the name of Agueda Dinguinbayan caused the inclusion of Lot 2 in TD No. 120-014-01013 such that Lot 1 consisting of
under TD Nos. 6418 and 9676 issued for the years 1966 and 1967. 4 For several years, 686 square meters originally declared in the name of Jose Gabriel was increased to
these lands lined with bamboo plants remained undeveloped and uninhabited. 1,763 square meters. They contended that the issuance of OCT No. 1035 on October
28, 1998 over the subject land in the name of respondents heirs of Jose Gabriel was
Petitioners claimed that Lot 1 was owned by Benita Gabriel, sister of Jose Gabriel, as null and void from the beginning.13 On the other hand, respondents asserted that
part of her inheritance as declared by her in a 1944 notarized instrument ("Affidavit of petitioners have no cause of action against them for they have not established their
Sale") whereby she sold the said property to spouses Gabriel Sulit and Cornelia ownership over the subject property covered by a Torrens title in respondents’ name.
Sanga. They further argued that OCT No. 1035 had become unassailable one year after its
issuance and petitioners failed to establish that it was irregularly or unlawfully procured.

Lot 1 allegedly came into the possession of Benita Gabriel’s own daughter, Florencia
RTC: In its decision, the trial court dismissed the complaint as well as the counterclaim,
Gabriel Sulit, when her father-in-law Gabriel Sulit gave it to her as part of inheritance of
his son, Eliseo Sulit who was Florencia’s husband. Florencia Sulit sold the same lot to holding that petitioners failed to establish ownership of the subject property and finding
Bienvenido S. Tanyag, father of petitioners, as evidenced by a notarized deed of sale the respondents to be the declared owners and legal possessors. It likewise ruled that
dated October 14, 1964.6 Petitioners then took possession of the property, paid the real petitioners were unable to prove by preponderance of evidence that respondents
estate taxes due on the land and declared the same for tax purposes, as shown by TD acquired title over the property through fraud and deceit.
No. 11445 issued in 1969 in the name of Bienvenido’s wife, Araceli C. Tanyag; TD No.
11445 cancelled TD No. 6425 in the name of Jose Gabriel. TD Nos. 3380 and 00486 CA: affirmed the trial court’s ruling
also in the name of Araceli Tanyag were issued in the years 1974 and 1979.7
ISSUE: Did these acts of respondents effectively interrupt the possession of petitioners
for purposes of prescription?
RULING: NO. In the case of Heirs of Marcelina Azardon-Crisologo v. Rañon44 this better right to the property must first fix the identity of the land he is claiming by
Court citing Article 1123 of the Civil Code45 held that civil interruption takes place with describing the location, area and boundaries thereof. 46 In this case, petitioners failed to
the service of judicial summons to the possessor and not by filing of a mere Notice of identify Lot 2 by providing evidence of the metes and bounds thereof, so that the same
Adverse Claim. Thus: may be compared with the technical description contained in OCT No. 1035, which
would have shown whether Lot 2 consisting of 147 square meters was erroneously
Article 1123 of the Civil Code is categorical. Civil interruption is produced by judicial included in respondents’ title. The testimony of Agueda Dinguinbayan’s son would not
summons to the possessor. Moreover, even with the presence of judicial summons, suffice because said witness merely stated the boundary owners as indicated in the
Article 1124 sets limitations as to when such summons shall not be deemed to have 1966 and 1967 tax declarations of his mother. On his part, Arturo Tayag claimed that
been issued and shall not give rise to interruption, to wit: 1) if it should be void for lack he had the lots surveyed in the 1970s in preparation for the consolidation of the two
of legal solemnities; 2) if the plaintiff should desist from the complaint or should allow parcels. However, no such plan was presented in court.
the proceedings to lapse; or 3) if the possessor should be absolved from the complaint.
In this case, the CA was mistaken in concluding that petitioners have not acquired any
Both Article 1123 and Article 1124 of the Civil Code underscore the judicial character right over the subject property simply because they failed to establish Benita Gabriel’s
of civil interruption.1âwphi1 For civil interruption to take place, the possessor must have title over said property. The appellate court ignored petitioners’ evidence of possession
received judicial summons. None appears in the case at bar. The Notice of Adverse that complies with the legal requirements of acquiring ownership by prescription.
Claim which was filed by petitioners in 1977 is nothing more than a notice of claim which
did not effectively interrupt respondents’ possession. Such a notice could not have Acquisitive prescription is a mode of acquiring ownership by a possessor through the
produced civil interruption. We agree in the conclusion of the RTC, which was affirmed requisite lapse of time. In order to ripen into ownership, possession must be in the
by the Court of Appeals, that the execution of the Notice of Adverse Claim in 1977 did concept of an owner, public, peaceful and uninterrupted. 39Possession is open when it
not toll or interrupt the running of the prescriptive period because there remains, as yet, is patent, visible, apparent, notorious and not clandestine.40 It is continuous when
a necessity for a judicial determination of its judicial validity. What existed was merely uninterrupted, unbroken and not intermittent or occasional; exclusive when the adverse
a notice. There was no compliance with Article 1123 of the Civil Code. What is striking possessor can show exclusive dominion over the land and an appropriation of it to his
is that no action was, in fact, filed by petitioners against respondents. As a own use and benefit; and notorious when it is so conspicuous that it is generally known
consequence, no judicial summons was received by respondents. As aptly held by the and talked of by the public or the people in the neighborhood. The party who asserts
Court of Appeals in its affirmance of the RTC’s ruling, the Notice of Adverse Claim ownership by adverse possession must prove the presence of the essential elements
cannot take the place of judicial summons which produces the civil interruption provided of acquisitive prescription.41
for under the law. In the instant case, petitioners were not able to interrupt respondents’
adverse possession since 1962. The period of acquisitive prescription from 1962 On the matter of prescription, the Civil Code provides:
continued to run in respondents’ favor despite the Notice of Adverse Claim. (Emphasis Art. 1117. Acquisitive prescription of dominion and other real rights may be ordinary or
supplied.) extraordinary.

Ordinary acquisitive prescription requires possession of things in good faith and with just title for
From 1969 until the filing of this complaint by the petitioners in March 2000, the latter
the time fixed by law.
have been in continuous, public and adverse possession of the subject land for 31
years. Having possessed the property for the period and in the character required by Art. 1134. Ownership and other real rights over immovable property are acquired by ordinary
law as sufficient for extraordinary acquisitive prescription, petitioners have indeed prescription through possession of ten years.
acquired ownership over the subject property. Such right cannot be defeated by
respondents’ acts of declaring again the property for tax purposes in 1979 and obtaining Art. 1137. Ownership and other real rights over immovables also prescribe through uninterrupted
a Torrens certificate of title in their name in 1998. adverse possession thereof for thirty years, without need of title or of good faith. (Emphasis
supplied.)

This notwithstanding, we uphold petitioners’ right as owner only with respect to Lot 1
consisting of 686 square meters. Petitioners failed to substantiate their claim over Lot Petitioners’ adverse possession is reckoned from 1969 with the issuance of TD No.
2 by virtue of a deed of sale from the original declared owner, Agueda Dinguinbayan. 1145 in the name of Araceli Tanyag, which tax declaration cancelled TD No. 6425 in
Respondents asserted that the 147 square meters covered by the tax declarations of the name of Jose Gabriel.42 It is settled that tax receipts and declarations are prima
Dinguinbayan being claimed by petitioners is not the same lot included in OCT No. facie proofs of ownership or possession of the property for which such taxes have been
1035. paid. Coupled with proof of actual possession of the property, they may become the
basis of a claim for ownership.43Petitioners’ caretaker, Juana Quinones, has since lived
in a nipa hut, planted vegetables and tended a piggery on the land. Aside from paying
Under Article 434 of the Civil Code, to successfully maintain an action to recover the taxes due on the property, petitioners also exercised other acts of ownership such as
ownership of a real property, the person who claims a better right to it must prove two selling the 468-square meter portion to Sta. Barbara who had constructed thereon a
(2) things: first, the identity of the land claimed; and second, his title thereto. In regard nine-door apartment building.
to the first requisite, in an accion reinvindicatoria, the person who claims that he has a
GERARDO MENDOZA, TRINIA and IYLENE all surnamed MENDOZA, Petitioners, Also, a writ of possession may be issued not only against the person who has been
vs. SOLEDAD SALINAS, Respondent. G.R. No. 152827, February 6, 2007 defeated in a registration case but also against anyone unlawfully and adversely
occupying the land or any portion thereof during the land registration proceedings up
to the issuance of the final decree,10 and it is the duty of the registration court to issue
Doctrine: A writ of possession may be issued under the following instances: (1) land
said writ when asked for by the successful claimant.11
registration proceedings under Sec. 17 of Act No. 496; (2) judicial foreclosure, provided
the debtor is in possession of the mortgaged realty and no third person, not a party to
the foreclosure suit, had intervened; and (3) extrajudicial foreclosure of a real estate Based on these tenets, the issuance of a writ of possession, therefore, is clearly a
mortgage under Sec. 7 of Act No. 3135 as amended by Act No. 4118. ministerial duty of the land registration court. Such ministerial duty, however, ceases to
be so with particular regard to petitioners who are actual possessors of the property
under a claim of ownership. Actual possession under claim of ownership raises a
The issuance of a writ of possession, therefore, is clearly a ministerial duty of the land
disputable presumption of ownership. This conclusion is supported by Article 433 of the
registration court. Such ministerial duty, however, ceases to be so with particular regard Civil Code, which provides: Actual possession under claim of ownership raises a
to petitioners who are actual possessors of the property under a claim of ownership. disputable presumption of ownership. The true owner must resort to judicial process for
Actual possession under claim of ownership raises a disputable presumption of the recovery of the property.
ownership.
Under said provision, one who claims to be the owner of a property possessed by
FACTS: A parcel of land situated in the Barrio of Barretto, Municipality of Olongapo, another must bring the appropriate judicial action for its physical recovery. The term
Province of Zambales, Island of Luzon by Public Land claimed by C. Panaligan. "judicial process" could mean no less than an ejectment suit or reinvindicatory action,
Petitioners opposed respondent's application for the issuance of a writ of possession in which the ownership claims of the contending parties may be properly heard and
claiming that they were not oppositors/parties to the registration case and they have adjudicated.12
been in actual physical possession of the property since 1964. The RTC, however,
rejected their arguments and granted respondent's application for the issuance of a writ
It is noted that there already exists a final and executory decision disregarding
of possession per herein assailed Order.
respondent's claim for possession over the property. In a Decision dated January 21,
2002, rendered by the Municipal Trial Court in Cities (MTCC) of Olongapo City, Branch
PETITIONER: the RTC erred in issuing the writ of possession and acted with grave 3, in Civil Case No. 4643, an action for unlawful detainer filed by respondent and her
abuse of discretion amounting to lack and excess of jurisdiction. 3 Petitioners reiterate spouse against petitioners and several other occupants of the property, the case
their argument that they cannot be ousted of their possession of the property, having against petitioners was dismissed by the MTCC for lack of cause of action.13 While the
been in actual possession of the property since 1964, as evidenced by petitioner MTCC Decision was appealed by the other defendants, respondent and her spouse
Gerardo C. Mendoza's Sales Application made in January 1986 over the following manifested that they will not appeal the decision and, instead, will file for a writ of
property possession in LRC Case No. N-04-0-97.

RESPONDENT: present petition should be dismissed, arguing that the petition should Note should also be made that petitioners registered their opposition to respondent's
have been initially with the Court of Appeals, based on the principle of hierarchy of application for the issuance of a writ of possession and apprised the RTC of their actual,
courts, and that the general order of default on October 8, 1998 issued by the RTC peaceful, physical and uninterrupted possession since 1964, 14 including therein
binds them and personal notice was not necessary. documents supporting their claim, consisting of Gerardo C. Mendoza's Sales
Application made on January 1986 and a Declaration of Real Property for the years
ISSUE: Whether or not the issuance of the writ of possession by the RTC is proper. 1976 and 1985, among others.15 The RTC, nevertheless, disregarded their opposition
and, instead, relied on the ruling in Serra Serra v. Court of Appeals [195 SCRA
RULING: YES. A writ of possession may be issued under the following instances: (1) 482],16 that a writ of possession may be issued in a land registration proceeding.
land registration proceedings under Sec. 17 of Act No. 496; (2) judicial foreclosure,
provided the debtor is in possession of the mortgaged realty and no third person, not a A reading of the Serra Serra case, however, supports the Court's conclusion that a writ
party to the foreclosure suit, had intervened; and (3) extrajudicial foreclosure of a real of possession should not have been issued in this case. It was ruled by the Court that
estate mortgage under Sec. 7 of Act No. 3135 as amended by Act No. 4118. while a writ of possession may be issued only pursuant to a decree of registration in an
original land registration proceedings, it cannot issue against possessors under claim
In land registration cases, principles regarding the issuance of a writ of possession are of ownership, as actual possession under claim of ownership raises a disputable
well-settled. A judgment confirming the title of the applicant in a registration case and presumption of ownership, and the true owner must resort to judicial process for the
ordering its registration in his name necessarily carries with it the delivery of possession recovery of the property, not summarily through a motion for the issuance of a writ of
which is an inherent element of the right of ownership. 8 This is sanctioned by existing possession.17 Thus, it was erroneous for the RTC to have issued the writ of possession
laws in this jurisdiction and by the generally accepted principle upon which the against petitioners. This conclusion, of course, is without prejudice to any case that
administration of justice rests.9 respondent may file for the recovery of the property.
MAGDALENA T. VILLASI, Petitioner, vs. FILOMENO GARCIA, substituted by his While the building was declared for taxation purposes in the name of FGCI, the lots in
heirs, namely, ERMELINDA H. GARCIA, LIZA GARCIA-GONZALEZ, THERESA which it was erected were registered in the names of the Spouses Filomeno Garcia and
GARCIA-TIANGSON, MARIVIC H. GARCIA, MARLENE GARCIA-MOMIN, Ermelinda Halili-Garcia (Spouses Garcia). After the mandatory posting and publication
GERARDO H. GARCIA, GIDEON H. GARCIA and GENEROSO H. GARCIA, and of notice of sale on execution of real property were complied with, a public auction was
ERMELINDA H. GARCIA, Respondents. G.R. No. 190106 January 15, 2014 scheduled on 25 January 2006.

Doctrine: It is a basic principle of law that money judgments are enforceable only To forestall the sale on execution, the Spouses Garcia filed an Affidavit of Third Party
against the property incontrovertibly belonging to the judgment debtor, and if the Claim10 and a Motion to Set Aside Notice of Sale on Execution,11 claiming that they are
property belonging to any third person is mistakenly levied upon to answer for another the lawful owners of the property which was erroneously levied upon by the sheriff. To
man’s indebtedness, such person has all the right to challenge the levy through any of persuade the court a quo to grant their motion, the Spouses Garcia argued that the
the remedies provided for under the Rules of Court. Section 16,18 Rule 39 specifically building covered by the levy was mistakenly assessed by the City Assessor in the name
provides that a third person may avail himself of the remedies of either terceria, to of FGCI. The motion was opposed by Villasi who insisted that its ownership belongs to
determine whether the sheriff has rightly or wrongly taken hold of the property not FGCI and not to the Spouses Garcia as shown by the tax declaration.
belonging to the judgment debtor or obligor, or an independent "separate action" to
vindicate his claim of ownership and/or possession over the foreclosed property. After weighing the arguments of the opposing parties, the RTC issued on 24 February
However, the person other than the judgment debtor who claims ownership or right 2005 an Order12 directing the Sheriff to hold in abeyance the conduct of the sale on
over levied properties is not precluded from taking other legal remedies to prosecute execution.
his claim.19
ISSUE: Whether or not CA erred in upholding the decision of the trial court to suspend
Indeed, the power of the court in executing judgments extends only to properties and hold in abeyance the sale on execution of the buildings levied upon on the basis of
unquestionably belonging to the judgment debtor alone. An execution can be issued respondents’ affidavit of third-party claim.
only against a party and not against one who did not have his day in court. The duty of
the sheriff is to levy the property of the judgment debtor not that of a third person. For, RULING: NO. The right of a third-party claimant to file a terceria is founded on his title
as the saying goes, one man's goods shall not be sold for another man's debts. or right of possession.1avvphi1 Corollary thereto, before the court can exercise its
supervisory power to direct the release of the property mistakenly levied and the
FACTS: Sometime in 1990, petitioner Magdalena T Villasi (Villasi) engaged the restoration thereof to its rightful owner, the claimant must first unmistakably establish
services of respondent Fil-Garcia Construction, Inc. (FGCI) to construct a seven-storey his ownership or right of possession thereon. In Spouses Sy v. Hon. Discaya, 21 we
condominium building located at Aurora Boulevard corner N. Domingo Street, Cubao, declared that for a third-party claim or a terceria to prosper, the claimant must first
Quezon City. For failure of Villasi to fully pay the contract price despite several sufficiently establish his right on the property:
demands, FGCI initiated a suit for collection of sum of money before the RTC of Quezon
City. [A] third person whose property was seized by a sheriff to answer for the obligation of
the judgment debtor may invoke the supervisory power of the court which authorized
Finding that FGCI was able to preponderantly establish by evidence its right to the such execution. Upon due application by the third person and after summary hearing,
unpaid accomplishment billings, the RTC rendered a Decision 4 dated 26 June 1996 in the court may command that the property be released from the mistaken levy and
FGCI’s favor. While the trial court brushed aside the allegation of Villasi that an excess restored to the rightful owner or possessor. What said court can do in these instances,
payment was made, it upheld the claim of FGCI to the unpaid amount of the contract however, is limited to a determination of whether the sheriff has acted rightly or wrongly
price. in the performance of his duties in the execution of judgment, more specifically, if he
has indeed taken hold of property not belonging to the judgment debtor. The court does
Elevated on appeal , the appellate court ruled that an overpayment was made by Villasi not and cannot pass upon the question of title to the property, with any character of
and thereby directed FGCI to return the amount that was paid in excess. finality. It can treat of the matter only insofar as may be necessary to decide if the sheriff
has acted correctly or not. It can require the sheriff to restore the property to the
claimant's possession if warranted by the evidence. However, if the claimant's proofs
To enforce her right as prevailing party, Villasi filed a Motion for Execution of the 20 do not persuade the court of the validity of his title or right of possession thereto, the
November 2000 Court of Appeals Decision, which was favorably acted upon by the claim will be denied.
RTC.9 A Writ of Execution was issued on 28 April 2004, commanding the Sheriff to
execute and make effective the 20 November 2000 Decision of the Court of Appeals.
Although tax declarations or realty tax payment of property are not conclusive evidence
of ownership, nevertheless, they are good indicia of possession in the concept of owner
To satisfy the judgment, the sheriff levied on a building located at No. 140 Kalayaan for no one in his right mind would be paying taxes for a property that is not in his actual
Avenue, Quezon City, covered by Tax Declaration No. D-021-01458, and built in the or at least constructive possession. They constitute at least proof that the holder has a
lots registered under Transfer Certificates of Title (TCT) Nos. 379193 and 379194. claim of title over the property. The voluntary declaration of a piece of property for
taxation purposes manifests not only one’s sincere and honest desire to obtain title to will result in the identification of the Spouses Garcia as FGCI itself and will make them
the property and announces his adverse claim against the State and all other interested liable for FGCI’s judgment debt.
parties, but also the intention to contribute needed revenues to the Government. Such
an act strengthens one’s bona fide claim of acquisition of ownership. 25

It likewise failed to escape our attention that FGCI is in actual possession of the building
and as the payment of taxes coupled with actual possession of the land covered by tax
declaration strongly supports a claim of ownership. 26 Quite significantly, all the court
processes in an earlier collection suit between FGCI and Villasi were served, thru the
former’s representative Filomeno Garcia, at No. 140 Kalayaan Avenue, Quezon City,
where the subject property is located. This circumstance is consistent with the tax
declaration in the name of FGCI.

The explanation proffered by the Spouses Garcia, that the City Assessor merely
committed an error when it declared the property for taxation purposes in the name of
FGCI, appears to be suspect in the absence of any prompt and serious effort on their
part to have it rectified before the onset of the instant controversy. The correction of
entry belatedly sought by the Spouses Garcia is indicative of its intention to put the
property beyond the reach of the judgment creditor. Every prevailing party to a suit
enjoys the corollary right to the fruits of the judgment and, thus, court rules provide a
procedure to ensure that every favorable judgment is fully satisfied. 27 It is almost trite
to say that execution is the fruit and end of the suit. Hailing it as the "life of the law,"ratio
legis est anima,28 this Court has zealously guarded against any attempt to thwart the
rigid rule and deny the prevailing litigant his right to savour the fruit of his victory. 29 A
judgment, if left unexecuted, would be nothing but an empty triumph for the prevailing
party.30

While it is a hornbook doctrine that the accessory follows the principal, 31 that is, the
ownership of the property gives the right by accession to everything which is produced
thereby, or which is incorporated or attached thereto, either naturally or
artificially,32 such rule is not without exception. In cases where there is a clear and
convincing evidence to prove that the principal and the accessory are not owned by
one and the same person or entity, the presumption shall not be applied and the actual
ownership shall be upheld. In a number of cases, we recognized the separate
ownership of the land from the building and brushed aside the rule that accessory
follows the principal.

The rule on accession is not an iron-clad dictum. On instances where this Court was
confronted with cases requiring judicial determination of the ownership of the building
separate from the lot, it never hesitated to disregard such rule. The case at bar is of
similar import. When there are factual and evidentiary evidence to prove that the
building and the lot on which it stands are owned by different persons, they shall be
treated separately. As such, the building or the lot, as the case may be, can be made
liable to answer for the obligation of its respective owner.

Finally, the issue regarding the piercing of the veil of corporate fiction is irrelevant in
this case. The Spouses Garcia are trying to protect FGCI from liability by asserting that
they, not FGCI, own the levied property. The Spouses Garcia are asserting their
separation from FGCI. FGCI, the judgment debtor, is the proven owner of the building.
Piercing FGCI’s corporate veil will not protect FGCI from its judgment debt. Piercing
FEDERICO GEMINIANO, MARIA GEMINIANO, ERNESTO GEMINIANO, ASUNCION mortgage. Lee, however, never sought a writ of possession in order that she gain
GEMINIANO, LARRY GEMINIANO and MARLYN GEMINIANO, petitioners, vs. possession of the property in question.12The petitioners' mother therefore remained in
COURT OF APPEALS, DOMINADOR NICOLAS, and MARY A. possession of the lot.
NICOLAS, respondents. G.R. No. 120303 July 24, 1996
It is undisputed that the private respondents came into possession of 126 square-meter
FACTS: It appears that Lot No. 3765-B-1 containing an area of 314 square meters was portion of the said lot by virtue of contract of lease executed by the petitioners' mother
originally owned by the petitioners' mother, Paulina Amado vda. de Geminiano. On a as lessor, and the private respondents as lessees, is therefore well-established, and
12-square-meter portion of that lot stood the petitioners' unfinished bungalow, which carries with it a recognition of the lessor's title. 13 The private respondents, as lessees
the petitioners sold in November 1978 to the private respondents for the sum of who had undisturbed possession for the entire term under the lease, are then estopped
P6,000.00, with an alleged promise to sell to the latter that portion of the lot occupied to deny their landlord's title, or to assert a better title not only in themselves, but also in
by the house. Subsequently, the petitioners' mother executed a contract of lease over some third person while they remain in possession of the leased premises and until
a 126 square-meter portion of the lot, including that portion on which the house stood, they surrender possession to the landlord.14 This estoppel applies even though the
in favor of the private respondents for P40.00 per month for a period of seven years lessor had no title at the time the relation of lessor and lessee was created,15 and may
commencing on 15 November 1978.1 The private respondents then introduced be asserted not only by the original lessor, but also by those who succeed to his title.16
additional improvements and registered the house in their names. After the expiration
of the lease contract in November 1985, however, the petitioners' mother refused to Being mere lessees, the private respondents knew that their occupation of the premises
accept the monthly rentals. would continue only for the life of the lease. Plainly, they cannot be considered as
possessors nor builders in good faith.17
It turned out that the lot in question was the subject of a suit, which resulted in its
acquisition by one Maria Lee in 1972. In 1982, Lee sold the lot to Lily Salcedo, who in In a plethora of cases,18 this Court has held that Article 448 of the Civil Code, in relation
turn sold it in 1984 to the spouses Agustin and Ester Dionisio. to Article 546 of the same Code, which allows full reimbursement of useful
improvements and retention of the premises until reimbursement is made, applies only
On 14 February 1992, the Dionisio spouses executed a Deed of Quitclaim over the said to a possessor in good faith, i.e., one who builds on land with the belief that he is the
property in favor of the petitioners.2 As such, the lot was registered in the latter's name.3 owner thereof. It does not apply where one's only interest is that of a lessee under a
rental contract; otherwise, it would always be in the power of the tenant to "improve"
his landlord out of his property.
On 9 February 1993, the petitioners sent, via registered mail, a letters addressed to
private respondent Mary Nicolas demanding that she vacate the premises and pay the
rentals in arrears within twenty days from notice.4 Anent the alleged promise of the petitioners to sell the lot occupied by the private
respondents' house, the same was not substantiated by convincing evidence. Neither
the deed of sale over the house nor the contract of lease contained an option in favor
Upon failure of the private respondents to heed the demand, the petitioners filed with of the respondent spouses to purchase the said lot. And even if the petitioners indeed
the MTCC of Dagupan City a complaint for unlawful detainer and damages. promised to sell, it would not make the private respondents possessors or builders in
good faith so as to covered by the provision of Article 448 of the Civil Code. The latter
During the pre-trial conference, the parties agreed to confine the issues to: (1) whether cannot raise the mere expectancy or ownership of the aforementioned lot because the
there was an implied renewal of the lease which expired in November 1985; (2) whether alleged promise to sell was not fulfilled nor its existence even proven. The first thing
the lessees were builders in good faith and entitled to reimbursement of the value of that the private respondents should have done was to reduce the alleged promise into
the house and improvements; and (3) the value of the house. writing, because under Article 1403 of the Civil Code, an agreement for the sale of real
property or an interest therein is unenforceable, unless some note or memorandum
The petitioners ask the Court to set aside the decision of the Court of Appeals affirming thereof be produced. Not having taken any steps in order that the alleged promise to
sell may be enforced, the private respondents cannot bank on the promise and profess
the decision of Branch 40 of the Regional Trial Court (RTC) of Dagupan City, which, in
any claim nor color of title over the lot in question.
turn, reversed the MTCC; ordered the petitioners to reimburse the private respondents
the value of the house in question and other improvements; and allowed the latter to
retain the premises until reimbursement was made. There is no need to apply by analogy the provisions of Article 448 on indemnity as was
done in Pecson vs. Court of Appeals,19 because the situation sought to be avoided and
ISSUE: Whether the private respondents are builder in good faith or mere lessees. which would justify the application of that provision, is not present in this case. Suffice
it to say, "a state of forced coownership" would not be created between the petitioners
and the private respondents. For, as correctly pointed out by the petitioners, the right
RULING: Private respondents are mere lessees. In this case, both parties admit that of the private respondents as lessees are governed by Article 1678 of the Civil Code
the land in question was originally owned by the petitioners' mother. The land was which allows reimbursement to the extent of one-half of the value of the useful
allegedly acquired later by one Maria Lee by virtue of an extrajudicial foreclosure of improvements.
It must be stressed, however, that the right to indemnity under Article 1678 of the Civil
Code arises only if the lessor opts to appropriate the improvements. Since the
petitioners refused to exercise that option, 20 the private respondents cannot compel
them to reimburse the one-half value of the house and improvements. Neither can they
retain the premises until reimbursement is made. The private respondents' sole right
then is to remove the improvements without causing any more impairment upon the
property leased than is necessary.
SPOUSES CRISPIN AQUINO and TERESA V. AQUINO, herein represented by METC: ordered respondents to vacate the property
their Attorney-in-Fact, AMADOR D. LEDESMA, Petitioners, RTC: affirmed the MeTC's Decision
vs. SPOUSES EUSEBIO AGUILAR and JOSEFINA V. AGUILAR, Respondents. G.R. CA: declared that respondents should be reimbursed for the necessary and useful
No. 182754, June 29, 2015 expenses they had introduced on petitioners' property, pursuant to Articles 1678 and
548 of the Civil Code.
Doctrine: Those who occupy the land of another at the latter's tolerance or permission, even
without any contract between them, are necessarily bound by an implied promise that the ISSUE: Whether the CA erred in remanding this case to the court of origin for the
occupants would vacate the property upon demand.70 Failure to comply with this demand renders determination of the necessary and useful expenses to be reimbursed to respondents
the possession unlawful and actual damages may be awarded to the owner from the date of the
pursuant to Articles 1678 and 546 of the Civil Code.
demand to vacate71 until the actual surrender of the property.

RULING: YES. By its express provision, Article 1678 of the Civil Code applies only to
FACTS: Teresa Vela Aquino (Teresa) and her husband, Crispin Aquino, are the owners
of a house and lot located at No. 6948, Rosal Street, Guadalupe lessees who build useful improvements on the leased property. It does not apply to
those who possess property by mere tolerance of the owners, without a contractual
right.
Since 1981, this property has been occupied by Teresa's sister, Josefina Vela Aguilar;
Josefina's spouse Eusebio; and their family.5 It appears from the record that
A careful reading of the statement made by this Court in Calubayan would show that it
respondents stayed on the property with the consent and approval of petitioners, who
were then residing in the United States.6 did not, as it could not, modify the express provision in Article 1678, but only noted an
"analogous" situation. According to the Court, the analogy between a tenant whose
term of lease has expired and a person who occupies the land of another at the latter's
While respondents were in possession of the property, the house previously tolerance lies in their implied obligation to vacate the premises upon demand of the
constructed therein was demolished, and a three-storey building built in its owner. The Court stated:
place.7 Respondents occupied half of the third floor of this new building)for the next 20
years without payment of rental.8
To begin with, it would appear that although the defendant is regarded by the plaintiffs
as a "squatter" his occupancy of the questioned premises had been permitted or
On 22 September 2003, petitioners sent a letter to respondents informing them that an tolerated even before the Philippine Realty Corporation sold the lots to the plaintiffs.
immediate family member needed to use the premises and demanding the surrender Otherwise, the latter would not have found him on the premises. It may be true that
of the property within 10 days from notice.9 Respondents failed to heed this demand, upon their acquisition of the parcels of land in 1957, plaintiffs notified and .requested
prompting petitioners to file a Complaint for ejectment against them before the office of defendant to see them, but despite defendant's failure to heed these requests, plaintiffs
the barangay captain of Guadalupe Viejo.10 The parties attempted to reach an amicable did not choose to bring an action in court but suffered the defendant instead to remain
settlement in accordance with Section 412 of the Local Government Code, but these in the premises for almost six years. Only on February 2, 1963, did the plaintiffs for the
efforts proved unsuccessful. first time notify the defendant that "they now need the two parcels of land in question"
and requested him to vacate the same. In allowing several years to pass without
On 19 November 2003, petitioner spouses Aquino filed a Complaint12 with the MeTC requiring the occupant to vacate the premises nor filing an action to eject him, plaintiffs
of Makati City praying that respondents be ordered to (a) vacate the portion of the have acquiesced to defendant's possession and use of the premises. It has been held
building they were then occupying; and (b) pay petitioner a reasonable amount for the that a person who occupies the land of another at the latter's tolerance or permission,
use and enjoyment of the premises from the time the formal demand to vacate was without any contract between them, is necessarily bound by an implied promise that he
made.13 will vacate upon demand, failing which a summary action for ejectment is the proper
remedy against them. The status of defendant is analogous to that of a lessee or tenant
whose term of lease has expired but whose occupancy continued by tolerance of the
In their Answer with Counterclaim,14 respondents claimed that they had contributed to
owner. In such a case, the unlawful deprivation or withholding of possession is to be
the improvement of the property and the construction of the building, both in terms of
counted from the date of the demand to vacate.60(Emphasis in the original)
money and management/supervision services. Petitioners purportedly agreed to let
them contribute to the costs of construction in exchange for the exclusive use of a
portion of the building. It is clear from the above that Calubayan is not sufficient basis to confer the status and
rights of a lessee on those who occupy property by mere tolerance of the owner.
Since they were allegedly co-owners of the building and builders in good faith,
respondents claimed that they had the right to be compensated for the current value of In this case, there is absolutely no evidence of any lease contract between the parties.
their contribution.16 Accordingly, they prayed for the dismissal of the Complaint and the In fact, respondents themselves never alleged that they were lessees of the lot or the
award of ₱5 million as compensation for their contributions to the construction of the building in question. Quite the opposite, they insisted that they were co-owners of the
building, as well as moral damages, attorney's fees and costs of litigation. building and builders in good faith under Article 448 of the Civil Code. For that reason,
respondents argue that it was erroneous for the CA to consider them as lessees and to latter had every intention of selling it. That this sale did not materialize is irrelevant.
determine their rights in accordance with Article 1678. What is crucial is that petitioners left respondents clear instructions not to build on the
land.
As builders in bad faith, respondents are not entitled to reimbursement of useful
expenses. We also agree with the RTC's ruling that the lack of constant reminders from petitioners
about the "prohibition" expressed in the 1983 letter was immaterial. The prohibition is
Furthermore, even if we were to subscribe to the CA' s theory that the situation of considered extant and continuing since there is no evidence that this letter was ever
respondents is "analogous to that of a lessee or tenant whose term of lease has expired withdrawn or modified. Moreover, no evidence was presented to show that petitioners
but whose occupancy continued by tolerance," the absence of good faith on their part were aware of what was happening: that respondents were constructing a portion of
prevents them from invoking the provisions of Article 1678. the building with their own funds and for their exclusive use and ownership. Neither
were respondents able to present evidence that petitioners had agreed to share the
expenses with them, or that the former had given consent to the latter's contribution, if
As discussed above, the MeTC, the RTC and the CA all rejected the claims of any.
respondents that they were builders in good faith. This pronouncement is considered
conclusive upon this Court, in view of respondents' failure to appeal from the CA
decision. This rule bars the application of Article 1678 as well as Articles 448 and 576 In view of the foregoing, this Court's previous rulings on Article 448 cannot be applied
of the Civil Code and all other provisions requiring good faith on the part of the builder. to this case. Hence, we hold that petitioners, as the owners of the land, have the right
to appropriate what has been built on the property, without any obligation to pay
indemnity therefor;65 and that respondents have no right to a refund of any
We are aware that in some instances, this Court has allowed the application of Article improvement built therein,66 pursuant to Articles 449 and 450 of the Civil Code:
448 to a builder who has constructed improvements on the land of another with the
consent of the owner.61 In those cases, the Court found that the owners knew and
approved of the construction of improvements on the property. Hence, we ruled therein Art. 449. He who builds, plants or sows in bad faith on the land of another, loses what
that the structures were built in good faith, even though the builders knew that they is built, planted or sown without right of indemnity.
were constructing the improvement on land owned by another.
Art. 450. The owner of the land on which anything has been built, planted or sown in
Although the factual circumstances in the instant case are somewhat similar, there is bad faith may demand the demolition of the work, or that the planting or sowing be
one crucial factor that warrants a departure from the above-described rulings: the removed, in order to replace things in their former condition at the expense of the
presence of evidence that petitioners prohibited respondents from building their own person who built, planted or sowed; or he may compel the builder or planter to pay the
structure on a portion of the property. Based on the findings of fact of the MeTC and price of the land, and the sower the proper rent.
the RTC, petitioners had already warned respondents not to build a structure on the
property as early as 1983. The MeTC explained: Likewise, in a letter dated 15 July Art. 451. In the cases of the two preceding articles, the landowner is entitled to damages
1983 sent by plaintiffs to the defendants marked as Exhibit "2" of defendants' Position from the builder, planter or sower.
Paper, Teresa Aquino made known to the defendants not to construct on the premises
as she planned to sell the same when the value of the property shall increase (sic). Respondents may recover the necessary expenses incurred for the
Defendants are undoubtedly builders in bad faith for despite the prohibition made upon preservation of the property but without the right of retention.
them, they continued their construction activities upon respondents' property.
Pursuant to Article 452 of the Civil Code, a builder in bad faith is entitled to recoup the
Their contention that pursuant to Article 453 of the Civil Code, they should be necessary expenses incurred for the preservation of the land.67 The CA correctly ruled
considered builders in good faith even if they have acted in bad faith, since their act of that respondents in this case are similarly entitled to this reimbursement. However,
introducing improvements to one-half of the third floor of the three storey building was being builders in bad faith, they do not have the right of retention over the premises. 6
with knowledge and without opposition on the part of the appellants, cannot be
sustained, principally on the ground that as stated earlier, their Exhibit "2" is very limpid
on the act that they were already forewarned as early as 1983 not to introduce any While the evidence before this Court does not establish the amount of necessary
improvements thereon as the property is slated to be sold as it was only bought for expenses incurred by respondents during their stay in the property, we note that even
investment purposes. The fact that the appellees did not thereafter remind them of this, petitioners do not deny that such expenses were incurred. In fact, in a letter dated 15
is of no moment, as this letter was not likewise withdrawn by a subsequent one or July 1983, petitioners acknowledged that respondents had spent personal money for
modified by the appellees.64 the maintenance of the property. Petitioners even promised to reimburse them for those
expenses.69 In this light, we find it proper to order the remand of this case to the court
a quo for the purpose of determining the amount of necessary expenses to be
We find no reason to depart from the conclusions of the trial courts. Respondents were reimbursed to respondents.
evidently prohibited by petitioners from building improvements on the land because the
TECNOGAS PHILIPPINES MANUFACTURING CORPORATION, petitioner, vs. ISSUES:
COURT OF APPEALS (FORMER SPECIAL SEVENTEENTH DIVISION) and
EDUARDO UY, respondents. G.R. No. 108894 February 10, 1997 1. Whether or not the respondent Court of Appeals erred in holding the petitioner a
builder in bad faith because it is "presumed to know the metes and bounds of his
FACTS: property."
2. Whether or not the respondent Court of Appeals erred when it ruled that the
 Plaintiff Tecnogas Philippines (herein petitioner) is the registered owner of a parcel amicable settlement entered into between petitioner and private respondent
of land situated in Barrio San Dionisio, Parañaque, which he bought from Pariz estops the former from questioning the private respondent's "right" over the
industries. disputed property.
3. What then is the applicable provision in this case which private respondent may
invoke as his remedy: Article 448 or Article 450 31 of the Civil Code?
 The defendant (herein private respondent) is the registered owner of a parcel of
land known which adjoins plaintiff's land and was purchased by defendant from a
certain Enrile Antonio also in 1970; RULING:

 In 1971, defendant purchased another lot also adjoining plaintiffs land from a certain 1. Yes. The CA erred in holding the petitioner in bad faith.
Miguel Rodriguez and the same was registered in defendant's name under Transfer
Certificate of Title No. 31390, of the Registry of Deeds for the Province of Rizal; The record is not clear as to who actually built those structures, but it may well be
assumed that petitioner's predecessor-in-interest, Pariz Industries, did so. Article 527
 The portions of the buildings and wall bought by plaintiff together with the land from of the Civil Code presumes good faith, and since no proof exists to show that the
Pariz Industries are occupying a portion of defendant's adjoining land; encroachment over a narrow, needle-shaped portion of private respondent's land was
done in bad faith by the builder of the encroaching structures, the latter should be
 Upon learning of the encroachment or occupation by its buildings and wall of a presumed to have built them in good faith. 21 It is presumed that possession continues
portion of defendant's land, plaintiff offered to buy from defendant that particular to be enjoyed in the same character in which it was acquired, until the contrary is
portion of defendant's land occupied by portions of its buildings and wall with an proved. 22 Good faith consists in the belief of the builder that the land he is building on
area of 770 square meters, more or less, but defendant, however, refused the offer. is his, and his ignorance of any defect or flaw in his title. 23 Hence, such good faith, by
law, passed on to Pariz's successor, petitioner in this case. Further, "(w)here one
derives title to property from another, the act, declaration, or omission of the latter, while
 In 1973, the parties entered into a private agreement before a certain Col. Rosales
holding the title, in relation to the property, is evidence against the former." 24 And
in Malacañang, wherein plaintiff agreed to demolish the wall at the back portion of
possession acquired in good faith does not lose this character except in case and from
its land thus giving to defendant possession of a portion of his land previously
the moment facts exist which show that the possessor is not unaware that he
enclosed by plaintiff's wall;
possesses the thing improperly or wrongfully. 25The good faith ceases from the moment
defects in the title are made known to the possessor, by extraneous evidence or by suit
 The defendant later filed a complaint before the office of Municipal Engineer of for recovery of the property by the true owner. 26
Parañaque, Metro Manila as well as before the Office of the Provincial Fiscal of
Rizal against plaintiff in connection with the encroachment or occupation by
plaintiff's buildings and walls of a portion of its land but said complaint did not Recall that the encroachment in the present case was caused by a very slight deviation
prosper; of the erected wall (as fence) which was supposed to run in a straight line from point 9
to point 1 of petitioner's lot. It was an error which, in the context of the attendant facts,
was consistent with good faith. Consequently, the builder, if sued by the aggrieved
 The defendant dug or caused to be dug a canal along plaintiff's wall, a portion of
landowner for recovery of possession, could have invoked the provisions of Art. 448 of
which collapsed in June, 1980, and led to the filing by plaintiff of the supplemental
the Civil Code, which reads:
complaint in the above-entitled case and a separate criminal complaint for malicious
mischief against defendant and his wife which ultimately resulted into the conviction
in court of defendant's wife for the crime of malicious mischief; The owner of the land on which anything has been built, sown or planted in good faith,
shall have the right to appropriate as his own the works, sowing or planting, after
 While trial of the case was in progress, plaintiff filed in Court a formal proposal for payment of the indemnity provided for in articles 546 and 548, or to oblige the one who
settlement of the case but said proposal, however, was ignored by defendant. built or planted to pay the price of the land, and the one who sowed, the proper rent.
However, the builder or planter cannot be obliged to buy the land if its value is
considerably more than that of the building or trees. In such case, he shall pay
 The RTC ruled in favor of the plaintiff and ordering the defendant to sell the land to reasonable rent, if the owner of the land does not choose to appropriate the building or
the plaintiff. trees after proper indemnity. The parties shall agree upon the terms of the lease and in
 The CA reversed and set aside the decision of the RTC case of disagreement, the court shall fix the terms thereof.
The obvious benefit to the builder under this article is that, instead of being outrightly xxx xxx xxx
ejected from the land, he can compel the landowner to make a choice between the two
options: (1) to appropriate the building by paying the indemnity required by law, or (2) The Civil Code not only defines and authorizes compromises, it in fact encourages them
sell the land to the builder. The landowner cannot refuse to exercise either option and in civil actions. Art. 2029 states that "The Court shall endeavor to persuade the litigants
compel instead the owner of the building to remove it from the land. in a civil case to agree upon some fair compromise." . . .

The question, however, is whether the same benefit can be invoked by petitioner who, In the context of the established facts, we hold that petitioner did not lose its rights
as earlier stated, is not the builder of the offending structures but possesses them as under Article 448 of the Civil Code on the basis merely of the fact that some years after
buyer. The answer is in the affirmative. acquiring the property in good faith, it learned about — and aptly recognized — the right
of private respondent to a portion of the land occupied by its building. The supervening
In the first place, there is no sufficient showing that petitioner was aware of the awareness of the encroachment by petitioner does not militate against its right to claim
encroachment at the time it acquired the property from Pariz Industries. We agree with the status of a builder in good faith. In fact, a judicious reading of said Article 448 will
the trial court that various factors in evidence adequately show petitioner's lack of readily show that the landowner's exercise of his option can only take place after the
awareness thereof. In any case, contrary proof has not overthrown the presumption of builder shall have come to know of the intrusion — in short, when both parties shall
good faith under Article 527 of the Civil Code, as already stated, taken together with have become aware of it. Only then will the occasion for exercising the option arise, for
the disputable presumptions of the law on evidence. These presumptions state, under it is only then that both parties will have been aware that a problem exists in regard to
Section 3 (a) of Rule 131 of the Rules of Court, that the person is innocent of a crime their property rights.
or wrong; and under Section 3 (ff) of Rule 131, that the law has been obeyed. In fact,
private respondent Eduardo Uy himself was unaware of such intrusion into his property 3. In view of the good faith of both petitioner and private respondent, their rights and
until after 1971 when he hired a surveyor, following his purchase of another adjoining obligations are to be governed by Art. 448.
lot, to survey all his newly acquired lots. Upon being apprised of the encroachment,
petitioner immediately offered to buy the area occupied by its building — a species of
conduct consistent with good faith. The essential fairness of this codal provision has been pointed out by Mme. Justice
Ameurfina Melencio-Herrera, citing Manresa and applicable precedents, in the case
of Depra vs. Dumlao, 32 to wit:
In the second place, upon delivery of the property by Pariz Industries, as seller, to the
petitioner, as buyer, the latter acquired ownership of the property. Consequently and
as earlier discussed, petitioner is deemed to have stepped into the shoes of the seller Where the builder, planter or sower has acted in good faith, a conflict of rights arises
in regard to all rights of ownership over the immovable sold, including the right to between the owners, and it becomes necessary to protect the owner of the
compel the private respondent to exercise either of the two options provided under improvements without causing injustice to the owner of the land. In view of the
Article 448 of the Civil Code. impracticality of creating a state of forced co-ownership, the law has provided a just
solution by giving the owner of the land the option to acquire the improvements after
payment of the proper indemnity, or to oblige the builder or planter to pay for the land
2. Yes. Petitioner cannot be held in estoppel for entering into the amicable settlement. and the sower to pay the proper rent. It is the owner of the land who is authorized to
exercise the option, because his right is older, and because, by the principle of
Petitioner agreed only to the demolition of a portion of the wall separating the adjoining accession, he is entitled to the ownership of the accessory thing. (3 Manresa 213;
properties of the parties — i.e. "up to the back of the building housing the machineries." Bernardo vs. Bataclan, 37 Off. Gaz. 1382; Co Tao vs. Chan Chico, G.R. No. 49167,
But that portion of the fence which served as the wall housing the electroplating April 30, 1949; Article applied; see Cabral, et al. vs. Ibanez [S.C.] 52 Off. Gaz. 217;
machineries was not to be demolished. Rather, it was to "be subject to negotiation by Marfori vs. Velasco, [C.A.] 52 Off. Gaz. 2050).
herein parties." The settlement may have recognized the ownership of private
respondent but such admission cannot be equated with bad faith. Petitioner was only The private respondent's insistence on the removal of the encroaching structures as
trying to avoid a litigation, one reason for entering into an amicable settlement. the proper remedy, which respondent Court sustained in its assailed Decisions, is thus
legally flawed. This is not one of the remedies bestowed upon him by law. It would be
As was ruled in Osmeña vs. Commission on Audit, 30 available only if and when he chooses to compel the petitioner to buy the land at a
reasonable price but the latter fails to pay such price. 33 This has not taken place.
A compromise is a bilateral act or transaction that is expressly acknowledged as a Hence, his options are limited to: (1) appropriating the encroaching portion of
juridical agreement by the Civil Code and is therein dealt with in some detail. "A petitioner's building after payment of proper indemnity, or (2) obliging the latter to buy
compromise," declares Article 2208 of said Code, "is a contract whereby the parties, the lot occupied by the structure. He cannot exercise a remedy of his own liking.
by making reciprocal concessions, avoid a litigation or put an end to one already
commenced." Neither is petitioner's prayer that private respondent be ordered to sell the land 34 the
proper remedy. While that was dubbed as the "more workable solution" in Grana and
Torralba vs. The Court of Appeals, et al., 35 it was not the relief granted in that case as
the landowners were directed to exercise "within 30 days from this decision their option
to either buy the portion of the petitioners' house on their land or sell to said petitioners
the portion of their land on which it stands." 36 Moreover, in Grana and Torralba, the
area involved was only 87 square meters while this case involves 520 square meters 37.
In line with the case of Depra vs. Dumlao, 38 this case will have to be remanded to the
trial court for further proceedings to fully implement the mandate of Art. 448. It is a rule
of procedure for the Supreme Court to strive to settle the entire controversy in a single
proceeding leaving no root or branch to bear the seeds of future
litigation. 39

Petitioner, however, must also pay the rent for the property occupied by its building as
prescribed by respondent Court from October 4, 1979, but only up to the date private
respondent serves notice of its option upon petitioner and the trial court; that is, if such
option is for private respondent to appropriate the encroaching structure. In such event,
petitioner would have a right of retention which negates the obligation to pay
rent. 40 The rent should however continue if the option chosen is compulsory sale, but
only up to the actual transfer of ownership.
PLEASANTVILLE DEVELOPMENT CORPORATION, petitioner, vs. COURT OF ISSUE: Whether or not Kee is a builder in good faith?
APPEALS, WILSON KEE, C.T. TORRES ENTERPRISES, INC. and ELDRED
JARDINICO, respondents. G.R. No. 79688 February 1, 1996 RULING: Yes. Kee is a builder in good faith. Good faith consists in the belief of the
builder that the land he is building on is his and his ignorance of any defect or flaw in
FACTS: his title 9 . And as good faith is presumed, petitioner has the burden of proving bad faith
 Edith Robillo purchased from petitioner a parcel of land designated as Lot 9, Phase II on the part of Kee 10 .
and located at Taculing Road, Pleasantville Subdivision, Bacolod City. In 1975,
respondent Eldred Jardinico bought the rights to the lot from Robillo. At that time, Lot 9 At the time he built improvements on Lot 8, Kee believed that said lot was what he
was vacant. bought from petitioner. He was not aware that the lot delivered to him was not Lot 8.
Thus, Kee's good faith. Petitioner failed to prove otherwise.
 Upon completing all payments, Jardinico secured from the Register of Deeds of
Bacolod City on December 19, 1978 Transfer Certificate of Title No. 106367 in his To demonstrate Kee's bad faith, petitioner points to Kee's violation of paragraphs 22
name. It was then that he discovered that improvements had been introduced on Lot 9 and 26 of the Contract of Sale on Installment.
by respondent Wilson Kee, who had taken possession thereof.

We disagree. Such violations have no bearing whatsoever on whether Kee was a


 It appears that on March 26, 1974, Kee bought on installment Lot 8 of the same
builder in good faith, that is, on his state of mind at the time he built the improvements
subdivision from C.T. Torres Enterprises, Inc. (CTTEI), the exclusive real estate agent
of petitioner. Under the Contract to Sell on Installment, Kee could possess the lot even on Lot 9. These alleged violations may give rise to petitioner's cause of action against
before the completion of all installment payments. On January 20, 1975, Kee paid Kee under the said contract (contractual breach), but may not be bases to negate the
presumption that Kee was a builder in good faith.
CTTEI the relocation fee of P50.00 and another P50.00 on January 27, 1975, for the
preparation of the lot plan. These amounts were paid prior to Kee's taking actual
possession of Lot 8. After the preparation of the lot plan and a copy thereof given to Petitioner also points out that, as found by the trial court, the Contract of Sale on
Kee, CTTEI through its employee, Zenaida Octaviano, accompanied Kee's wife, Installment covering Lot 8 between it and Kee was rescinded long before the present
Donabelle Kee, to inspect Lot 8. Unfortunately, the parcel of land pointed by Octaviano action was instituted. This has no relevance on the liability of petitioner, as such fact
was Lot 9. Thereafter, Kee proceeded to construct his residence, a store, an auto repair does not negate the negligence of its agent in pointing out the wrong lot. to Kee. Such
shop and other improvements on the lot. circumstance is relevant only as it gives Jardinico a cause of action for unlawful detainer
against Kee.
 After discovering that Lot 9 was occupied by Kee, Jardinico confronted him. The parties
tried to reach an amicable settlement, but failed. Petitioner next contends that Kee cannot "claim that another lot was erroneously
pointed out to him" because the latter agreed to the following provision in the Contract
 On January 30, 1981, Jardinico's lawyer wrote Kee, demanding that the latter remove of Sale on installment, to wit:
all improvements and vacate Lot 9. When Kee refused to vacate Lot 9, Jardinico filed
with the Municipal Trial Court in Cities, Branch 3, Bacolod City (MTCC), a complaint for 13. The Vendee hereby declares that prior to the execution of his contract
ejectment with damages against Kee. he/she has personally examined or inspected the property made subject-
matter hereof, as to its location, contours, as well as the natural condition of
 Kee, in turn, filed a third-party complaint against petitioner and CTTEI. the lots and from the date hereof whatever consequential change therein
made due to erosion, the said Vendee shall bear the expenses of the
 The MTCC ordered Wilson Kee to vacate the premises of Lot 9, covered by TCT No. necessary fillings, when the same is so desired by him/her. 11
106367 and to remove all structures and improvements he introduced thereon.
The subject matter of this provision of the contract is the change of the location, contour
 The Regional Trial Court ruled that petitioner and CTTEI were not at fault or were not and condition of the lot due to erosion. It merely provides that the vendee, having
negligent, there being no preponderant evidence to show that they directly participated examined the property prior to the execution of the contract, agrees to shoulder the
in the delivery of Lot 9 to Kee . It found Kee a builder in bad faith. It further ruled that expenses resulting from such change. We do not agree with the interpretation of
even assuming arguendo that Kee was acting in good faith, he was, nonetheless, guilty petitioner that Kee contracted away his right to recover damages resulting from
of unlawfully usurping the possessory right of Jardinico over Lot 9 from the time he was petitioner's negligence. Such waiver would be contrary to public policy and cannot be
served with notice to vacate said lot, and thus was liable for rental. allowed. "Rights may be waived, unless the waiver is contrary to law, public order,
public policy, morals, or good customs, or prejudicial to a third person with a right
 The CA reversed the RTC decision and declared Wilson Kee a builder in good faith recognized by law."
with respect to the improvements he introduced on Lot 9, and is entitled to the rights
granted him under Articles 448, 546 and 548 of the New Civil Code.
ERLINDA DINGLASAN DELOS SANTOS and her daughters, namely, VIRGINIA, foregoing stipulations and admissions, the RTC limited the issue as to who among the
AUREA, and BINGBING, all surnamed DELOS SANTOS, Petitioners vs ALBERTO parties should be held liable for damages and attorney's fees.
ABEJON and the estate of TERESITA DINGLASAN ABEJON, Respondents
G.R. No. 215820, March 20, 2017  The RTC (a) declared the Deed of Sale null and void; (b) ordered the cancellation of
TCT No. 180286 and the reinstatement of TCT No. 131753; and (c) ordered petitioners
FACTS: to pay respondents ₱l00,000.00 plus twelve percent (12%) per annum (4) that
respondents should be reimbursed for the amount of the loan, as well as the expenses
incurred for the construction of the three (3)-storey building in view of petitioners'
 The instant case arose from a Complaint for Cancellation of Title with collection of sum
categorical admission of their indebtedness to her, as well as the construction of the
of money5 filed by respondents against petitioners before the RTC. The complaint
building from which they derived benefit being the actual occupants of the property.
alleged that Erlinda and her late husband Pedro Delos Santos (Pedro) borrowed the
amount of ₱l00,000.00 from the former's sister, Teresita, as evidenced by a Promissory
Note dated April 8, 1998. As security for the loan, Erlinda and Pedro mortgaged their  The CA affirmed the decision of the RTC.
property consisting of 43.50 square meters situated at 2986 Gen. Del Pilar Street,
Bangkal, Makati City covered by TCT No. 131753 which mortgage was annotated on ISSUE: Whether or not the CA correctly held that petitioners should be held liable to
the title. respondents?

 After Pedro died, Erlinda ended up being unable to pay the loan, and as such, agreed RULING: No.
to sell the subject land to Teresita for ₱l50,000.00, or for the amount of the loan plus
an additional ₱50,000.00. On July 8, 1992, they executed a Deed of Sale and a Release
In the case at bar, it must be reiterated that during the pre-trial proceedings, the parties
of Mortgage, and eventually, TCT No. 131753 was cancelled and TCT No. 180286 was
agreed/stipulated that: (a) the subject land was previously covered by TCT No. 131753
issued in the name of "Teresita, Abejon married to Alberto S. Abejon."
in the name of Erlinda and Pedro, but such title was cancelled and replaced by TCT
No. 180286 in the name of Teresita; (b) the Deed of Sale and Release of Mortgage
 Thereafter, respondents constructed a three (3)-storey building worth ₱2,000,000.00 both executed on July 8, 1992 were forged, and thus, should be cancelled; (c) in view
on the subject land. Despite the foregoing, petitioners refused to acknowledge the sale, of said cancellations, TCT No. 180286 should likewise be cancelled and TCT No.
pointing out that since Pedro died in 1989, his signature in the Deed of Sale executed 131753 should be reinstated; (d) from the time when the spurious deed of sale was
in 1992 was definitely forged. As such, respondents demanded from petitioners the executed until the present, petitioners have been the actual occupants of the subject
amounts of ₱l50,000.00 representing the consideration for the sale of the subject land land as well as all improvements therein, including the three (3)-storey building
and ₱2,000,000.00 representing the construction cost of the three (3)-storey building, constructed by respondents; and (e) the ₱l00,000.00 loan still subsists and that
but to no avail. Thus, respondents filed the instant case. respondents paid for the improvements being currently occupied by petitioners, i.e., the
three (3)storey building.21 As such, the parties in this case are bound to honor the
 In defense, petitioners denied any participation relative to the spurious Deed of Sale, admissions and/or stipulations they made during the pre-trial.22
and instead, maintained that it was Teresita who fabricated the same and caused its
registration before the Register of Deeds of Makati City. They likewise asserted that Thus, in view of the foregoing admissions and/or stipulations, there is now a need to
Erlinda and Pedro never sold the subject land to Teresita for ₱l50,000.00 and that they properly determine to whom the following liabilities should devolve: (a) the ₱l00,000.00
did not receive any demand for the payment of ₱l00,000.00 representing the loan, as loan obligation; (b) the ₱50,000.00 extra consideration Teresita paid for the sale of the
well as the ₱2,000,000.00 representing the construction cost of the building. Finally, subject land, which was already declared void - a matter which the R TC and the CA
they claimed that the improvements introduced by Teresita on the subject land were all completely failed to resolve; and (c) the ₱2,000,000.00 construction cost of the three
voluntary on her part.7 (3)-storey building that was built on the subject land.

 During the pre-trial proceedings, the parties admitted and/or stipulated that: (a) the
While petitioners admitted the existence of the ₱l00,000.00 loan obligation as well as
subject land was previously covered by TCT No. 131753 in the name of Erlinda and
respondents' right to collect on the same, it does not necessarily follow that respondents
Pedro, but such title was cancelled and replaced by TCT No. 180286 in the name of
Teresita; (b) the Deed of Sale and Release of Mortgage executed on July 8, 1992 were should collect the loan amount from petitioners, as concluded by both the RTC and the
CA. It must be pointed out that such loan was contracted by Erlinda, who is only one
forged, and thus, should be cancelled; (c) in view of said cancellations, TCT No. 180286
should likewise be cancelled and TCT No. 131753 should be reinstated; (d) from the (1) out of the four (4) herein petitioners, and her deceased husband, Pedro, during the
latter's lifetime and while their marriage was still subsisting.23 As they were married
time when the spurious Deed of Sale was executed until the present, petitioners have
been the actual occupants of the subject land as well as all improvements therein, before the effectivity of the Family Code of the Philippines 24 and absent any showing
including the three (3)-storey building constructed by respondents; and (e) the of any pre-nuptial agreement between Erlinda and Pedro, it is safe to conclude that
their property relations were governed by the system of conjugal partnership of gains.
₱l00,000.00 loan still subsists and that respondents paid for the improvements being
currently occupied by petitioners, i.e., the three (3)-storey building. In view of the Hence, pursuant to Article 12125 of the Family Code, the ₱l00,000.00 loan obligation,
including interest, if any, is chargeable to Erlinda and Pedro's conjugal partnership as such, she knew all along that the aforesaid Deed of Sale - which contained a signature
it was a debt contracted by the both of them during their marriage; and should the purportedly belonging to Pedro, who died in 1989, or three (3) years prior to its
conjugal partnership be insufficient to cover the same, then Erlinda and Pedro (more execution - was void and would not have operated to transfer any rights over the subject
particularly, his estate as he is already deceased) shall be solidarily liable for the unpaid land to her name. Despite such awareness of the defect in their title to the subject land,
balance with their separate properties. While the portion attributable to Pedro was not respondents still proceeded in constructing a three (3)-storey building thereon.
considered extinguished by his death, it is merely passed on to his estate; and thus, Indubitably, they should be deemed as builders in bad faith.
his heirs, i.e., herein petitioners, could not be held directly answerable for the same,
contrary to the CA's conclusion.26 In sum, both the RTC and the CA erred in holding On the other hand, petitioners knew of the defect in the execution of the Deed of Sale
petitioners liable to respondents for the loan obligation in the amount of ₱l00,000.00. from the start, but nonetheless, still acquiesced to the construction of the three (3)-
storey building thereon. Hence, they should likewise be considered as landowners in
Alternative to the collection of the said sum, respondents may also choose to foreclose bad faith.
the mortgage on the subject land as the same was duly constituted to secure the
₱l00,000.00 loan obligation. In other words, respondents have the option to either file In this relation, Article 453 of the Civil Code provides that where both the landowner
a personal action for collection of sum of money or institute a real action to foreclose and the builder, planter, or sower acted in bad faith, they shall be treated as if both of
on the mortgage security. The aforesaid remedies are alternative, meaning the choice them were in good faith, viz.:
of one will operate to preclude the other.27
Article 453. If there was bad faith, not only on the part of the person who built, planted
II. or sowed on the land of another, but also on the part of the owner of such land, the
rights of one and the other shall be the same as though both had acted in good faith.
It is settled that "the declaration of nullity of a contract which is void ab initio operates
to restore things to the state and condition in which they were found before the It is understood that there is bad faith on the part of the landowner whenever the act
execution thereof."28 Pursuant to this rule, since the Deed of Sale involving the subject was done with his knowledge and without opposition on his part.
land stands to be nullified in view of the parties' stipulation to this effect, it is incumbent
upon the parties to return what they have received from said sale. Accordingly, Erlinda
and the rest of petitioners (as Pedro's heirs) are entitled to the return of the subject land Whenever both the landowner and the builder/planter/sower are in good faith (or in bad
as stipulated during .the pre-trial faith, pursuant to the afore-cited provision), the landowner is given two (2) options under
Article 44836 of the Civil Code, namely: (a) he may appropriate the improvements for
himself after reimbursing the buyer (the builder in good faith) the necessary and useful
III. expenses under Articles 54637 and 54838 of the Civil Code; or (b) he may sell the land
to the buyer, unless its value is considerably more than that of the improvements, in
As correctly argued by petitioners, it is more accurate to apply31 the rules on accession which case, the buyer shall pay reasonable rent.39
with respect to immovable property, specifically with regard to builders, planters, and
sowers,32 as this case involves a situation where the landowner (petitioners) is different Applying the aforesaid rule in this case, under the first option, petitioner may
from the owner of the improvement built therein, i.e., the three (3)-storey building appropriate for themselves the three (3)-storey building on the subject land after
(respondents). Thus, there is a need to determine whether petitioners as landowners payment of the indemnity provided for in Articles 546 and 548 of the Civil Code, as
on the one hand, and respondents on the other, are in good faith or bad faith. applied in existing jurisprudence.1âwphi1 Under this option, respondents would have a
right of retention over the three (3)-storey building as well as the subject land until
The terms builder, planter, or sower in good faith as used in reference to Article 448 of petitioners complete the reimbursement. Under the second option, petitioners may sell
the Civil Code, refers to one who, not being the owner of the land, builds, plants, or the subject land to respondents at a price equivalent to the current market value thereof.
sows on that land believing himself to be its owner and unaware of the defect in his title However, if the value of the subject land is considerably more than the value of the
or mode of acquisition. "The essence of good faith lies in an honest belief in the validity three (3)-storey building, respondents cannot be compelled to purchase the subject
of one's right, ignorance of a superior claim, and absence of intention to overreach land. Rather, they can only be obliged to pay petitioners reasonable rent.40
another."33 On the other hand, bad faith may only be attributed to a landowner when
the act of building, planting, or sowing was done with his knowledge and without Thus, following prevailing jurisprudence, the instant case is remanded to the court a
opposition on his part.34 quo for the purpose of determining matters necessary for the proper application of
Articles 448 and 453, in relation to Articles 546 and 548 of the Civil Code,41 as applied
In this case, it bears stressing that the execution of the Deed of Sale involving the in existing jurisprudence.
subject land was done in 1992. However, and as keenly pointed out by Justice Alfredo
Benjamin S. Caguioa during the deliberations of this case, Teresita was apprised of
Pedro's death as early as 1990 when she went on a vacation in the Philippines. 35 As
BANK OF THE PHILIPPINE ISLANDS, Petitioner, vs. VICENTE VICTOR C.  Thus, Yap wrote a letter dated December 26, 1988 12 to Garcia informing him that the
SANCHEZ, HEIRS OF KENNETH NEREO SANCHEZ, represented by FELISA two (2) checks were dishonored and asking that the checks be replaced within five (5)
GARCIA YAP, and HEIRS OF IMELDA C. VDA. DE SANCHEZ, represented by days from receipt of the letter. Such request was left unheeded.
VICENTE VICTOR C. SANCHEZ, Respondents. G.R. No. 179518 November 19,
2014  On January 10, 1989, Yap informed Garcia in a letter13 that she and Vicente were
rescinding the Agreement while demanding the return of the original owner’s copy of
FACTS: TCT 156254. This prompted Garcia to offer two (2) manager’s checks in the aggregate
amount of ₱300,000 which Yap flatly refused, reiterating the rescission of their
Agreement and demanding for the return of all documents entrusted to Garcia through
 Vicente Victor C. Sanchez (Vicente), Kenneth Nereo Sanchez and Imelda C. V da. De
a January 21, 1989 letter.14
Sanchez owned a parcel of land located at No. 10 Panay A venue, Quezon City
consisting of 900 square meters. The property was registered under Transfer Certificate
of Title No. (TCT) 156254 of the Registry of Deeds of Quezon City (the Subject  However, in a letter dated January 27, 1989,15 Garcia’s counsel, Atty. Francisco Beato,
Property).2 Jr. (Beato), informed Yap that they (Garcia, Vicente and Yap) had an agreement that
the ₱800,000 balance of the purchase price was due to be paid by Garciaonly upon
Yap and Vicente’s payment of the realty, inheritance and capital gains taxes due onthe
 On October 10, 1988, Jesus V. Garcia (Garcia), doing business under the name
transfer of the property. Thus, Garcia effectively refused to return the documents and
TransAmerican Sales and Exposition, Inc. (TSEI), wrote a letter3 to Vicente offering to
to vacate the subject property. Yap referred Beato’s letter to her own counsel, Atty.
buy the Subject Property for One Million Eight Hundred Thousand Pesos (₱1,800,000)
Julian S. Yap, who wrote back in a letter dated February 16, 1989, refuting the claim of
Garcia that the ₱800,000 was not yet dueand reiterating their decision to rescind the
 The offer was good for only seven (7) days. The period elapsed with the parties failing Agreement and demanding that Garcia vacate the property and return the documents
to come to an agreement. that were surrendered to him by Yap.1

 Sometime in the third week of October 1988, Felisa Yap (Yap), the widow of Kenneth  Subsequently, the first four (4) checks were deposited with no issue. However, the last
Nereo Sanchez, and Garcia had a meeting at the Quezon City Sports Club wherein the two (2) checks, amounting to ₱400,000 each, were dishonored for the reason of "DAIF"
parties agreed to the sale of the subject property. or drawn against insufficient funds.11

 Pursuant to this agreement, Yap turned over to Garcia the original owner’s copy of TCT  Thus, Yap wrote a letter dated December 26, 1988 12 to Garcia informing him that the
156254, the copy of the filed Application for Restitution of Title to the property, and two (2) checks were dishonored and asking that the checks be replaced within five (5)
copies of all receipts for the payment of real estate taxes on the property,while Garcia days from receipt of the letter. Such request was left unheeded.
paid Yap 50,000 as earnest money.6
 On January 10, 1989, Yap informed Garcia in a letter13 that she and Vicente were
 Afterwards, Yap required the occupants of the subject property to vacate the same. rescinding the Agreement while demanding the return of the original owner’s copy of
Immediately after it was vacated, Garcia, without Yap’s knowledge and consent, took TCT 156254. This prompted Garcia to offer two (2) manager’s checks in the aggregate
possession of the lot and installed his own caretaker thereon with strict instructions not amount of ₱300,000 which Yap flatly refused, reiterating the rescission of their
to allow anyone to enter the property. Yap later learned that Garcia had also Agreement and demanding for the return of all documents entrusted to Garcia through
demolished the house on the property and advertised the construction and sale of a January 21, 1989 letter.14
"Trans American Townhouse V" thereon. The foregoing developments notwithstanding
and despite numerous demands, Garcia failed to pay the balance of the purchase price
 However, in a letter dated January 27, 1989,15 Garcia’s counsel, Atty. Francisco Beato,
as agreed upon.7
Jr. (Beato), informed Yap that they (Garcia, Vicente and Yap) had an agreement that
the ₱800,000 balance of the purchase price was due to be paid by Garciaonly upon
 Then, on December 5, 1988, Yap was informed that the checks representing the Yap and Vicente’s payment of the realty, inheritance and capital gains taxes due onthe
purchase price of the subject property were ready but that Vicente must pick up his transfer of the property. Thus, Garcia effectively refused to return the documents and
checks personally. On December 8, 1988, Vicente came to Manila from Laguna and to vacate the subject property. Yap referred Beato’s letter to her own counsel, Atty.
proceeded to Garcia’s office to get the checks. However, out of the six (6) checks that Julian S. Yap, who wrote back in a letter dated February 16, 1989, refuting the claim of
were presented to them, four (4) of them were post-dated, further delaying their overdue Garcia that the ₱800,000 was not yet dueand reiterating their decision to rescind the
paymen Agreement and demanding that Garcia vacate the property and return the documents
that were surrendered to him by Yap.1
 Subsequently, the first four (4) checks were deposited with no issue. However, the last
two (2) checks, amounting to ₱400,000 each, were dishonored for the reason of "DAIF"  The RTC rendered a Decision in favor of the Sanchezes as plaintiffs declaring that the
or drawn against insufficient funds.11 Sanchezes have the right to rescind the Agreement they entered into with Garcia and
TSEI under the Agreement. In fact, the RTC enunciated that because the Agreement having bought the townhouse units in early 1989, it can be seen that the preselling was
is in the nature of a contract to sell, the ownership over the subject property remained done almost immediately after the Sanchezes and Garcia/TSEI agreed on the terms of
with the Sanchezes as the suspensive condition––that the check payments shall be the sale of the subject lot, or shortly after Garcia and TSEI had taken over the property
honored––was not complied with. Thus, the RTC concluded that there was not even and demolished the old house built thereon. In either case, the pre-selling already
any need for rescission in this case. Moreover, the RTC found that TSEI and Garcia commenced and was continuing when the two postdated checks amounting to the
were builders in bad faith as the Sanchezes never consented to the construction of the remaining balance of ₱800,000 bounced. And whenthe Sanchezes informed Garcia
townhouses. and TSEI that they were rescinding the Agreement in early 1989, the intervenors
apparently werealready in the process of closing their deals with TSEI for the purchase
 The Court of Appeals affirmed the RTC decision. of townhouse units. As to the transactions between FEBTC and Garcia/TSEI and that
between VTCI and Garcia/TSEI, it is suffice to state that the Sanchezes, despite the
actions they undertook, were not aware of the said dealings.
ISSUE:

Garcia, TSEI, BPI, and the intervenors acted in bad faith


1. Whether the parties all acted in bad faith
2. Whether there was a valid rescission of the Agreement between the Sanchezes a. Garcia and TSEI acted in bad faith
and TSEI/Garcia

The Court agrees with both the RTC and the CA that Garcia and/or TSEI are builders
RULING:
in bad faith. They knew for a fact that the property still belonged to the Sanchezes and
yet proceeded to build the townhouses not just without the authority of the landowners,
1. The Sanchez did not act in bad faith while Garcia, TSEI, BPI, and the but also against their will. Thus, the CA wrote:
intervenors acted in bad faith
Anent the improvements constructed on the subject property, the defendants were
The Sanchezes did not act in bad faith undoubtedly builders in bad faith. As borne out by the evidence, the defendants took
possession of the subject property and constructed the 20-unit townhouses thereon
Contrary to the finding of the CA, the Sanchezes cannot be considered to be in bad without prior consent of the plaintiffs-appellants/appellees. On top of this, defendant
faith for failing to file an action for injunction against the construction of the townhouses Garcia was aware that the defendants have not as yet fully paid the purchase price
on the subject property thereof and therefore are not yet owner/s of the subject property. In fact, no contract of
sale over the subject property has been executed by the plaintiffs/appellants/appellees
in defendants’ favor.54 x x x
Article 453 of the Civil Code relevantly states:

The next query: are the intervenors purchasers in good faith?


Article 453. If there was bad faith, not only on the part of the person who built, planted
or sowed on the land of another, but also on the part of the owner of such land, the
rights of one and the other shall be the same as though both had acted in good faith. The Court rules otherwise.

It is understood that there is bad faith on the part of the landowner whenever the act b. Intervenors Sps. Caminas,
was done with his knowledge and without opposition on his part. (emphasis supplied) Maniwang, Tulagan, and

The second paragraph of the provision clearly reads that a landowner is considered in Marquez acted in bad faith
bad faith if he does not oppose the unauthorized construction thereon despite
knowledge of the same. It does not, however, state what form such opposition should Prevailing jurisprudence reveals the following established rules:
take. The fact of the matter is that the Sanchezes did take action to oppose the
construction on their property by writing the HLURB and the City Building Official of
1. Well settled is the rule that all persons dealing with property covered by a
Quezon City. As a result, the HLURB issued two (2) Cease and Desist Orders and
torrens certificate of title are not required to go beyond what appears on the
several directives against Garcia/TSEI which, however, were left unheeded. In addition,
face of the title. Whenthere is nothing on the certificate of title to indicate any
the Sanchezes could not be faulted for not having been able to enjoin the sale of the
cloud or vice in the ownership of the property, or any encumbrance thereon,
townhouses by Garcia and TSEI to the intervenors Sps. Caminas, Maniwang, Tulagan,
the purchaser is not required to explore further than what the torrens title upon
and Marquez who bought their townhouse units during the sameperiod that the
its face indicates in quest for any hidden defect or inchoate right that may
Sanchezes were demanding the full payment of the subject lot and were exercising
subsequently defeat his right thereto.55
their right of extrajudicial rescission of the Agreement. As the intervenors asserted
2. This rule, however, admits of an exception as where the purchaser or Thirdly, the intervenors should havebeen suspicious of the explanation of Garcia that
mortgagee has knowledge of a defect or lack of title in the vendor, or that he TCT No. 383697, reflecting TSEI as the owner of the property, has been burned and
was aware ofsufficient facts to induce a reasonably prudent man to inquire that he is in the process of reconstituting the title. Before signing the contract of sale or
into the status of the property in litigation.56 (emphasis supplied) contract to sell, they should have asked Garcia where the reconstitution case has been
filed or is pending and proceeded to verify with the said court the status of the
3. Likewise, one who buys property withfull knowledge of the flaws and defects reconstitution. Had they done so, they would have known that neither Garcia nor TSEI
in the title of the vendor is enough proof of his bad faith and estopped from had a deed of absolute sale executed in their favor over the lot in question. The truth of
claiming that he acquired the property in good faith against the owners.57 the matter is that it is the duplicate certificate of title of TCT No. 156254 that has been
lost or misplaced, and is being sought to be reconstituted, not TCT No. 383697. Had
intervenors been prudent enough to verify with the court the status of the alleged TCT
4. To prove good faith, the following conditions must be present: (a) the seller No. 383697, they would have known that Garcia planned to deceive them in the sale of
is the registered owner of the land; (b) the owner is in possession thereof; and the subject property.
(3) at the time of the sale, the buyer was not aware of any claim or interest of
some other personin the property, or of any defect or restriction in the titleof
the seller or in his capacity to convey title to the property. All these conditions Fourthly, the intervenors knew that they were buying a townhouse over a subdivision
must be present, otherwise, the buyer is under obligation to exercise extra lot from TSEI and Garcia. Such being the case, they should have verified with the
ordinary diligence by scrutinizing the certificates of title and examining all HLURB whether said project is registered with said housing agency and if a license to
factual circumstances to enable him to ascertain the seller’s title and capacity sell has been issued to TSEI or Garcia. Had they made such an inquiry, they would
to transfer any interest in the property.58 have known that instead of a permit for the project and a license to sell the property, a
cease and desist order was issued by the HLURB precisely to enjoin TSEI and Garcia
from selling said property to the public. Similarly, they could have inquired from the City
The factual milieu of the case reveals that intervenors are buyers in bad faith for the Building Official of Quezon City if a building permit was issued to TSEI and Garcia for
following reasons, viz: the construction of the townhouses, which would have yielded the same negative result.

Firstly, they admitted that they executed either contracts of sale or contracts to sell c. VCTI acted in bad faith
indicating that the lot is covered by TCT No. 156254 registered under the name of the
respondent Sanchezes. While the established rule is that persons dealing with property
covered by a Torrens certificate of title are not required togo beyond what appears on As compared to the other purchasers,the Deeds of Absolute Sale of intervenor VTCI
the face of the title, intervenors cannot seek haven from such doctrine as the title of the cited TCT 383697 inthe name of VTCI and not TCT 156254. Nevertheless, the Court
lot does not pertain to the vendor (Garcia or TSEI) they dealt with. The fact that the lot finds that respondent VTCI is a purchaser in bad faith for the following reasons:
being sold to them belonged to persons other than TSEI or Garcia should have driven
the intervenors, as prudence would dictate, to investigate the true status of the property. Firstly, respondent VTCI has not shown that it verified with the RD if the alleged TCT
They should have gone to the Register of Deeds of Quezon City (RD) to verifyif in fact 383697 of respondent TSEI is valid and genuine. It did not present any certified true
TCT No. 156254 had already been cancelled and a new title has been issued to TSEI copy of said TCT 383697 to demonstrate that based on the RD’s records, said title
or Garcia. They should have asked for the deed of absolute sale filed and registered exists and that it is genuine and valid. It should be remembered that the duplicate
with the RD to find out if the Sanchezes indeedsold the lot in question to TSEI. They certificate of TCT 156254 was lost and subject of reconstitution. Yet respondents
could have verified from the primary entry book of said office if the deed of absolute Garcia and TSEI were not able to show that it was already reconstituted. In addition,
sale from the Sanchezes in favor of TSEI was registered in said book, which, under the there was no deed of absolute sale executed by the Sanchezes in favor of TSEI as the
Property Registration Decree (PD No. 1529), is considered as an effective and legal latter failed to pay the last two (2) installments and subsequently, the agreement to sell
notice to third persons and the whole world of such transfer. Evidently, the intervenors was rescinded by the Sanchezes for non-payment.There being no deed of absolute
failed to do so. sale, there is, consequently, no ground for the RD to cancel TCT No. 156254 and
subsequently issue TCT 383697 in the name of TSEI. This goes to show that TCT
Secondly, the intervenors know, based on the contract of sale or contract to sell, that 383697 of TSEI appears to be spurious and a fake title. This is buttressed by the fact
the property isregistered under TCT No. 156254 in the name of the Sanchezes. As that the date of the issuance of TCT 383697 is June 9, l988, pre-dating the execution
such, they should have insisted that they talk to the Sanchezes before executing said of the Agreement between the Sanchezes and TSEI on December 8, l988. With the
conveyances. Had they done so, they would have known that the Sanchezes have not failure of VTCI to exert earnest efforts to verify the authenticity of TCT 383697, then it
executed a written deed of absolute sale in favor of TSEI for the latter’s failure to pay is not a purchaser in good faith.
the consideration in full. Having failed to ferret out the truth from the Sanchezes,
intervenors cannot be considered innocent purchasers for failure to exercise utmost Secondly, Garcia and TSEI stopped the construction of the townhouses on March 30,
caution and extra diligence in determining the true owner of the property. l989 pursuant to the CDO of the HLURB. Thus, the townhouses were not fully finished
and completed. Yet on December 27, l989 (date of notarization), VTCI entered into
three (3) Deeds of Absolute sale over three (3) townhouses on three (3) lots covered
by TCT 383697 and despite the non-completion of the townhouses, it still fully paid the Garcia/TSEI to submit a Special Power of Attorney (SPA) in their favor authorizing them
uniform price of ₱700,000 for the townhouse on each of the 3 lots – 1st lot with an area to mortgage the subject property covered by TCT 156254.
of 52.5 square meters; 2nd lot with an area of 72.5 square meters; and 3rd lot with an
area of 42.5 square meters. The price of ₱700,000 was even applied to all lots even if Secondly, considering that Garcia/TSEI were already selling the townhouse units to the
ordinarily a bigger lot will commend a higher price. These are doubtful transactions public as early as January 1989, FEBTC was also remiss in not requiring Garcia/TSEI
since a man of average intellect will not fully pay the price of a townhouse which has to submit a written approval from the HLURB for the mortgage of the subject property
not yet been completed. The alleged purchases are not in accord with the normal where the townhouse units were being constructed as required under Sec. 18 59 of
business practice and common behavior of an ordinary human being. These Presidential Decree No. (PD) 957.60 Thirdly, considering further that Garcia presented
circumstances sway the Court to believe that said alleged conveyances are not genuine the Agreement between the Sanchezes and Garcia/TSEI asbasis for ownership of the
and that VTCI is not a purchaser in good faith. subject property covered by TCT 156254, FEBTC was remiss in neither ascertaining
whether the full payment of the ₱1.8 million covered by six (6) checks in view of the
Thirdly, with the CDO and the warnings to the public and prospective buyers published proviso number 6 of the Agreement nor requiring the presentment of the EXTRA-
in the Philippine Daily Inquirer on April 16, 1989 and in the Manila Bulletin on April 19, JUDICIAL SETTLEMENT OF ESTATE WITH SALE from the Sanchezes in favor of
2014, VTCI should have been aware of the irregularities in the proposed sale of Garcia/TSEI.
townhouses by Garcia and TSEI. The failure of VTCI to heed the warnings and
prohibition to buy said townhouses tends to show that said respondent is not a Fourthly, FEBTC was again negligent in not scrutinizing the TCT 383697 considering
purchaser in good faith. that the title has the purported issuance date of June 9, 1988 way before the December
Agreement was executed and when the loan was negotiated. More, the purported
Fourthly, with the issuance of the CDO by the HLURB and the notices in the major issuance of TCT 383697 was made more than six (6) months before Garcia/TSEI
dailies, VTCI should have inquired with the said HLURB if Garcia and TSEI have a approached the bank for the loan. Thus, FEBTC should have been placed on guard as
permit to sell the townhouses. Had it done so, it would have discovered that the project, to why Garcia/TSEI initially gave it TCT 156254 in the name of the Sanchezes when
as it lacks the necessary permits, is unauthorized and that the title over the townhouses TCTC 383697 was purportedly already issued and in Garcia’s possession way before
is questionable. the bank loan was negotiated. Again, FEBTC did not exercise the due diligence
required of banks.
Fifthly, a buyer of a townhouse will ordinarily visit the project site and look at and
investigate the lot, the title and the townhouses being sold. If it inspected the site of the Fifthly, the Court notes that FEBTC released portions of the loan proceeds in April even
construction project, it would have known from the other purchasers that the project before it approved the loan secured by a real estate mortgage on May 22, 1989. And
has no permit from the HLURB and that construction has been stopped because of the more anomalous is the fact that FEBTC had TCT 383697 verified for its veracity and
CDO. Had VTCI done the inspection and investigation, then it would not have entered genuineness way after it approved the loan to Garcia/TSEI. The Certification61 from the
into the deeds of absolute sale with Garcia and TSEI. Thus, respondent VTCI cannot Register of Deeds was issued only on June 13,1989 upon the request of Garcia.
be considered as a purchaser in good faith.
Verily, given the foregoing anomalies, the general rule that a mortgagee need not look
From the foregoing, the fact that all the intervenors turned a blind eye to the flaws and beyond the titledoes not apply tobanks and other financial institutions as greater care
defects in the ownership of TSEI over the property and miserably failed to undertake and due diligence are required of them,62 and FEBTC should have exercised the
measures required of a reasonably prudent man to investigate the title of the pseudo appropriate due diligence review and made the requisite inquiries about the subject
owner and the legality of the townhouse project constitutes bad faith for which there is property which was offered to secure the loan applied for by Garcia/TSEI under a real
no available relief under the law. estate mortgage. FEBTC (now BPI) was negligent and cannot be considered as a
mortgagee in good faith.
d. BPI cannot be considered a mortgagee in good faith
The effects of attributing bad faith to the intervenors, BPI, TSEI, and Garcia
Even as the intervenors have been found to be in bad faith, BPI, the successor of
FEBTC, cannot be considered a mortgagee in good faith, considering the glaring a. Rescission of the Agreement was not barred by the subsequent transfer
anomalies in the loan transaction between TSEI and FEBTC. This can be gleaned from
several undisputed factual circumstances: Article 1191 of the Civil Code states that rescission is available to a party in a reciprocal
obligation where one party fails to comply therewith:
Firstly, when Garcia gave TCT 156254 to FEBTC for the processing of a loan secured
by a mortgage, it indubitably showed that Garcia/TSEI did not yet own the subject Article 1191. The power to rescind obligations is implied in reciprocal ones, in case one
property as said title was in the name of the Sanchezes. But FEBTC did not require of the obligors should not comply with what is incumbent upon him.
The injured party may choose between the fulfillment and the rescission of the Should the Sanchezes opt to ask from the intervenors the value of the land, the case
obligation, with the payment of damages in either case. He may also seek rescission, shall be remanded to the RTC for the sole purpose of determining the fair market value
even after he has chosen fulfillment, if the latter should become impossible. of the lot at the time the same were taken from the Sanchezes in 1988.

The court shall decree the rescission claimed, unless there be just cause authorizing If the Sanchezes decide to appropriate the townhouses, other structures and
the fixing of a period. improvements as their own pursuant to Article 449 of the Civil Code, then the
intervenors-purchasers Caminas, Maniwang, Tulagan, Marquez and VCTI shall be
This is understood to be without prejudice to the rights of third persons who have ordered to vacate said premises within a reasonable time from notice of the finality of
acquired the thing, in accordance with Articles 1385 and 1388 and the Mortgage Law. the decision by the Sanchezes. They have a right to recover their investment in the
(emphasis supplied) Article 1385 of the Civil Code does provide that rescission shall townhouses from Garcia and TSEI. If the Sanchezes do not want to make use of the
not take place if the subject matter of the prior agreement is already in the hands of a townhouses and improvements on the subject lot, thenthe purchasers can be ordered
third party who did not act in bad faith, to wit: to demolish said townhouses or if theydon’t demolish the same within a reasonable
time, then it can be demolished at their expense. On the 3rd option, if the Sanchezes
do not want toappropriate the townhouses or have the same demolished, then they can
Article 1385. Rescission creates the obligation to return the things which were the ask that the townhouse purchasers pay to them the fair market value of the respective
object of the contract, together with their fruits, and the price with its interest; areas allotted to their respective townhouses subject of their deeds of sale.
consequently, it can be carried out only when he who demands rescission can return
whatever he may be obliged to restore. Neither shall rescission take place when the
things which are the object of the contract are legally in the possession of third persons
who did not act in bad faith.

In this case, indemnity for damages may be demanded from the person causing the
loss. (emphasis added)

In the extant case, the failure of TSEI to pay the consideration for the sale of the subject
property entitled the Sanchezes to rescind the Agreement. And in view of the finding
that the intervenors acted in bad faith in purchasing the property, the subsequent
transfer in their favor did not and cannot bar rescission.

b. The Sanchezes are to elect their option under the Arts.449-450 of the New Civil Code

Moreover, bad faith on the part of TSEI, Garcia and the intervenors leads to the
application of Articles 449-450 of the New Civil Code, which provide:

Article 449. He who builds, plants or sows in bad faith on the land of another, loses
what is built, planted or sown without right to indemnity.

Article 450. The owner of the land on which anything has been built, planted or sown
in bad faith may demand the demolition of the work, or that the planting or sowing be
removed, in order to replace things in their former condition at the expense of the
person who built, planted or sowed; or he may compel the builder or planter to pay the
price of the land, and the sower the proper rent.

Consequently, the Sanchezes have the following options: (1) acquire the property with
the townhouses and other buildings and improvements that may be thereon without
indemnifying TSEI or the intervenors;63 (2) demand from TSEI or the intervenors to
demolish what has been built on the property at the expense of TSEI or the intervenors;
or (3) ask the intervenors to pay the price of the land.64 As such, the Sanchezes must
choose from among these options within thirty (30) days from finality of this Decision.
FLORENTINO, TROADIO and PEDRO, all surnamed OCHOA, petitioners, vs. RULING: Yes
MAURO APETA and APOLONIA ALMAZAN, respondents. G.R. No.
146259 September 13, 2007 The issue raised by petitioners that they are the actual owners of Lot No. 1580 is factual
in nature and requires a review of the pieces of evidence presented by the parties.
FACTS: Thus, we can no longer pass upon and evaluate the lower courts’ finding that based on
the evidence presented before them, specifically the result of the resurvey conducted
 Since 1910, the above-named petitioners and their predecessors-in-interest have by Engr. Romulo Unciano, respondents are "the true and lawful owners of Lot 1580."
been occupying Lot No. 1580 consisting of 886 square meters situated in
Malaban, Biñan, Laguna. The lot is covered by Transfer Certificate of Title (TCT) Anent petitioners’ second contention that respondents’ action has been barred by
No. T-40624 of the Registry of Deeds of that province. They built their houses and prescription, suffice it to state that no title to registered land in derogation to that of the
apartment building thereon. registered owner shall be acquired by prescription or adverse possession. 6 Neither can
prescription be allowed against the hereditary successors of the registered owner,
 Sometime in May 10, 1982, Mauro Apeta and Apolonia Almazan, respondents, because they step into the shoes of the decedent and are merely the continuation of
found that they are the true owners of Lot No. 1580 being occupied by petitioners. the personality of their predecessor-in-interest.7

 On January 22, 1988, respondents filed with the Regional Trial Court (RTC), Verily, the Court of Appeals did not err when it ruled that respondents are the true and
Branch 24, Biñan, Laguna a complaint for recovery of possession and damages lawful owners of Lot No. 1580. Hence, they "should now be placed in possession
against petitioners, docketed as Civil Case No. B-2777. Respondents alleged in thereof."
the main that they are the lawful owners of Lot No. 1580 covered by Certificate of
Title No. RT-599 (10731) issued by the Registry of Deeds of Laguna. Parenthetically, considering that petitioners and their predecessors-in-interest have
built their houses and apartment building on Lot No. 1580, should respondents be
 Petitioners specifically denied the allegations in the complaint, contending that allowed to take possession of those improvements? In order to settle this matter, we
they are the owners of Lot No. 1580 as shown by TCT No. T-40624 issued by the should determine whether petitioners were builders in good faith.
Registry of Deeds of Laguna.
Good faith is an intangible and abstract quality with no technical meaning or statutory
 During the proceedings before the RTC, upon agreement of the parties, the trial definition, and it encompasses, among other things, an honest belief, the absence of
judge commissioned Engr. Romulo Unciano of the Bureau of Lands of Region IV malice and the absence of design to defraud or to seek an unconscionable advantage.
to conduct a resurvey of the disputed property. The result of the resurvey It implies honesty of intention, and freedom from knowledge of circumstances which
(approved by the Bureau of Lands) shows that Lot No. 1580, occupied by ought to put the holder upon inquiry.8 The essence of good faith lies in an honest belief
petitioners, was registered in the name of Margarita Almada, respondents’ in the validity of one’s right, ignorance of a superior claim and absence of intention to
predecessor-in-interest; and that the lot covered by TCT No. T-40624 is not Lot overreach another.9 Applied to possession, one is considered in good faith if he is not
No. 1580, but Lot No. 1581 registered in the name of Servillano Ochoa, aware that there exists in his title or mode of acquisition any flaw which invalidates it. 10
petitioners’ predecessor-in-interest. This lot has been occupied by Isidro Jasmin.
Using the above parameters, we are convinced that petitioners and their predecessors-
 On March 24, 1995, the trial court rendered a Decision in favor of respondents in-interest were in good faith when they built their houses and apartment building on
declaring plaintiffs as the true and lawful owners of Lot 1580 of the Biñan Estate Lot No. 1580 since they were convinced it was covered by their TCT No. T-40624.
Subdivision covered by Transfer Certificate of Title No. RT-599 (10731) and
declaring the defendants without right whatsoever to continue in possession
thereof. The following provisions of the Civil Code are relevant:

 The Court of Appeals, in its Decision dated September 8, 2000, affirmed the Article 448. The owner of the land on which anything has been built, sown or
judgment of the RTC. planted in good faith, shall have the right to appropriate as his own the works,
sowing or planting, after payment of the indemnity provided for in Articles 546
and 548, or to oblige the one who built or planted to pay the price of the land,
 Petitioners contend that Lot No. 1580 belongs to them and that respondents’
and the one who sowed, the proper rent. However, the builder or planter
action is barred by prescription.
cannot be obliged to buy the land if its value is considerably more than that of
the building or trees. In such case, he shall pay reasonable rent, if the owner
ISSUE: Whether or not the petitioners and their predecessors-in-interest were in good of the land does not choose to appropriate the building or trees after proper
faith when they built their houses and apartment building on Lot No. 1580 indemnity. The parties shall agree upon the terms of the lease and in case of
disagreement, the court shall fix the terms thereof.
Article 546. Necessary expenses shall be refunded to every possessor; but
only the possessor in good faith may retain the thing until he has been
reimbursed therefor.

Useful expenses shall be refunded only to the possessor in good faith with the
same right of retention, the person who has defeated him in the possession
having the option of refunding the amount of the expenses or of paying the
increase in value which the thing may have acquired by reason thereof.

Article 548. Expense for pure luxury or mere pleasure shall not be refunded to
the possessor in good faith; but he may remove the ornaments with which he
has embellished the principal thing if it suffers no injury thereby, and if his
successors in the possession do not prefer to refund the amount expended.

Under the foregoing provisions, the landowner can make a choice - either by
appropriating the building by paying the proper indemnity or obliging the builder to pay
the price of the land. The choice belongs to the owner of the land, a rule that accords
with the principle of accession that the accessory follows the principal and not the other
way around. He must choose only one.

Following the above provisions, respondents, as owners of Lot No. 1580, may choose
between appropriating as their own the houses and apartment building constructed
thereon by petitioners and their predecessors-in-interest by paying the proper
indemnity or value; or obliging petitioners to pay the price of Lot No. 1580 which is not
more than that of the improvements.

WHEREFORE, we DENY the petition. The assailed Decision and Resolution of the
Court of Appeals in CA G.R. CV No. 56109 are AFFIRMED with MODIFICATION in
the sense that respondents have the option to pay for the houses and apartment
building constructed by petitioners and their predecessors-in-interest on Lot No. 1580;
or to oblige petitioners to pay the price of the lot in an amount not more than the value
of the said improvements.

SO ORDERED.
DESAMPARADO VDA. DE NAZARENO and LETICIA NAZARENO  Antonio Nazareno filed a motion for reconsideration with respondent Rolleo Ignacio,
TAPIA, petitioners, vs. THE COURT OF APPEALS, MR. & MRS. JOSE SALASALAN, Undersecretary of the Department of Natural Resources and Officer-in-Charge of the
MR. & MRS. LEO RABAYA, AVELINO LABIS, HON. ROBERTO G. HILARIO, Bureau of Lands who denied the motion. Respondent Director of Lands Abelardo Palad
ROLLEO I. IGNACIO, ALBERTO M. GILLERA and HON. ABELARDO G. PALAD, then ordered him to vacate the portions adjudicated to private respondents and remove
JR., in their official and/or private capacities, respondents. G.R. No. 98045 June whatever improvements they have introduced thereon. He also ordered that private
26, 1996 respondents be placed in possession thereof.

The only issue involved in this petition is whether or not petitioners exhausted  Upon the denial of the late Antonio Nazareno's motion for reconsideration, petitioners
administrative remedies before having recourse to the courts. Desamparado Vda. de Nazareno and Leticia Tapia Nazareno, filed a case before the
RTC. The RTC dismissed the complaint for failure to exhaust administrative remedies
FACTS: which resulted in the finality of the administrative decision of the Bureau of Lands.

The CA affirmed the decision of the RTC dismissing the complaint.


 The subject of this controversy is a parcel of land situated in Telegrapo, Puntod,
Cagayan de Oro City. Said land was formed as a result of sawdust dumped into the
dried-up Balacanas Creek and along the banks of the Cagayan river. ISSUE: Whether or not the subject land is public land

 Sometime in 1979, private respondents Jose Salasalan and Leo Rabaya leased the RULING: The Subject land is a public land.
subject lots on which their houses stood from one Antonio Nazareno, petitioners'
predecessor-in-interest. In the latter part of 1982, private respondents allegedly
Petitioners claim that the subject land is private land being an accretion to his titled
stopped paying rentals. As a result, Antonio Nazareno and petitioners filed a case for
property, applying Article 457 of the Civil Code which provides:
ejectment with the Municipal Trial Court. A decision was rendered against private
respondents, which decision was affirmed by the Regional Trial Court.
To the owners of lands adjoining the banks of rivers belong the
 Private respondents filed a case for annulment of judgment before the Regional Trial accretion which they gradually receive from the effects of the current
Court of Misamis Oriental, Branch 24 which dismissed the same. Antonio Nazareno of the waters.
and petitioners again moved for execution of judgment but private respondents filed
another case for certiorari with prayer for restraining order and/or writ of preliminary In the case of Meneses v. CA, 2 this Court held that accretion, as a mode of acquiring
injunction with the Regional Trial Court of Misamis Oriental, Branch 25 which was property under Art. 457 of the Civil Code, requires the concurrence of these requisites
likewise dismissed. The decision of the lower court was finally enforced with the private : (1) that the deposition of soil or sediment be gradual and imperceptible; (2) that it be
respondents being ejected from portions of the subject lots they occupied. the result of the action of the waters of the river (or sea); and (3) that the land where
accretion takes place is adjacent to the banks of rivers (or the sea coast). These are
 Before he died, Antonio Nazareno caused the approval by the Bureau of Lands of the called the rules on alluvion which if present in a case, give to the owners of lands
survey plan designated as Plan Csd-106-00571 with a view to perfecting his title over adjoining the banks of rivers or streams any accretion gradually received from the
the accretion area being claimed by him. Before the approved survey plan could be effects of the current of waters.
released to the applicant, however, it was protested by private respondents before the
Bureau of Lands. For petitioners to insist on the application of these rules on alluvion to their case, the
above-mentioned requisites must be present. However, they admit that the accretion
 In compliance with the order of respondent District Land Officer Alberto M. Gillera, was formed by the dumping of boulders, soil and other filling materials on portions of
respondent Land Investigator Avelino G. Labis conducted an investigation and the Balacanas Creek and the Cagayan River bounding their land. 3 It cannot be
rendered a report to the Regional Director recommending that Survey Plan No. MSI- claimed, therefore, that the accumulation of such boulders, soil and other filling
10-06-000571-D (equivalent to Lot No. 36302, Cad. 237) in the name of Antonio materials was gradual and imperceptible, resulting from the action of the waters or the
Nazareno, be cancelled and that private respondents be directed to file appropriate current of the Balacanas Creek and the Cagayan River. In Hilario v. City of
public land applications. Manila, 4 this Court held that the word "current" indicates the participation of the body
of water in the ebb and flow of waters due to high and low tide. Petitioners' submission
 Based on said report, respondent Regional Director of the Bureau of Lands Roberto not having met the first and second requirements of the rules on alluvion, they cannot
Hilario rendered a decision ordering the amendment of the survey plan in the name of claim the rights of a riparian owner.
Antonio Nazareno by segregating therefrom the areas occupied by the private
respondents who, if qualified, may file public land applications covering their respective In any case, this court agrees with private respondents that petitioners are estopped
portions. from denying the public character of the subject land, as well as the jurisdiction of the
Bureau of Lands when the late Antonio Nazareno filed his Miscellaneous Sales
Application MSA (G-6) 571. 5 The mere filing of said Application constituted an
admission that the land being applied for was public land, having been the subject of
Survey Plan No. MSi-10-06-000571-D (Equivalent to Lot No. 36302, Cad-237) which
was conducted as a consequence of Antonio Nazareno's Miscellaneous Sales
Application wherein said land was described as an orchard. Said description by Antonio
Nazareno was, however, controverted by respondent Labis in his investigation report
to respondent Hilario based on the findings of his ocular inspection that said land
actually covers a dry portion of Balacanas Creek and a swampy portion of Cagayan
River. The investigation report also states that, except for the swampy portion which is
fully planted to nipa palms, the whole area is fully occupied by a part of a big concrete
bodega of petitioners and several residential houses made of light materials, including
those of private respondents which were erected by themselves sometime in the early
part of 1978. 6

Furthermore, the Bureau of Lands classified the subject land as an accretion area which
was formed by deposits of sawdust in the Balacanas Creek and the Cagayan river, in
accordance with the ocular inspection conducted by the Bureau of Lands. 7 This Court
has often enough held that findings of administrative agencies which have acquired
expertise because their jurisdiction is confined to specific matters are generally
accorded not only respect but even finality. 8 Again, when said factual findings are
affirmed by the Court of Appeals, the same are conclusive on the parties and not
reviewable by this Court. 9

It is this Court's irresistible conclusion, therefore, that the accretion was man-made or
artificial. In Republic v. CA, 10this Court ruled that the requirement that the deposit
should be due to the effect of the current of the river is indispensable. This excludes
from Art. 457 of the Civil Code all deposits caused by human intervention. Putting it
differently, alluvion must be the exclusive work of nature. Thus, in Tiongco v. Director
of Lands, et al., 11 where the land was not formed solely by the natural effect of the
water current of the river bordering said land but is also the consequence of the direct
and deliberate intervention of man, it was deemed a man-made accretion and, as such,
part of the public domain.

In the case at bar, the subject land was the direct result of the dumping of sawdust by
the Sun Valley Lumber Co. consequent to its sawmill
operations. 12 Even if this Court were to take into consideration petitioners' submission
that the accretion site was the result of the late Antonio Nazareno's labor consisting in
the dumping of boulders, soil and other filling materials into the Balacanas Creek and
Cagayan River bounding his land, 13 the same would still be part of the public domain.

WHEREFORE, the petition is DISMISSED for lack of merit.


HEIRS OF EMILIANO NAVARRO, petitioner, vs. INTERMEDIATE APPELLATE Bureau of Fisheries and confirmed by the Office of the President; and that he had
COURT & HEIRS OF SINFOROSO PASCUAL, respondents. already converted the area covered by the lease into a fishpond.
G.R. No. 68166. February 12, 1997.*
HERMOSISIMA, JR., J.: During the pendency of the land registration case, that is, on November 6, 1960,
Sinforoso Pascual filed a complaint for ejectment against Emiliano Navarro, one
Summary of facts: The applicant owns the property immediately adjoining the land Marcelo Lopez and their privies, alleged by Pascual to have unlawfully claimed and
sought to be registered. His registered property is bounded on the east by the Talisay possessed, through stealth, force and strategy, a portion of the subject property. The
River, on the west by the Bulacan River, and on the north by the Manila Bay. The defendants in the case were alleged to have built a provisional dike thereon: thus they
Talisay River and the Bulacan River flow down towards the Manila Bay and act as have thereby deprived Pascual of the premises sought to be registered. This,
boundaries of the applicant’s registered land on the east and on the west. notwithstanding repeated demands for defendants to vacate the property.

The land sought to be registered was formed at the northern tip of the applicant’s land. The case was decided adversely against Pascual. Thus, Pascual appealed to the CFI-
Applicant’s registered property is bounded on the north by the Manila Bay. Balanga, Bataan. Because of the similarity of the parties and the subject matter, the
appealed case for ejectment was consolidated with the land registration case and was
FACTS: On October 3, 1946, Sinforoso Pascual, now deceased, filed an application jointly tried by the court a quo.
for foreshore lease covering a tract of foreshore land in Sibocon, Balanga, Bataan,
having an area of approximately 17 hectares. This application was denied on January Pascual and Navarro died and were substituted by petitioner-heirs and respondent-
15, 1953. So was his MR. heirs, respectively.

Subsequently, petitioners’ predecessor-in-interest, also now deceased, Emiliano CFI rendered judgment finding the subject property to be foreshore land and, being a
Navarro, filed a fishpond application with the Bureau of Fisheries covering 25 hectares part of the public domain, it cannot be the subject of land registration proceedings.
of foreshore land also in Sibocon, Balanga, Bataan. Initially, such application was Appellate court reversed CFI and granted the petition for registration of the subject
denied by the Director of Fisheries on the ground that the property formed part of the property but excluding therefrom 50 meters from corner 2 towards corner 1; and 50
public domain. Upon MR, the Director of Fisheries, on May 27, 1958, gave due course meters from corner 5 towards corner 6 of the subject lot.
to his application but only to the extent of 7 hectares of the property as may be certified
by the Bureau of Forestry as suitable for fishpond purposes. Respondent appellate court ordered the issuance of the corresponding decree of
registration in the name of private respondents and the reversion to private respondents
The Municipal Council of Balanga, Bataan, had opposed Emiliano Navarro’s of the possession of the portion of the subject property included in Navarro’s fishpond
application. Aggrieved by the decision of the Director of Fisheries, it appealed to the permit. On December 20, 1978, petitioners filed a MR. The Director of Forestry also
Secretary of Natural Resources who, however, affirmed the grant. The then Executive moved for the reconsideration of the same decision. Both were denied. It, however,
Secretary, acting in behalf of the President of the Philippines, similarly affirmed the modified its decision, to read, viz:
grant. “(3). Ordering private oppositors Heirs of Emiliano Navarro to vacate that
portion included in their fishpond permit covered by Plan Psu-175181 and
On the other hand, sometime in the early part of 1960, Sinforoso Pascual filed an hand over possession of said portion to applicants-appellants, if the said
application to register and confirm his title to a parcel of land, situated in Sibocon, portion is not within the strip of land fifty (50) meters wide along Manila Bay
Balanga, Bataan, described in Plan Psu-175181 and said to have an area of 146,611 on the northern portion of the land subject of the registration proceedings and
sqm. Pascual claimed that this land is an accretion to his property, situated in Barrio which area is more particularly referred to as fifty (50) meters from corner 2
Puerto Rivas, Balanga, Bataan. It is bounded on the eastern side by the Talisay River, towards corner 1; and fifty (50) meters from corner 5 towards corner 6 of Plan
on the western side by the Bulacan River, and on the northern side by the Manila Bay. Psu-175181.” x x x
The Talisay River as well as the Bulacan River flow downstream and meet at the Manila
Bay thereby depositing sand and silt on Pascual’s property resulting in an accretion ISSUE: WON the land sought to be registered be deemed an accretion in the sense
thereon. Sinforoso Pascual claimed the accretion as the riparian owner. that it naturally accrues in favor of the riparian owner or should the land be considered
as foreshore land
On March 25, 1960, the Director of Lands, represented by the ASG, filed an opposition
thereto stating that neither Pascual nor his predecessors-in-interest possessed RULING: NO.
sufficient title to the subject property, the same being a portion of the public domain
and, therefore, it belongs to the Republic of the Philippines. The disputed property was brought forth by both the withdrawal of the waters of
Manila Bay and the accretion formed on the exposed foreshore land by the action
Navarro claimed that the land sought to be registered has always been part of the public of the sea which brought soil and sand sediments in turn trapped by the palapat
domain, it being a part of the foreshore of Manila Bay; that he was a lessee and in and bakawan trees planted thereon by petitioner Sulpicio Pascual in 1948
possession of a part of the subject property by virtue of a fishpond permit issued by the
Anchoring their claim of ownership on Article 457 of the Civil Code, petitioners ‘Bay.—An opening into the land where the water is shut in on all sides except
vigorously argue that the disputed 14-hectare land is an accretion caused by the joint at the entrance; an inlet of the sea; an arm of the sea, distinct from a river, a
action of the Talisay and Bulacan Rivers which run their course on the eastern and bending or curbing of the shore of the sea or of a lake.’
western boundaries, respectively, of petitioners’ own tract of land.
The disputed land, thus, is an accretion not on a river bank but on a sea bank, or
Accretion as a mode of acquiring property under said Article 457, requires the on what used to be the foreshore of Manila Bay which adjoined petitioners’ own
concurrence of the following requisites: (1) that the accumulation of soil or sediment tract of land on the northern side. As such, the applicable law is not Article 457
be gradual and imperceptible; of the Civil Code but Article 4 of the Spanish Law of Waters of 1866.
(2) that it be the result of the action of the waters of the river; and
(3) that the land where the accretion takes place is adjacent to the bank of The process by which the disputed land was formed, is not difficult to discern from the
the river. facts of the case. As the trial court correctly observed:
“A perusal of the survey plan x x x of the land subject matter of these cases
Accretion is the process whereby the soil is deposited, while alluvium is the soil shows that on the eastern side, the property is bounded by Talisay River, on
deposited on the estate fronting the river bank; the owner of such estate is called the western side by Bulacan River, on the southern side by Lot 1436 and on
the riparian owner. Riparian owners are, strictly speaking, distinct from littoral the northern side by Manila Bay. It is not correct to state that the Talisay and
owners, the latter being owners of lands bordering the shore of the sea or lake Bulacan Rivers meet at a certain portion because the two rivers both flow
or other tidal waters. The alluvium, by mandate of Article 457 of the Civil Code, towards Manila Bay. The Talisay River is straight while the Bulacan River is a
is automatically owned by the riparian owner from the moment the soil deposit little bit meandering and there is no portion where the two rivers meet before
can be seen but is not automatically registered property, hence, subject to they end up at Manila Bay. The land which is adjacent to the property
acquisition through prescription by third persons. belonging to Pascual cannot be considered an accretion [caused by the action
of the two rivers].
Petitioners’ claim of ownership over the disputed property under the principle of
accretion, is misplaced. Applicant Pascual x x x has not presented proofs to convince the Court that
the land he has applied for registration is the result of the settling down on his
First, the title of petitioners’ own tract of land reveals its northeastern boundary to be registered land of soil, earth or other deposits so as to be rightfully be
Manila Bay. Petitioners’ land, therefore, used to adjoin, border or front the Manila Bay considered as an accretion [caused by the action of the two rivers]. Said Art.
and not any of the two rivers whose torrential action, petitioners insist, is to account for 457 finds no applicability where the accretion must have been caused by
the accretion on their land. In fact, one of the petitioners, Sulpicio Pascual, testified in action of the bay.”
open court that the waves of Manila Bay used to hit the disputed land being part of the
bay’s foreshore but, after he had planted palapat and bakawan trees thereon in 1948, The conclusion formed by the trial court on the basis of the aforegoing observation is
the land began to rise. that the disputed land is part of the foreshore of Manila Bay and therefore, part of the
public domain. The respondent appellate court, however, perceived the fact that
Moreover, there is no dispute as to the location of: (a) the disputed land; (b) petitioners’ petitioners’ own land lies between the Talisay and Bulacan Rivers, to be basis to
own tract of land; (c) the Manila Bay; and, (d) the Talisay and Bulacan Rivers. conclude that the disputed land must be an accretion formed by the action of the two
rivers because petitioners’ own land acted as a barricade preventing the two rivers to
Petitioners’ own land lies between the Talisay and Bulacan Rivers; in front of their land meet and that the current of the two rivers carried sediments of sand and silt
on the northern side lies now the disputed land where before 1948, there lay the Manila downwards to the Manila Bay which accumulated somehow to a 14- hectare land.
Bay. If the accretion were to be attributed to the action of either or both of the Talisay These conclusions, however, are fatally incongruous in the light of the one undisputed
and Bulacan Rivers, the alluvium should have been deposited on either or both of the critical fact: the accretion was deposited, not on either the eastern or western
eastern and western boundaries of petitioners’ own tract of land, not on the northern portion of petitioners’ land where a river each runs, but on the northern portion
portion thereof which is adjacent to the Manila Bay. Clearly lacking, thus, is the third of petitioners’ land which adjoins the Manila Bay. Worse, such conclusions are
requisite of accretion, which is, that the alluvium is deposited on the portion of further eroded of their practical logic and consonance with natural experience in the
claimant’s land which is adjacent to the river bank. light of Sulpicio Pascual’s admission as to having planted palapat and bakawan trees
on the northern boundary of their own land.
Second, there is no dispute as to the fact that petitioners’ own tract of land adjoins the In other words, the combined and interactive effect of the planting of palapat and
Manila Bay. Manila Bay is obviously not a river, and jurisprudence is already settled as bakawan trees, the withdrawal of the waters of Manila Bay eventually resulting in the
to what kind of body of water the Manila Bay is. It is to be remembered that we held drying up of its former foreshore, and the regular torrential action of the waters of Manila
that: Bay, is the formation of the disputed land on the northern boundary of petitioners’ own
“Appellant next contends that x x x Manila Bay cannot be considered as a sea. tract of land.
We find said contention untenable. A bay is part of the sea, being a mere
indentation of the same:
The disputed property is an accretion on a sea bank, Manila Bay being an inlet
or an arm of the sea; as such, the disputed property is, under Article 4 of the
Spanish Law of Waters of 1866, part of the public domain

At the outset, there is a need to distinguish between Manila Bay and Laguna de Bay.

While we held in the case of Ignacio v. Director of Lands and Valeriano that Manila
Bay is considered a sea for purposes of determining which law on accretion is
to be applied in multifarious situations, we have ruled differently insofar as
accretions on lands adjoining the Laguna de Bay are concerned.

In the cases of Government of the P.I. v. Colegio de San Jose, Republic v. Court of
Appeals, Republic v. Alagad, and Meneses v. Court of Appeals, we categorically ruled
that Laguna de Bay is a lake the accretion on which, by the mandate of Article 84
of the Spanish Law of Waters of 1866, belongs to the owner of the land
contiguous thereto.

The instant controversy, however, brings a situation calling for the application of Article
4 of the Spanish Law of Waters of 1866, the disputed land being an accretion on the
foreshore of Manila Bay which is, for all legal purposes, considered a sea. Article 4 of
the Spanish Law of Waters of August 3, 1866 provides as follows:
“Lands added to the shores by accretions and alluvial deposits caused by the
action of the sea, form part of the public domain. When they are no longer
washed by the waters of the sea and are not necessary for purposes of public
utility, or for the establishment of special industries, or for the coast-guard
service, the Government shall declare them to be the property of the owners
of the estates adjacent thereto and as increment thereof.”

In the light of the aforecited vintage but still valid law, unequivocal is the public nature
of the disputed land in this controversy, the same being an accretion on a sea
bank which, for all legal purposes, the foreshore of Manila Bay is. As part of the
public domain, the herein disputed land is intended for public uses, and “so long
as the land in litigation belongs to the national domain and is reserved for public
uses, it is not capable of being appropriated by any private person, except
through express authorization granted in due form by a competent authority.”
Only the executive and possibly the legislative departments have the right and
the power to make the declaration that the lands so gained by action of the sea
is no longer necessary for purposes of public utility or for the cause of
establishment of special industries or for coast guard services. Petitioners utterly
fail to show that either the executive or legislative department has already declared the
disputed land as qualified, under Article 4 of the Spanish Law of Waters of 1866, to be
the property of petitioners as owners of the estates adjacent thereto.

WHEREFORE, the instant Petition for Review is hereby DENIED and DISMISSED.

Costs against petitioners.

SO ORDERED.
LUCIO ROBLES, EMETERIA ROBLES, ALUDIA ROBLES and EMILIO ROBLES, The spouses Hilario Robles failed to redeem the property and so the tax declaration
petitioners, vs. COURT OF APPEALS, Spouses VIRGILIO SANTOS and BABY was transferred in the name of defendant Rural Bank. On September 25, 1987,
RUTH CRUZ, RURAL BANK OF CARDONA, Inc., HILARIO ROBLES, ALBERTO defendant Rural Bank sold the same to the Spouses Vergel Santos and Ruth Santos.
PALAD, JR. in his capacity as Director of Lands, and JOSE MAULEON in his
capacity as District Land Officer of the Bureau of Lands, respondents. In September 1987, plaintiff discovered the mortgage and attempted to redeem the
G.R. No. 123509. March 14, 2000.* property, but was unsuccessful. On May 10, 1988, defendant spouses Santos took
PANGANIBAN, J.: possession of the property in question and was able to secure Free Patent in their
names.
To be entitled to the remedy of quieting of title, petitioners must show that they
have title to the real property at issue, and that some deed or proceeding As the heirs of Silvino Robles who, likewise inherited the parcel from Leon Robles, the
beclouds its validity or efficacy. Buyers of unregistered real property, especially siblings Lucio, Emeteria, Aludia and Emilio, all surnamed Robles, commenced the
banks, must exert due diligence in ascertaining the titles of mortgagors and instant suit with the filing of their March 14, 1988 complaint against Spouses Virgilio
sellers, lest some innocent parties be prejudiced. Failure to observe such and Ruth Santos, as well as the Rural Bank of Cardona, Inc. Contending that they had
diligence may amount to bad faith and may result in the nullity of the mortgage, been in possession of the land since 1942, the plaintiff alleged, among other matters,
as well as of the subsequent foreclosure and/or auction sale. Unless the co- that it was only in September of 1987 that they came to know of the foreclosure of the
ownership is clearly repudiated, a co-owner cannot, by prescription, acquire title REM constituted thereon by the half-brother, Hilario Robles, in favor of defendant Rural
to the shares of the other co-owners. Bank; and that they likewise learned upon further inquiry, that the latter had already
sold the self-same parcel in favor of the Santos spouses. Twice amended to implead
FACTS: Leon Robles primitively owned the land situated in Kay Taga, Lagundi. Hilario Robles and, upon subsequent discovery of the issuance of Free Patent in favor
Morong, Rizal with an area of 9,985 square meters. He occupied the same openly and of the defendant spouses, the Director of Lands and the District Land Officer of the
adversely. He also declared the same in his name for taxation purposes as early as Bureau of Lands as parties-defendants. The plaintiffs’ complaint sought the following
1916 and paid the corresponding taxes thereon. When Leon Robles died, his son reliefs on the theory that the encumbrance of their half-brother, constituted on the land,
Silvino Robles inherited the land, who took possession of the land, declared it in his as well as all proceedings taken subsequent thereto, were null and void.
name for taxation purposes and paid the taxes thereon.
TC ruled that real estate mortgage allegedly executed by Hilario Robles is not valid
A parcel of land located at Kay Taga, Lagundi, Morong, Rizal. Bounded in the north by because his signature in the mortgage deed was forged based on the admission of
the property of Venancio Ablay y Simeon Ablay; in the east by the property of Veronica Andrea Robles.
Tulak y Dionisio Ablay; in the south by the property of Simeon Ablay y Dionisio Ablay; Therefore, the defendant bank did not acquire any right arising out of the
and in the west by the property of Dionisio Ablay y Simeon Ablay, with an area of 9,985 foreclosure proceedings. Consequently, defendant bank could not have
square meters, more or less, assessed in the year 1935 at P60.00. transferred any right to the spouses Santos.

Upon the death of Silvino Robles in 1942, his widow Maria de la Cruz and his children The fact that the land was covered by a free patent will not help the defendant
inherited the property. They took adverse possession of said property and paid taxes Santos any.
thereon. The task of cultivating the land was assigned to plaintiff Lucio Robles who
planted trees and other crops. He also built a nipa hut on the land. The plaintiffs There can be no question that the subject [property was held] in the concept
entrusted the payment of the land taxes to their coheir and half-brother, Hilario Robles. of owner by Leon Robles since 1916. Likewise, his successor-in-interest,
Silvino Robles, his wife Maria de la Cruz and the plaintiffs occupied the
In 1962, for unknown reasons, the tax declaration of the parcel of land in the name of property openly, continuously and exclusively until they were ousted from their
Silvino Robles was canceled and transferred to one Exequiel Ballena, father of Andrea possession in 1988 by the spouses Vergel and Ruth Santos.
Robles who is the wife of defendant Hilario Robles. Thereafter, Exequiel Ballena
secured a loan from the Antipolo Rural Bank, using the tax declaration as security. Under the circumstances, therefore, and considering that “open, exclusive and
Somehow, the tax declaration was transferred [to] the name of Antipolo Rural Bank and undisputed possession of alienable public lands for the period prescribed by
later on, was transferred [to] the name of defendant Hilario Robles and his wife. law (30 years), creates the legal fiction whereby the land, upon completion of
the requisite period, ipso jure and without the need of judicial or other action,
In 1996, Andrea Robles secured a loan from the Cardona Rural Bank, Inc., using the ceases to be public land and becomes private property. Possession of public
tax declaration as security. Andrea Robles testified without contradiction that somebody land x x x which is [of] the character and duration prescribed by the statute is
else, not her husband Hilario Robles, signed the loan papers because Hilario Robles the equivalent of an express grant from the State, considering the dictum of
was working in Marinduque at that time as a carpenter. the statute itself[:]; “The possessor x x x shall be conclusively presumed to
have performed all the conditions essential to a government grant and shall
For failure to pay the mortgage debt, foreclosure proceedings were had and defendant be entitled to a certificate of title x x x.” No proof is admissible to overcome a
Rural Bank emerged as the highest bidder during the auction sale in October 1968. conclusive presumption[,] and confirmation proceedings would be a little more
than a formality, at the most limited to ascertaining whether the possession
claimed is of the required character and length of time. Registration such, it retains the presumption of validity in the absence of a full, clear and
thereunder would not confer title, but simply recognize a title already vested. convincing evidence to overcome such presumption.
(Cruz v. IAC, G.R. No. 75042, November 29, 1988) The land in question has
become private land. The foregoing principles take even more greater [sic] when it is, moreover,
borne in mind that Hilario Robles made the following admissions in his March
Consequently, the issuance of [a] free patent title to the Spouses Vergel 8, 1989 answer, viz.:
Santos and Ruth C. Santos is not valid because at the time the property 3. The complaint filed against herein answering defendant has no
subject of this case was already private land, the Bureau of Lands having no legal basis considering that as the lawful owner of the subject real
jurisdiction to dispose of the same. property, defendant Hilario Robles has the right to mortgage the said
real property and could dispose the same in whatever manner he
CA reversed TC and held that petitioners no longer had any title to the subject property wishe[s] to do.
at the time they instituted the Complaint for quieting of title. The CA ratiocinated as
follows: Appropriately underscored by the appellants, the foregoing admission is
As correctly urged by the appellants, the plaintiff-appellees no longer had any binding against Hilario [Robles]. Judicial admissions, verbal or written, made
title to the property at the time of the institution of the instant complaint. The by the parties in the pleadings or in the course of the trial or other proceedings
latter’s claim of continuous possession notwithstanding, the aforesaid loss of in the same case are conclusive, no evidence being required to prove the
title is amply evidenced by the subsequent declaration of the subject realty for same. They cannot be contradicted unless shown to have been made through
taxation purposes not only in the name of Exequiel Ballena but also in the [a] palpable mistake or [unless] no such admission was actually made.
name of the Rural Bank of Antipolo. On the theory that tax declarations can
be evincive of the transfer of a parcel of land or a portion thereof, the court a It does not help the plaintiffs-appellees’ cause any that, aside from complying
quo clearly erred in simply brushing aside the apparent transfers [which] the with the requirements for the foreclosure of the subject real estate mortgage,
land in litigation had undergone. Whether legal or equitable, it cannot, under the appellant Rural Bank had not only relented to the mortgagor’s request to
the circumstances, be gainsaid that the plaintiff-appellees no longer had any postpone the but had likewise granted the latter’s request for an extension of
title to speak of when Exequiel Ballena executed the November 7, 1966 Deed the redemption period. Without going into minute detail in discussing the
of Absolute Sale transferring the land in favor of the spouses Hilario and Santos spouses’ rights as purchasers for value and in good, the mortgagor
Andrea Robles. and the plaintiffs-appellees cannot now be heard to challenge the validity of
Even on the theory that the plaintiffs-appellees and their half-brother, Hilario the sale of the land after admittedly failing to redeem the same within the
Robles, are co-owners of the land left behind by their common father, Silvino extension the appellant Rural Bank granted.
Robles, such title would still be effectively discounted by what could well serve
as the latter’s acts of repudiation of the co-ownership, i.e., his possession and Being dependent on the supposed invalidity of the constitution and foreclosure
declaration thereof for taxation purposes in his own name. In view of the of the subject real estate mortgage, the plaintiffs-appellees’ attack upon x x x
plaintiffs- appellees’ inaction for more than twenty (20) years from the time the Free Patent No. IV-I must necessarily fail. The trial court, therefore, misread,
subject realty was transferred in favor of Hilario Robles, the appellants and ignored the evidence o[n] record, to come up with erroneous conclusion.
correctly maintain that prescription had already set in. While it may be readily
conceded that an action to quiet title to property in the possession of the Hence, the present petition.
plaintiff is imprescriptible, it equally bears emphasis that a co-owner or, for
that matter, the said co-owner[’]s successors-in-interest who occupy the Petitioners allege that their title as owners and possessors of the disputed property is
community property other than as co-owner[s] can claim prescription as clouded by the tax declaration and, subsequently, the free patent thereto granted to
against the other co-owners . If only in this latter sense, the appellants Spouses Vergel and Ruth Santos.
correctly argue that the plaintiffs-appellees have lost their cause of action by
prescription. Petitioners anchor their claim to the disputed property on their continued and open
occupation and possession as owners thereof. They allege that they inherited it from
Over and above the foregoing considerations, the court a quo gravely erred in their father, Silvino, who in turn had inherited it from his father, Leon. They maintain
invalidating the real estate mortgage constituted on the land solely on the that after their father’s death, they agreed among themselves that Petitioner Lucio
basis of Andrea Robles’ testimony that her husband’s signature thereon was Robles would be tending and cultivating it for everyone, and that their half-brother
forged. Hilario would be paying the land taxes.

In according to the foregoing testimony x x x credibility which, while admittedly Petitioners insist that they were not aware that from 1962 until 1987, the subject
unrebutted, was altogether uncorroborated, the trial court lost sight of the fact property had been declared in the names of Exequiel Ballena, the Rural Bank of
that the assailed deed of real estate mortgage is a public document, the Antipolo, Hilario Robles, the Rural Bank of Cardona, Inc., and finally, Spouses Vergel
acknowledgment of which is a prima facie evidence of its due execution. As and Ruth Santos. Maintaining that, as co-owners of the subject property, they did not
agree to the real estate mortgage constituted on it, petitioners insist that their shares Ostensibly, the Court of Appeals failed to consider irregularities in the transactions
therein should not have been prejudiced by Hilario’s actions. involving the disputed property. First, while it was declared in the name of Exequiel in
1962, there was no instrument or deed of conveyance evidencing its transfer from the
On the other hand, Private Respondents Vergel and Ruth Santos trace their claim to heirs of Silvino to him. This fact is important, considering that the petitioners are alleging
the subject property to Exequiel Ballena, who had purportedly sold it to Hilario and continued possession of the property. Second, Exequiel was the father-in-law of
Andrea Robles. According to private respondents, the Robles spouses then mortgaged Hilario, to whom petitioners had entrusted the pay ment of the land taxes. Third,
it to the Rural Bank of Cardona, Inc.—not as co-owners but as absolute owners—in considering that the subject property had been mortgaged by Exequiel to the Rural
order to secure an agricultural loan worth P2,000. Upon their failure to pay their Bank of Antipolo, and that it was foreclosed and in fact declared in the bank’s name in
indebtedness, the mortgage was foreclosed and the property sold to the bank as the 1965, why was he able to sell it to Spouses Hilario and Andrea in 1966? Lastly,
highest bidder. Thereafter, private respondents purchased the property from the bank. inasmuch as it was an unregistered parcel of land, the Rural Bank of Cardona, Inc., did
not observe due diligence in determining Hilario’s title thereto.
ISSUES:
1. WON petitioners have the appropriate title that will entitle them to avail themselves The failure to show the indubitable title of Exequiel to the property in question is vital to
of the remedy of quieting of title the resolution of the present Petition. It was from him that Hilario had allegedly derived
2. WON the REM in the case at bar is valid his title thereto as owner, an allegation which thereby enabled him to mortgage it to the
3. WON the free patent granted to Santos Spouses was valid Rural Bank of Cardona. The occupation and the possession thereof by the petitioners
and their predecessors-in-interest until 1962 was not disputed, and Exequiel’s
RULING: acquisition of the said property by prescription was not alleged. Thus, the deed of
1. YES. conveyance purportedly evidencing the transfer of ownership and possession from the
Article 476 of the Civil Code provides: heirs of Silvino to Exequiel should have been presented as the best proof of that
“Whenever there is cloud on title to real property or any interest therein, by reason of transfer. No such document was presented, however.
any instrument, record, claim, encumbrance or proceeding which is apparently valid or
effective but is in truth and in fact invalid, ineffective, voidable or unenforceable, and Therefore, there is merit to the contention of the petitioners that Hilario mortgaged the
may be prejudicial to said title, an action may be brought to remove such cloud or to disputed property to the Rural Bank of Cardona in his capacity as a mere co-owner
quiet title. thereof. Clearly, the said transaction did not divest them of title to the property at the
time of the institution of the Complaint for quieting of title.
“An action may also be brought to prevent a cloud from being cast upon title to real
property or any interest therein.” Contrary to the disquisition of the Court of Appeals, Hilario effected no clear and evident
repudiation of the co-ownership. It is a fundamental principle that a co-owner
Based on the above definition, an action to quiet title is a common law remedy for cannot acquire by prescription the share of the other co-owners, absent any clear
the removal of any cloud or doubt or uncertainty on the title to real property. It is repudiation of the co-ownership. In order that the title may prescribe in favor of
essential for the plaintiff or complainant to have a legal or an equitable title to or a co-owner, the following requisites must concur: (1) the co-owner has
interest in the real property which is the subject matter of the action. Also, the performed unequivocal acts of repudiation amounting to an ouster of the other
deed, claim, encumbrance or proceeding that is being alleged as a cloud on co-owners; (2) such positive acts of repudiation have been made known to the
plaintiff’s title must be shown to be in fact invalid or inoperative despite its prima other co-owners; and (3) the evidence thereof is clear and convincing.
facie appearance of validity or legal efficacy.
In the present case, Hilario did not have possession of the subject property;
That there is an instrument or a document which, on its face, is valid and efficacious is neither did he exclude the petitioners from the use and the enjoyment thereof, as
clear in the present case. they had indisputably shared in its fruits. Likewise, his act of entering into a
mortgage contract with the bank cannot be construed to be a repudiation of the
Undisputed is the fact that the land had previously been occupied by Leon and later by co-ownership. As absolute owner of his undivided interest in the land, he had the
Silvino Robles, petitioners’ predecessors-in-interest, as evidenced by the different tax right to alienate his share, as he in fact did. Neither should his payment of land
declarations issued in their names. Also undisputed is the fact that the petitioners taxes in his name, as agreed upon by the co-owners, be construed as a
continued occupying and possessing the land from the death of Silvino in 1942 until repudiation of the co-ownership. The assertion that the declaration of ownership
they were allegedly ousted therefrom in 1988. In 1962, the subject property was was tantamount to repudiation was belied by the continued occupation and
declared in the name of Exequiel for taxation purposes. On September 30, 1965, it was possession of the disputed property by the petitioners as owners.
again declared in the same name; on October 28, 1965, in the name of the Rural Bank
of Antipolo; on November 7, 1966, in the name of Hilario and Andrea; and thereafter, 2. NO.
in the name of the Rural Bank of Cardona and, finally, in the name of the Santos In a real estate mortgage contract, it is essential that the mortgagor be the
spouses. absolute owner of the property to be mortgaged; otherwise, the mortgage is void.
In the present case, it is apparent that Hilario Robles was not the absolute owner of the
entire subject property; and that the Rural Bank of Cardona, Inc., in not fully
ascertaining his title thereto, failed to observe due diligence and, as such, was a any reason why the petitioners had continued occupying it, even as he admitted on the
mortgagee in bad faith. stand that he had visited it twice.

First, the bank was utterly remiss in its duty to establish who the true owners and In the light of their open, continuous, exclusive and notorious possession and
possessors of the subject property were. It acted with precipitate haste in approving the occupation of the land, petitioners are “deemed to have acquired, by operation
Robles spouses’ loan application, as well as the real estate mortgage covering the of law, a right to a grant, a government grant, without the necessity of a certificate
disputed parcel of land. Had it been more circumspect and assiduous, it would have of title being issued.” The land was “segregated from the public domain.”
discovered that the said property was in fact being occupied by the petitioners, who Accordingly, the director of lands had no authority to issue a free patent thereto
were tending and cultivating it. in favor of another person. Verily, jurisprudence holds that a free patent covering
private land is null and void.
Second, the bank should not have relied solely on the Deed of Sale purportedly
showing that the ownership of the disputed property had been transferred from Exequiel Worth quoting is the disquisition of the Court in Agne v. Director of Lands, in which it
Ballena to the Robles spouses, or that it had subsequently been declared in the name held that a riparian owner presently in possession had a better right over an
of Hilario. Because it was dealing with unregistered land, and the circumstances abandoned river bed than had a registered owner by virtue of a free patent .
surrounding the transaction between Hilario and his father-in-law Exequiel were Under the provisions of Act 2874 pursuant to which the title of private
suspicious, the bank should have exerted more effort to fully determine the title of the respondents’ predecessor-in-interest was issued, the President of the
Robleses. Rural Bank of Compostela v. Court of Appeals invalidated a real estate Philippines, or his alter ego, the Director of Lands, has no authority to
mortgage after a finding that the bank had not been in good faith. The Court explained: grant a free patent for land that has ceased to be a public land and has
“The rule that persons dealing with registered lands can rely solely on the passed to private ownership and a title so issued is null and void. The
certificate of title does not apply to banks.” In Tomas v. Tomas, the Court held: nullity arises, not from fraud or deceit, but from the fact that the land is
“x x x. Banks, indeed, should exercise more care and prudence in dealing not under the jurisdiction of the Bureau of Lands. The jurisdiction of the
even with registered lands, than private individuals, for their business is one Director of Lands is limited only to public lands and does not cover lands
affected with public interest, keeping in trust money belonging to their publicly owned. The purpose of the Legislature in adopting the former Public
depositors, which they should guard against loss by not committing any act of Land Act, Act No. 2874, was and is to limit its application to lands of the public
negligence which amounts to lack of good faith by which they would be denied domain, and lands held in private ownership are not included therein and are
the protective mantle of land registration statute, Act 496, extended only to not affected in any manner whatsoever thereby. Land held freehold or fee
purchasers for value and in good faith, as well as to mortgagees of the same title, or of private ownership, constitutes no part of the public domain,
character and description. x x x.” and cannot possibly come within the purview of said Act 2874, inasmuch
as the ‘subject’ of such freehold or private land is not embraced in any
Lastly, the Court likewise finds it unusual that, notwithstanding the bank’s insistence manner in the title of the Act and the same is excluded from the
that it had become the owner of the subject property and had paid the land taxes provisions of the text thereof.
thereon, the petitioners continued occupying it and harvesting the fruits therefrom.
We reiterate that private ownership of land is not affected by the issuance
Considering that Hilario can be deemed to have mortgaged the disputed property not of the free patent over the same land because the Public Land Act
as absolute owner but only as a co-owner, he can be adjudged to have disposed to the applies only to lands of the public domain. Only public land may be
Rural Bank of Cardona, Inc., only his undivided share therein. The said bank, being the disposed of by the Director of Lands. Since as early as 1920, the land in
immediate predecessor of the Santos spouses, was a mortgagee in bad faith. Thus, dispute was already under the private ownership of herein petitioners and no
justice and equity mandate the entitlement of the Santos spouses, who merely stepped longer a part of the lands of the public domain, the same could not have been
into the shoes of the bank, only to what legally pertains to the latter—Hilario’s share in the subject matter of a free patent. The patentee and his successors-in-
the disputed property. interest acquired no right or title to said land. Necessarily, Free Patent No.
23263 issued to Herminigildo Agpoon is null and void and the subsequent
3. NO. titles issued pursuant thereto cannot become final and indefeasible. Hence we
The petitioner’s claim is an assertion that the subject property is private land. The ruled in Director of Lands v. Sicsican, et al. that if at the time the free patents
petitioners do not concede, and the records do not show, that it was ever an alienable were issued in 1953 the land covered therein were already private property of
land of the public domain. They allege private ownership thereof, as evidenced by their another and, therefore, not part of the disposable land of the public domain,
testimonies and the tax declarations issued in the names of their predecessors-in- then applicants patentees acquired no right or title to the land.
interest. It must be noted that while their claim was not corroborated by other witnesses,
it was not controverted by the other parties, either. Now, a certificate of title fraudulently secured is null and void ab initio if
the fraud consisted in misrepresenting that the land is part of the public
Carlos Dolores insisted that the Rural Bank of Cardona, Inc., of which he was the domain, although it is not. As earlier stated, the nullity arises, not from
manager, had acquired and possessed the subject property. He did not, however, give the fraud or deceit, but from the fact that the land is not under the
jurisdiction of the Bureau of Lands. Being null and void, the free patent
granted and the subsequent titles produce no legal effect whatsoever. owners—openly, peacefully, publicly, continuously and adversely since 1916. Because
Quod nullum est, nullum producit effectum. they and their predecessors-in-interest have occupied, possessed and cultivated it as
owners for more than thirty years, only one conclusion can be drawn—it has become
A free patent which purports to convey land to which the government private land and is therefore beyond the authority of the director of lands.
did not have any title at the time of its issuance does not vest any title in
the patentee as against the true owner. The Court has previously held that Epilogue
the Land Registration Act and the Cadastral Act do not give anybody who
resorts to the provisions thereof a better title than what he really and lawfully We recognize that both the petitioners and the Santos spouses fell victim to the dubious
has. transaction between Spouses Hilario and Andrea Robles and the Rural Bank of
Cardona, Inc. However, justice and equity mandate that we declare Petitioners Lucio,
The long and continued possession of petitioners under a valid claim of Emerita, Aludia and Emilio Robles to have the requisite title essential to their suit for
title cannot be defeated by the claim of a registered owner whose title is quieting of title. Considering the circumstances peculiar to this complicated problem,
defective from the beginning.” the Court finds this conclusion the logical and just solution.

The Santos spouses argue that petitioners do not have the requisite personality to The claim that petitioners were guilty of laches in not asserting their rights as owners
question the free patent granted them, inasmuch as “it is a well-settled rule that actions of the property should be viewed in the light of the fact that they thought their brother
to nullify free patents should be filed by the OSG at the behest of the Director of Lands.” was paying the requisite taxes for them, and more important, the fact that they
continued cultivating it and harvesting and gaining from its fruits.
Private respondents’ reliance on this doctrine is misplaced. Indeed, the Court held in
Peltan Development, Inc. v. Court of Appeals that only the solicitor general could file From another viewpoint, it can even be said that it was the Rural Bank of Cardona, Inc.,
an action for the cancellation of a free patent. Ruling that the private respondents, who which was guilty of laches because, granting that it had acquired the subject property
were applicants for a free patent, were not the proper parties in an action to cancel the legally, it failed to enforce its rights as owner. It was oblivious to the petitioners’
transfer certificates covering the parcel of land that was the subject of their application, continued occupation, cultivation and possession thereof. Considering that they had
the Court ratiocinated thus: possessed the property in good faith for more than ten years, it can even be argued
“The Court also holds that private respondents are not the proper parties to that they thus regained it by acquisitive prescription. In any case, laches is a remedy in
initiate the present suit. The complaint, praying as it did for the cancellation of equity, and considering the circumstances in this case, the petitioners cannot be held
the transfer certificates of title of petitioners on the ground that they were guilty of it.
derived from a “spurious” OCT No. 4216, assailed in effect the validity of said
title. While private respondents did not pray for the reversion of the land to the In sum, the real estate mortgage contract covering the disputed property—a contract
government, we agree with the petitioners that the prayer in the complaint will executed between Spouses Hilario and Andrea on the one hand and the Rural Bank of
have the same result of reverting the land to the government under the Cardona, Inc., on the other— is hereby declared null and void insofar as it prejudiced
Regalian Doctrine. Gabila v. Barriga ruled that only the government is entitled the shares of Petitioners Lucio, Emerita, Aludia and Emilio Robles; it is valid as to Hilario
to this relief.” Robles’ share therein. Consequently, the sale of the subject property to the Santos
spouses is valid insofar as it pertained to his share only. Likewise declared null and
Because the cancellation of the free patent as prayed for by the private respondents in void is Free Patent No. IV-1-010021 issued by the Bureau of Lands covering the subject
Peltan would revert the property in question to the public domain, the ultimate property.
beneficiary would be the government, which can be represented by the solicitor general
only. Therefore, the real party-in-interest is the government, not the private WHEREFORE, the Petition is hereby GRANTED. The assailed Decision is REVERSED
respondents. and SET ASIDE. Except as modified by the last paragraph of this Decision, the trial
court’s Decision is REINSTATED. No costs.
This ruling does not, however, apply to the present case. While the private respondents
in Peltan recognized that the disputed property was part of the public domain when SO ORDERED.
they applied for free patent, herein petitioners asserted and proved private ownership
over the disputed parcel of land by virtue of their open, continued and exclusive
possession thereof since 1916.

Neither does the present case call for the reversion of the disputed property to the
State. By asking for the nullification of the free patent granted to the Santos spouses,
the petitioners are claiming the property which, they contend, rightfully belongs to them.
The foregoing considered, we sustain the contention of petitioners that the free patent
granted to the Santos spouses is void. It is apparent that they are claiming ownership
of the disputed property on the basis of their possession thereof in the concept of
ANASTACIA VDA. DE AVILES, ET AL., petitioners, vs. COURT OF APPEALS and On appeal, the CA affirmed in part the decision of the trial court, reasoning that a special
CAMILO AVILES, respondents. civil action for quieting of title is not the proper remedy for settling a boundary dispute,
G.R. No. 95748. November 21, 1996.* and that petitioners should have instituted an ejectment suit instead.
PANGANIBAN, J.:
Petitioners deem to be “without basis” the respondent Court’s holding that quieting of
FACTS: PLAINTIFFS aver that they are the actual possessors of a parcel of land title is not the proper remedy in the case a quo. They assert that private respondent is
situated in Malawa, Lingayen, Pangasinan, more particularly described as fishpond, occupying the disputed lot because he claimed it to be part of his share in the partitioned
cogonal, unirrigated rice and residential land, bounded on the N by Camilo Aviles; on property of his parents, whereas petitioners are claiming the said lot as part and parcel
the E by Malawa River, on the S by Anastacio Aviles and on the W by Juana and of the land allotted to Eduardo Aviles, petitioners’ predecessor-in-interest. They
Apolonio Joaquin, with an area of 18,900 square meters (land is declared for taxation contend that they have been occupying the aforesaid land as heirs of Eduardo Aviles
purposes). This property is the share of their father, Eduardo Aviles and brother of the in “open, actual, continuous, peaceful, public and adversed (sic) (possession) against
defendant, in the estate of their deceased parents, Ireneo Aviles and Anastacia the whole world.” Further, they argue that, if indeed the disputed lot belonged to private
Salazar. respondent, why then did it take him “almost 26 long years from June 27, 1957 or until
March 27, 1983” to assert his ownership; why did he not “assert his ownership” over
SINCE 1957, Eduardo Aviles was in actual possession of the aforedescribed property. the property when Eduardo Aviles was still alive; and why did he not take any “action”
In fact, the latter mortgaged the same with the Rural Bank and Philippine National Bank when the mortgage over the disputed property was foreclosed?
branch in Lingayen. When the property was inspected by a bank representative,
Eduardo Aviles, in the presence of the boundary owners, namely, defendant Camilo Private respondent corrects the petitioners’ claim in regard to the date when he had the
Aviles, Anastacio Aviles and Juana and Apolonio Joaquin(,) pointed to the inspector bamboo fence constructed. He alleges that the petitioners maliciously concocted the
the existing earthen dikes as the boundary limits of the property and nobody objected. story that private respondent had purportedly encroached some 1,200 meters on their
When the real estate mortgage was foreclosed, the property was sold at public auction property when, in fact, “he was merely repairing the old bamboo fence existing where
but this was redeemed by plaintiffs’ mother and the land was subsequently transferred it had always been since 1957.”
and declared in her name.
ISSUE: WON petitioners availed of the proper remedy of quieting of title in the case at
ON March 23, 1983, defendant Camilo Aviles asserted a color of title over the northern bar in settling the boundary dispute
portion of the property with an area of approximately 1,200 square meters by
constructing a bamboo fence (thereon) and moving the earthen dikes, thereby RULING: NO.
molesting and disturbing the peaceful possession of the plaintiffs over said portion.
UPON the other hand, defendant Camilo Aviles admitted the agreement of partition RATIONALE: Quieting of title is a common law remedy for the removal of any
executed by him and his brothers, Anastacio and Eduardo. In accordance therewith, cloud upon or doubt or uncertainty with respect to title to real property.
the total area of the property of their parents which they divided is 46,795 square meters
and the area alloted (sic) to Eduardo Aviles is 16,111 square meters more or less, to The Civil Code authorizes the said remedy in the following language:
Anastacio Aviles is 16,214 square meters more or less, while the area alloted to “Art. 476. Whenever there is a cloud on title to real property or any interest
defendant Camilo Aviles is 14,470 square meters more or less. The respective area(s) therein, by reason of any instrument, record, claim, encumbrance or
alloted to them was agreed and measured before the execution of the agreement but proceeding which is apparently valid or effective but is, in truth and in fact,
he was not present when the measurement was made. invalid, ineffective, voidable, or unenforceable, and may be prejudicial to said
title, an action may be brought to remove such cloud or to quiet the title.
Defendant agreed to have a smaller area because his brother Eduardo asked him that
he wanted a bigger share because he has several children to support. The portion in An action may also be brought to prevent a cloud from being cast upon a title
litigation however is part of the share given to him in the agreement of partition. At to real property or any interest therein.”
present, he is only occupying an area of 12,686 square meters which is smaller than
his actual share of 14,470 square meters. The riceland portion of his land is 13,290 In fine, to avail of the remedy of quieting of title, a plaintiff must show that there
square meters, the fishpond portion is 500 square meters and the residential portion is is an instrument, record, claim, encumbrance or proceeding which constitutes
680 square meters, or a total of 14,470 square meters. That the topography of his land or casts a cloud, doubt, question or shadow upon the owner’s title to or interest
is not the same, hence, the height of his pilapils are likewise not the same.” in real property. Thus, petitioners have wholly misapprehended the import of the
foregoing rule by claiming that respondent Court erred in holding that there was “no x
In its decision, the trial court ordered the parties to employ the services of a Land x x evidence of any muniment of title, proceeding, written contract, x x x,” and that there
Surveyor of the Bureau of Lands, Region I, San Fernando, La Union, to relocate and were, as a matter of fact, two such contracts, viz., (i) the Agreement of Partition
determine the extent and the boundary limit of the land of the defendant on its southern executed by private respondent and his brothers (including the petitioners’ father and
side in order that the 14,470 square meters which is the actual area given to the predecessor-in-interest), in which their respective shares in the inherited property were
defendant be determined. agreed upon, and (ii) the Deed of Sale evidencing the redemption by petitioner
Anastacia Vda. de Aviles of the subject property in a foreclosure sale. However, these
documents in no way constitute a cloud or cast a doubt upon the title of petitioners.
Rather, the uncertainty arises from the parties’ failure to situate and fix the boundary
between their respective properties.

As correctly held by the respondent Court, “(i)n fact, both plaintiffs and defendant
admitted the existence of the agreement of partition dated June 8, 1957 and in
accordance therewith, a fixed area was alloted (sic) to them and that the only
controversy is whether these lands were properly measured. There is no adverse claim
by the defendant “which is apparently valid, but is, in truth and in fact, invalid,
ineffective, voidable, or unenforceable” and which constitutes a cloud thereon.

Corollarily, and equally as clear, the construction of the bamboo fence enclosing
the disputed property and the moving of earthen dikes are not the “clouds” or
“doubts” which can be removed in an action for quieting of title.

An action to quiet title or to remove cloud may not be brought for the purpose of
settling a boundary dispute.

Even assuming arguendo that the action to quiet title had been brought under Rule 64
(of the Rules of Court), the same would still not have prospered, the subject matter
thereof not referring to “a deed, will, contract or other written instrument, or to a statute
or ordinance,” but to a boundary dispute, and therefore not warranting the grant of
declaratory relief.
From another perspective, we hold that the trial court (and likewise the respondent
Court) cannot, in an action for quieting of title, order the determination of the boundaries
of the claimed property, as that would be tantamount to awarding to one or some of the
parties the disputed property in an action where the sole issue is limited to whether the
instrument, record, claim, encumbrance or proceeding involved constitutes a cloud
upon the petitioners’ interest
or title in and to said property. Such determination of boundaries is appropriate in
adversarial proceedings where possession or ownership may properly be considered
and where evidence aliunde, other than the “instrument, record, claim, encumbrance
or proceeding” itself, may be introduced. An action for forcible entry, whenever
warranted by the period prescribed in Rule 70, or for recovery of possession de facto,
also within the prescribed period, may be availed of by the petitioners, in which
proceeding the boundary dispute may be fully threshed out.

WHEREFORE, in view of the foregoing considerations, the instant petition is hereby


DENIED and the Decision appealed from is AFFIRMED. Costs against petitioners.

SO ORDERED.
JENESTOR B. CALDITO and MARIA FILOMENA T. CALDITO, petitioners, vs. over the property of Felipe based on the governing law and on the order of intestate
ISAGANI V. OBADO and GEREON V. OBADO, respondents. succession at that time and the established facts.
G.R. No. 181596. January 30, 2017.*
REYES, J.: On appeal, the CA reversed and set aside the RTC decision upon finding that: (1) the
petitioners failed to prove the title of their immediate predecessors-in-interest, the
FACTS: This petition stemmed from a complaint for quieting of ownership over a parcel Spouses Ballesteros; (2) the petitioners failed to support their claim that Felipe and his
of land covering the 272.33 square meters eastern portion of Lot No. 1633 situated at siblings, Eladia, Estanislao, Maria, Severino and Tomasa, co-owned Lot No. 1633; (3)
Barangay No. 5, San Vicente, Sarrat, Ilocos Norte, filed by Spouses Jenestor B. Caldito Antonio should have been called to the witness stand to testify on the contents of his
and Ma. Filomena Tejada Caldito (petitioners) against Isagani V. Obado and Gereon Affidavit of Ownership; (4) the Deed of Absolute Sale is not a sufficient and convincing
V. Obado (respondents). evidence that the petitioners’ predecessors-in-interest have a title on the subject parcel
of land which they can transfer; (5) the petitioners are not innocent purchasers for value
The record showed that as early as 1921, Lot No. 1633 was declared for taxation since the subject lot is not registered and is in the possession of another person, other
purposes in the name of Felipe Obado. After Felipe’s death, Paterno Obado, whom than the Spouses Ballesteros; (6) nothing in the record could establish the relationship
Felipe treated like his own son, subsequently occupied Lot No. 1633 and continued to between Felipe and his supposed legal heirs; and (7) the respondents enjoy a legal
pay the realty taxes of the same. presumption of just title in their favor since they are in possession of the entire Lot No.
1633. MR of petitioners was denied. Hence, this present petition.
Sometime in 1995, Antonio Ballesteros executed an Affidavit of Ownership dated
February 23, 1995 narrating his claim over the subject parcel of land. In his affidavit, ISSUE: WON petitioners were able to prove ownership over the subject parcel of land
Antonio claimed that Lot No. 1633 was co-owned by Felipe with his five siblings,
namely: Eladia, Estanislao, Maria, Severino and Tomasa, all surnamed Obado. RULING: NO.

On the next day following the execution of the said affidavit or on February 24, 1995, RATIONALE: At the outset, it bears to emphasize that there is no dispute with respect
Antonio and Elena Ballesteros (Spouses Ballesteros) sold the subject parcel of land to to the fact that Felipe was the original owner of the entire parcel of unregistered land
the petitioners for the sum of P70,000.000 evidenced by a Deed of Absolute Sale. known as Lot No. 1633 which he started declaring as his property for taxation purposes
Thereafter, the petitioners declared the subject lot for taxation purposes and paid the as early as 1921. When Felipe died without issue, Lot No. 1633 was subsequently
realty taxes thereon. occupied by Paterno who then declared the same for taxation purposes and paid the
realty taxes thereon.
In 2002, the petitioners attempted to build a house on the subject parcel of land but the
respondents prevented them from completing the same. The respondents then filed a The petitioners’ complaint styled as being for the “quieting of ownership” is in fact an
complaint before the barangay but no amicable settlement was reached between the action for quieting of title. The petitioners anchor their cause of action upon the Deed
parties. Hence, on December 8, 2003, the petitioners instituted a complaint for quieting of Sale and the Affidavit of Ownership executed by Antonio. On the other hand, the
of ownership against the respondents before the RTC, as well as an injunctive writ to respondents countered that: (1) they inherited from their father, Paterno, Lot No. 1633,
prevent the respondents from interfering with the construction of their house. of which the herein disputed subject parcel of land is part; and (2) they have been in
possession of the same for more than 30 years in the concept of an owner.
For their part, the respondents averred that the Spouses Ballesteros were not the
owners and possessors of the subject parcel of land. They maintained that Lot No. In resolving the issue, the pertinent point of inquiry is whether the petitioners’
1633 was inherited by their father, Paterno, from its original owner Felipe, and they predecessors-in-interest, the Spouses Ballesteros, have lawful title over the subject
have been paying the real property taxes for the entire property. They asserted that the parcel of land.
petitioners are buyers in bad faith since their family had been in possession of the entire
Lot No. 1633 since 1969 and had been in open, peaceful and uninterrupted possession Even if the Court were to reevaluate the evidence presented, there is still no reason to
of the whole property up to the present or for more than 30 years in the concept of an depart from the CA’s ruling that Lot No. 1633 is owned by the respondents. The Court
owner. concurs with the disquisition of the CA that the petitioners failed to: (1) prove the title
of their immediate predecessors-in-interest, the Spouses Ballesteros; and (2) present
After trial, the court a quo rendered its judgment in favor of the petitioners. The trial evidence supporting the claim that Lot No. 1633 was co-owned by Felipe and his
court upheld the validity of the sale between the petitioners and the Spouses siblings, Eladia, Estanislao, Maria, Severino and Tomasa. Also, the Court finds that the
Ballesteros and dismissed the respondents’ claim of ownership over Lot No. 1633. The RTC mistakenly relied upon the Affidavit of Ownership, executed by Antonio, to
trial court held that the petitioners presented convincing evidence of ownership over conclude that the petitioners were possessors in good faith and with just title who
the subject parcel of land which consists of the following: (a) the Deed of Absolute Sale acquired the subject parcel of land through a valid deed of sale.
executed between the petitioners and the Spouses Ballesteros; (b) the tax declarations
all paid by the petitioners only; and (c) the Affidavit of Ownership allegedly executed by In this case, the petitioners’ cause of action relates to an action to quiet title which
Antonio. The trial court also found that the respondents have no successional rights has two indispensable requisites, namely: (1) the plaintiff or complainant has a legal
or an equitable title to or interest in the real property subject of the action; and (2) the
deed, claim, encumbrance or proceeding claimed to be casting cloud on his title must the real property taxes of the subject parcel of land. Besides, it seems that the
be shown to be in fact invalid or inoperative despite its prima facie appearance of petitioners knew of the fact that they did not have a title to the subject parcel of land
validity or legal efficacy. and could not, therefore, have validly registered the same, because of the respondents’
possession of the entire property.
From the foregoing provisions, it is clear that the petitioners’ cause of action must
necessarily fail mainly in view of the absence of the first requisite since the petitioners The respondents also presented the following pieces of evidence: (1) old certified
were not able to prove equitable title or ownership over the subject parcel of land. photocopies of declarations of real property and original copy of tax receipts from year
1921 to 1944 in the name of Felipe, covering payments by the latter for Lot No. 1633
The petitioners’ claim of legal title over the subject parcel of land by virtue of the Deed from which the subject parcel of land was taken; (2) original copy of tax receipts from
of Sale and Affidavit of Ownership issued by Antonio cannot stand because they failed year 1961 to year 1989 in the name of the respondents’ father Paterno, covering
to prove the title of their immediate predecessors-in-interest, the Spouses Ballesteros. payments by the latter for Lot No. 1633;
The Court cannot give full credence to Antonio’s Affidavit of Ownership for he simply (3) original copy of tax receipt dated July 21, 2004 in the name of Isagani, covering
made general and self-serving statements therein which were favorable to him, and payments by the latter for Lot No. 1633; (4) original copy of the Certification issued by
which were not supported with documentary evidence, with no specifics as to when the Municipal Treasurer of Sarrat, Ilocos Norte that Lot No. 1633 covered by Tax
their predecessors-in-interest acquired the subject parcel of land, and when the Declaration No. 03-001-00271 declared in the name of Felipe is not delinquent in the
Donations Propter Nuptias were made. Indeed, such is hardly the well-nigh payment of realty taxes.
incontrovertible evidence required in cases of this nature. The petitioners must present
proof of specific acts of ownership to substantiate his claim and cannot just offer general Although tax declarations or realty tax payment of property are not conclusive
statements which are mere conclusions of law than factual evidence of possession. evidence of ownership, as in the instant case, they are good indicia of
More so, Antonio was not even called to the witness stand to testify on the contents of possession in the concept of owner, for no one in his right mind would be paying
his Affidavit of Ownership, thus, making the affidavit hearsay evidence and its probative taxes for a property that is not in his actual or constructive possession. They
value questionable. Accordingly, this affidavit must be excluded from the judicial constitute evidence of great weight in support of the claim of title of ownership
proceedings being inadmissible hearsay evidence. by prescription when considered with the actual possession of the property by
the applicant.
Furthermore, the said affidavit was executed by Antonio only a day before the subject
parcel of land was sold to the petitioners. The trial court should have considered this in Indeed, the respondents’ presentation of the tax declarations and tax receipts
evaluating the value of the said affidavit in relation to the ownership of the subject parcel which all are of ancient era indicates possession in the concept of an owner by
of land. The trial court’s reliance on the Affidavit of Ownership executed by Antonio that the respondents and their predecessors-in interests. The tax declarations in the
the entire Lot No. 1633 was co-owned by Felipe and his siblings, Eladia, Estanislao, name of Paterno take on great significance because the respondents can tack their
Maria, Severino and Tomasa is mis placed, considering that nothing on record shows claim of ownership to that of their father. It is worthy to note that the respondents’ father
the relationship between Felipe and his supposed legal heirs. It also indicates the fact Paterno to whom they inherited the entire Lot No. 1633 paid the taxes due under his
that there is no evidence showing Felipe predeceasing all his supposed siblings. name from 1961 to 1989; and subsequently, the respondents paid the taxes due after
Moreover, no other piece of evidence was ever presented to prove that Lot No. 1633 the death of Paterno in 2003. Granting without admitting that Felipe’s possession of Lot
was ever subdivided. In fact, the petitioners admitted that the subject lot has always No. 1633 cannot be tacked with the respondents’ possession, the latter’s possession
been declared for taxation purposes in the name of Felipe and that the Spouses can be tacked with that of Paterno. Thus, from 1961 to the time of the filing of the
Ballesteros or the siblings of Felipe have never declared the same for taxation purposes quieting of title by the petitioners in 2003, the respondents have been in
in their names. possession of the entire Lot No. 1633 in the concept of an owner for almost 42
years. This period of time is sufficient to vest extraordinary acquisitive
While the petitioners submitted official receipts and tax declarations to prove payment prescription over the property on the respondents. As such, it is immaterial now
of taxes, nowhere in the evidence was it shown that Spouses Ballesteros declared the whether the respondents possessed the property in good faith or not.
subject parcel of land in their name for taxation purposes or paid taxes due thereon.
True, a tax declaration by itself is not sufficient to prove ownership. Nonetheless, Admittedly, the respondents built their house at the western portion of Lot No. 1633,
it may serve as sufficient basis for inferring possession. In fact, what the and Isagani has declared that the eastern part was their family’s garden. Thus, it was
petitioners presented as their pieces of evidence are receipts and tax declarations fenced with bamboo and was planted with banana trees and different vegetables.
which they, as the new owners of the subject parcel of land, have paid. Thus, the Clearly, there is no doubt that the respondents did not only pay the taxes due for the
petitioners could not also rely on these tax declarations and receipts because those are whole Lot No. 1633, in which the eastern portion is a part, but rather, the respondents
of recent vintage and do not reflect the fact that their predecessors-in-interest have were able to prove that they have possession of the whole lot.
been paying realty taxes for the subject parcel of land.
While the findings of the CA that the petitioners were a buyer in bad faith is in accord
Be that as it may, the rights of the respondents as owners of Lot No. 1633 were never with the evidence on record, it must be pointed out, however, that they overlooked the
alienated from them despite the sale of the subject parcel of land by the Spouses fact that Lot No. 1633 is an unregistered piece of land. The Court had already ruled that
Ballesteros to the petitioners nor does the fact that the petitioners succeeded in paying the issue of good faith or bad faith of a buyer is relevant only where the subject
of the sale is a registered land but not where the property is an unregistered land.
One who purchases an unregistered land does so at his peril. His claim of having
bought the land in good faith, i.e., without notice that some other person has a right to,
or interest in, the property, would not protect him if it turns out that the seller does not
actually own the property. All the same, the application of this doctrine will not affect
the outcome of this case.

Obviously, the petitioners cannot benefit from the deed of sale of the subject parcel of
land, executed by the Spouses Ballesteros in their favor, to support their claim of
possession in good faith and with just title. The Court noted that in Filomena’s
testimony, she even admitted that the respondents own the bigger portion of Lot No.
1633. Thus, it is clear that the petitioners chose to close their eyes to facts which should
have put a reasonable man on his guard. The petitioners failed to ascertain whether
the Spouses Ballesteros were the lawful owner of the subject parcel of land being sold.
Far from being prudent, the petitioners placed full faith on the Affidavit of Ownership
that Antonio executed. Hence, when the subject parcel of land was bought by the
petitioners, they merely stepped into the shoes of the Spouses Ballesteros and
acquired whatever rights and obligations appertain thereto.

It is also worthy to note of the respondents’ reaction when the petitioners tried to
construct a house in the subject parcel of land in 2002. Upon learning that a house was
being built on the eastern portion of Lot No. 1633, the respondents went to the barangay
to file a complaint. Clearly, this indicates the respondents’ vigilance to protect their
property. The Court also notes that in the respondent’s possession of the entire Lot No.
1633 for almost 42 years, there was no instance during this time that the petitioners or
their predecessors-in-interest, for that matter, questioned the respondents’ right over
Lot No. 1633.

WHEREFORE, the petition is DENIED. The Decision dated July 17, 2007 and the
Resolution dated January 29, 2008 of the Court of Appeals in C.A.-G.R. CV No. 87021
are AFFIRMED.

SO ORDERED.
REY CASTIGADOR CATEDRILLA, petitioner, vs. MARIO and MARGIE1 LAURON, even before 1992 and only upon the acquiescence of the petitioner and his
respondents. predecessor-in-interest.
G.R. No. 179011. April 15, 2013.*
PERALTA, J.: The MTC found that respondents would like to focus their defense on the ground that
Mildred is an indispensable party, because she is the owner of the residential building
FACTS: On February 12, 2003, petitioner Rey Castigador Catedrilla filed with the MTC- on the subject lot and that there was already a perfected contract to sell between
Lambunao, Iloilo a Complaint for ejectment against the spouses Mario and Margie Mildred and Maximo because of an amicable settlement executed before the Office of
Lauron alleging as follows: that Lorenza Lizada is the owner of a parcel of land known the Punong Barangay. However, the MTC, without dealing on the validity of the
as Lot 183, located in Mabini Street, Lambunao, Iloilo, which was declared for taxation document and its interpretation, ruled that it was clear that respondent Margie was
purposes in her name; that on February 13, 1972, Lorenza died and was succeeded to representing her parents, Mr. and Mrs. Bienvenido Loraña, in the dispute presented
her properties by her sole heir Jesusa Lizada Losañes, who was married to Hilarion with the Punong Barangay. It also found that even Mildred’s letter to petitioner’s father
Castigador; that the spouses Jesusa and Hilarion Castigador had a number of children, Maximo recognized the title of petitioner’s father over the subject lot and that it had not
which included Lilia Castigador, who was married to Maximo Catedrilla; that after the been established by respondents if Teresito Castigador, the person who signed the
death of the spouses Castigador, their heirs agreed among themselves to subdivide receipt evidencing Mildred’s downpayment of P10,000.00 for the subject lot, is also one
Lot 183 and, pursuant to a consolidation subdivision plan dated January 21, 1984, the of the heirs of Lilia. The MTC concluded that respondents could not be allowed to
parcel of lot denominated as Lot No. 5 therein was to be apportioned to the heirs of Lilia deflect the consequences of their continued stay over the property, because it was their
since the latter already died on April 9, 1976; Lilia was succeeded by her heirs, her very occupation of the property which is the object of petitioner’s complaint; that in an
husband Maximo and their children, one of whom is herein petitioner; that petitioner action for ejectment, the subject matter is material possession or possession de facto
filed the complaint as a co-owner of Lot No. 5; that sometime in 1980, respondents over the real property, and the side issue of ownership over the subject lot is tackled
Mario and Margie Lauron, through the tolerance of the heirs of Lilia, constructed a here only for the purpose of determining who has the better right of possession which
residential building of strong materials on the northwest portion of Lot No. 5 covering is to prove the nature of possession; that possession of Lot 183 should be relinquished
an area of one hundred square meters; that the heirs of Lilia made various demands by respondents to petitioner, who is a co-owner, without foreclosing other remedies that
for respondents to vacate the premises and even exerted earnest efforts to compromise may be availed upon by Mildred in the furtherance of her supposed rights.
with them but the same was unavailing; and that petitioner reiterated the demand on
respondents to vacate the subject lot on January 15, 2003, but respondents continued RTC, on appeal, appeal MTC. It found that petitioner, being one of the co-owners of
to unlawfully withhold such possession. the subject lot, is the proper party in interest to prosecute against any intruder thereon.
It found that the amicable settlement signed and executed by the representatives of the
In their Answer, respondents claimed that petitioner had no cause of action against registered owner of the premises before the Lupon is not binding and unenforceable
them, since they are not the owners of the residential building standing on petitioner’s between the parties. It further ruled that even if Mildred has her name in the tax
lot, but Mildred Kascher, sister of respondent Margie, as shown by the tax declaration declaration signifying that she is the owner of the house constructed on the subject lot,
in Mildred’s name; that in 1992, Mildred had already paid P10,000.00 as downpayment tax declarations are not evidence of ownership but merely issued to the declarant for
for the subject lot to Teresito Castigador; that there were several instances that the purposes of payment of taxes; that she cannot be considered as an indispensable party
heirs of Lilia offered the subject Lot 183 for sale to respondents and Mildred and in a suit for recovery of possession against respondents; that Mildred should have
demanded payment, however, the latter was only interested in asking money without intervened and proved that she is an indispensable party because the records showed
any intention of delivering or registering the subject lot; that in 1998, Maximo, that she was not in actual possession of the subject lot. MR was denied.
petitioner’s father, and respondent Margie entered into an amicable settlement before
the Barangay Lupon of Poblacion Ilawod, Lambunao, Iloilo wherein Maximo offered On a petition for review before the CA, CA reversed and set aside the RTC decision. It
the subject lot to the spouses Alfons and Mildred Kascher in the amount of P90,000.00 found that only petitioner filed the case for ejectment against respondents and ruled
with the agreement that all documents related to the transfer of the subject lot to that the other heirs should have been impleaded as plaintiffs citing Section 1, Rule 7
Maximo and his children be prepared by Maximo, but the latter failed to comply; and and Section 7, Rule 3 of the Rules of Court; that the presence of all indispensable
that the amicable settlement should have the force and effect of a final judgment of a parties is a condition sine qua non for the exercise of judicial power; that when an
court, hence, the instant suit is barred by prior judgment. Respondents counterclaimed indispensable party is not before the court, the action should be dismissed as without
for damages. the presence of all the other heirs as plaintiffs, the trial court could not validly render
judgment and grant relief in favor of the respondents.
MTC ruled in favor of plaintiff. The MTC found that from the allegations and evidence
presented, it appeared that petitioner is one of the heirs of Lilia Castigador Catedrilla, The CA also ruled that while petitioner asserted that the proper parties to be sued are
the owner of the subject lot and that respondents are occupying the subject lot; that the respondents as they are the actual possessors of the subject lot and not Mildred,
petitioner is a party who may bring the suit in accordance with Article 487 of the Civil petitioner still cannot disclaim knowledge that it was to Mildred to whom his co-owners
Code; and as a co-owner, petitioner is allowed to bring this action for ejectment under offered the property for sale, thus, he knew all along that the real owner of the house
Section 1, Rule 70 of the Rules of Court; that respondents are also the proper party to on the subject lot is Mildred and not respondents; that Mildred even paid P10,000.00
be sued as they are the occupants of the subject lot which they do not own; and that out of the total consideration for the subject lot and required respondents’ relatives to
the MTC assumed that the house standing on the subject lot has been standing thereon secure the documents that proved their ownership over the subject lot; that Maximo
and Mildred had previously settled the matter regarding the sale of the subject lot before which would redound to the benefit of the co-owners. Since petitioner recognized the
the Barangay as contained in an amicable settlement signed by Maximo and existence of a co-ownership, he, as a co-owner, can bring the action without the
respondent Margie. Thus, the question in this case extends to mere possessory rights necessity of joining all the other co-owners as co-plaintiffs.
and non-inclusion of indispensable parties made the complaint fatally defective. From
the facts obtaining in this case, ejectment being a summary remedy is not the Petitioner contends that the CA committed a reversible error in finding that Mildred
appropriate action to Kascher is an indispensable party and that her non-inclusion as a party defendant in
file against the alleged deforciant of the property. the ejectment case made the complaint fatally defective, thus, must be dismissed.

The CA found that petitioner’s co-heirs to the subject lot should have been impleaded We agree with petitioner.
as co-plaintiffs in the ejectment case against respondents, since without their presence,
the trial court could not validly render judgment and grant relief in favor of petitioner. The CA based its findings that Mildred is an indispensable party because it found that
petitioner knew all along that Mildred is the owner of the house constructed on the
ISSUE: WON petitioner’s co-heirs to the subject lot should have been impleaded as subject lot as shown in the affidavits of Maximo and petitioner stating that petitioner’s
co-plaintiffs in the ajectment case against respondents co-owners had offered for sale the subject lot to Mildred, and that Maximo, petitioner’s
father, and Mildred had previously settled before the Barangay the matter regarding the
RULING: NO. sale of the subject lot to the latter as contained in the amicable settlement.

RATIONALE: Petitioner can file the action for ejectment without impleading his co- We find that the affidavits of Maximo and petitioner merely stated that the lot was
owners. In Wee v. De Castro, wherein petitioner therein argued that the respondent offered for sale to Mildred, but nowhere did it admit that Mildred is the owner of the
cannot maintain an action for ejectment against him, without joining all his co-owners, house constructed on the subject lot.
we ruled in this wise:
ART. 487. Any one of the co-owners may bring an action in ejectment. Also, it appears that the amicable settlement before the Barangay wherein it was stated
that Maximo will sell the subject lot to the spouses Alfons and Mildred Kascher was
This article covers all kinds of action for the recovery of possession, i.e., signed by Maximo on behalf of his children and respondent Margie on behalf of Mr. and
forcible entry and unlawful detainer (accion interdictal), recovery of Mrs. Bienvenido Loraña. Thus, there is no basis for the CA’s conclusion that it was
possession (accion publiciana), and recovery of ownership (accion de Mildred and Maximo who had previously settled the sale of the subject lot. Moreover, it
reivindicacion). As explained by the renowned civilest, Professor Arturo M. appears however, that while there was a settlement, Liah C. Catedrilla, one of
Tolentino: petitioner’s co-heirs, wrote a letter dated October 30, 2002, to the Spouses Loraña and
A co-owner may bring such an action, without the necessity of joining respondent Margie stating that the latter had made a change on the purchase price for
all the other co-owners as co-plaintiffs, because the suit is deemed to be the subject lot which was different from that agreed upon in the amicable settlement.
instituted for the benefit of all. If the action is for the benefit of the Records neither show that respondent Margie had taken steps to meet with Liah or any
plaintiff alone, such that he claims possession for himself and not for of her co-heirs to settle the matter of the purchase price nor rebut such allegation in the
the co-ownership, the action will not prosper. letter if it was not true. The letter dated July 5, 2003 of respondent Margie’s counsel
addressed to petitioner’s counsel, stating that his client is amenable in the amount as
In the more recent case of Carandang v. Heirs of De Guzman, this Court declared that proposed in the amicable settlement, would not alter the fact of respondents’ non-
a co-owner is not even a necessary party to an action for ejectment, for complete compliance with the settlement since the letter was sent after the ejectment case had
relief can be afforded even in his absence, thus: already been filed by petitioner.
In sum, in suits to recover properties, all co-owners are real parties in interest.
However, pursuant to Article 487 of the Civil Code and the relevant While the amicable settlement executed between Maximo and respondent Margie
jurisprudence, any one of them may bring an action, any kind of action for the before the Barangay had the force and effect of a final judgment of a court, it appears
recovery of co-owned properties. Therefore, only one of the co-owners, that there was non-compliance thereto by respondent Margie on behalf of her parents
namely the co-owner who filed the suit for the recovery of the co-owned which may be construed as repudiation. The settlement is considered rescinded in
property, is an indispensable party thereto. The other co-owners are not accordance with the provision of Article 2041 of the Civil Code. Since the settlement
indispensable parties. They are not even necessary parties, for a was rescinded, petitioner, as a co-owner, properly instituted the action for ejectment to
complete relief can be afforded in the suit even without their recover possession of the subject lot against respondents who are in possession of the
participation, since the suit is presumed to have been filed for the benefit same.
of all co-owners.
Even the receipt signed by a certain Teresito Castigador, acknowledging having
In this case, although petitioner alone filed the complaint for unlawful detainer, he stated received from Mildred the amount of P10,000.00 as downpayment for the purchase of
in the complaint that he is one of the heirs of the late Lilia Castigador, his mother, who the subject lot, would not also prove respondents’ allegation that there was already a
inherited the subject lot, from her parents. Petitioner did not claim exclusive ownership perfected contract to sell the subject lot to Mildred, since the authority of Teresito to sell
of the subject lot, but he filed the complaint for the purpose of recovering its possession on behalf of the heirs of Lilia Castigador was not established.
In ejectment cases, the only issue to be resolved is who is entitled to the physical
or material possession of the property involved, independent of any claim of
ownership set forth by any of the party-litigants. In an action for unlawful
detainer, the real party-in-interest as party-defendant is the person who is in
possession of the property without the benefit of any contract of lease and only
upon the tolerance and generosity of its owner. Well settled is the rule that a
person who occupies the land of another at the latter’s tolerance or permission,
without any contract between them, is bound by an implied promise that he will
vacate the same upon demand, failing which a summary action for ejectment is
the proper remedy against him. His status is analogous to that of a lessee or
tenant whose term of lease has expired but whose occupancy continued by
tolerance of the owner.

Here, records show that the subject lot is owned by petitioner’s mother, and petitioner,
being an heir and a co-owner, is entitled to the possession of the subject lot. On the
other hand, respondent spouses are the occupants of the subject lot which they do not
own. Respondents’ possession of the subject lot was without any contract of lease as
they failed to present any, thus lending credence to petitioner’s claim that their stay in
the subject lot is by mere tolerance of petitioner and his predecessors. It is indeed
respondents spouses who are the real parties-in-interest who were correctly impleaded
as defendants in the unlawful detainer case filed by petitioner.

WHEREFORE, premises considered, the petition is hereby GRANTED. The Decision


dated February 28, 2007 and the Resolution dated July 11, 2007 of the Court of
Appeals are hereby REVERSED and SET ASIDE. The Order dated March 22, 2005 of
the Regional Trial Court, Branch 26, Iloilo City, in Civil Case No. 04-27978, is hereby
REINSTATED.

SO ORDERED.
SPOUSES MANUEL and SALVACION DEL CAMPO, petitioners, vs. HON. COURT of these small lots is Lot No. 162-C-6 with an area of 11,732 square meters which was
OF APPEALS and HEIRS OF JOSE REGALADO, SR., respondents. registered on February 24, 1977 under TCT No. 14566.
G.R. No. 108228. February 1, 2001.*
QUISUMBING, J.: In 1987, petitioners Manuel and Salvacion del Campo brought this complaint for
“repartition, resurvey and reconveyance” against the heirs of the now deceased Jose
FACTS: Salome, Consorcia, Alfredo, Maria, Rosalia, Jose, Quirico and Julita, all Regalado, Sr. Petitioners claimed that they owned an area of 1,544 square meters
surnamed Bornales, were the original co-owners of Lot 162 of the Cadastral Survey of located within Lot 162-C-6 which was erroneously included in TCT No. 14566 in the
Ponteverda, Capiz under OCT No. 18047. As appearing therein, the lot, which name of Regalado. Petitioners alleged that they occupied the disputed area as
consisted of a total area of 27,170 square meters was divided in aliquot shares among residential dwelling ever since they purchased the property from the Distajos way back
the 8 co-owners as follows: in 1951. They also declared the land for taxation purposes and paid the corresponding
Salome Bornales 4/16 taxes.
Consorcia Bornales 4/16
Alfredo Bornales 2/16 On April 1, 1987, summons were served on Regalado’s widow, Josefina Buenvenida,
Maria Bornales 2/16 and two of her children, Rosemarie and Antonio. Josefina and Rosemarie were
Jose Bornales 1/16 declared in default on May 10, 1989 because only Antonio filed an answer to the
Quirico Bornales 1/16 complaint.
Rosalia Bornales 1/16
Julita Bornales 1/16 During trial, petitioners presented the Deed of Absolute Sale executed between
Soledad Daynolo and Salome Bornales as well as the Deed of Mortgage and Deed of
On July 14, 1940, Salome sold part of her 4/16 share in Lot 162 for P200.00 to Soledad Discharge signed by Jose Regalado, Sr. The Deed of Absolute Sale showing the
Daynolo. In the Deed of Absolute Sale signed by Salome and 2 other co-owners, purchase by the Del Campos of the property from the Distajos was likewise given in
Consorcia and Alfredo, the portion of Lot 162 sold to Soledad was described as having evidence. Antonio failed to present evidence to refute the evidence presented by
more or less the following measurements: petitioners.
63-1/2 meters from point “9” to “10,” 35 meters from point “10” to point “11”,
30 meters from point “11” to a certain point parallel to a line drawn from points TC rendered judgment dismissing the complaint. It held that while Salome could
“9” to “10”; and then from this “Certain Point” to point “9” and as shown in the alienate her pro-indiviso share in Lot 162, she could not validly sell an undivided part
accompanying sketch, and made an integral part of this deed, to SOLEDAD thereof by metes and bounds to Soledad, from whom petitioners derived their title. The
DAYNOLO, her heirs and assigns. trial court also reasoned that petitioners could not have a better right to the property
even if they were in physical possession of the same and declared the property for
Thereafter, Soledad Daynolo immediately took possession of the land described above taxation purposes, because mere possession cannot defeat the right of the Regalados
and built a house thereon. A few years later, Soledad and her husband, Simplicio who had a Torrens title over the land. CA affirmed TC. Hence, the present petition
Distajo, mortgaged the subject portion of Lot 162 as security for a P400.00 debt to Jose before the SC.
Regalado, Sr. This transaction was evidenced by a Deed of Mortgage dated May 1,
1947. ISSUE:
1. WON the sale by a co-owner of a physical portion of an undivided property held in
On April 14, 1948, 3 of the 8 co-owners of Lot 162, specifically, Salome, Consorcia and common be valid
Alfredo, sold 24,993 square 2. WON respondents are estopped from denying petitioners’ right and title over the
meters of said lot to Jose Regalado, Sr. disputed area
3. WON petitioners entitled to ‘repartition, resurvey and reconveyance’ of the property
On May 4, 1951, Simplicio Distajo, heir of Soledad Daynolo who had since died, paid in question
the mortgage debt and redeemed the mortgaged portion of Lot 162 from Jose
Regalado, Sr. The latter, in turn, executed a Deed of Discharge of Mortgage in favor of RULING:
Soledad’s heirs, namely: Simplicio Distajo, Rafael Distajo and Teresita Distajo- 1. YES.
Regalado. On same date, the said heirs sold the redeemed portion of Lot 162 for It seems plain to us that the trial court concluded that petitioners could not have
P1,500.00 to herein petitioners, the spouses Manuel Del Campo and Salvacion acquired ownership of the subject land which originally formed part of Lot 162, on the
Quiachon. ground that their alleged right springs from a void sale transaction between Salome and
Soledad. The mere fact that Salome purportedly transferred a definite portion of the co-
Meanwhile, Jose Regalado, Sr. caused the reconstitution of OCT No. 18047. The owned lot by metes and bounds to Soledad, however, does not per se render the sale
reconstituted OCT No. RO-4541 initially reflected the shares of the original co-owners a nullity. This much is evident under Article 493 of the Civil Code and pertinent
in Lot 162. However, title was transferred later to Jose Regalado, Sr. who subdivided jurisprudence on the matter. More particularly in Lopez vs. Vda. De Cuaycong, et al.
the entire property into smaller lots, each covered by a respective title in his name. One which we find relevant, the Court, speaking through Mr. Justice Bocobo, held that:
. . . The fact that the agreement in question purported to sell a concrete as co-owner of the land. Needless to say, these rights are preserved notwithstanding
portion of the hacienda does not render the sale void, for it is a well- the issuance of TCT No. 14566 in Regalado’s name in 1977.
established principle that the binding force of a contract must be
recognized as far as it is legally possible to do so. “Quando res non valet Be that as it may, we find that the area subject matter of this petition had already been
ut ago, valeat quantum valere potest.” (When a thing is of no force as I do it, effectively segregated from the ’mother lot’ even before title was issued in favor of
it shall have as much force as it can have.) Regalado. It must be noted that 26 years had lapsed from the time petitioners bought
and took possession of the property in 1951 until Regalado procured the issuance of
Applying this principle to the instant case, there can be no doubt that the transaction TCT No. 14566. Additionally, the intervening years between the date of petitioners’
entered into by Salome and Soledad could be legally recognized in its entirety since purchase of the property and 1987 when petitioners filed the instant complaint,
the object of the sale did not even exceed the ideal shares held by the former in the co- comprise all of 36 years. However, at no instance during this time did respondents or
ownership. As a matter of fact, the deed of sale executed between the parties expressly Regalado, for that matter, question petitioners’ right over the land in dispute. In the case
stipulated that the portion of Lot 162 sold to Soledad would be taken from Salome’s of Vda. de Cabrera vs. Court of Appeals, we had occasion to hold that where the
4/16 undivided interest in said lot, which the latter could validly transfer in whole or in transferees of an undivided portion of the land allowed a co-owner of the
part even without the consent of the other co-owners. Salome’s right to sell part of her property to occupy a definite portion thereof and had not disturbed the same for
undivided interest in the co-owned properly is absolute in accordance with the well- a period too long to be ignored, the possessor is in a better condition or right
settled doctrine that a co-owner has full ownership of his pro-indiviso share and than said transferees. (Potior est condition possidentis). Such undisturbed
has the right to alienate, assign or mortgage it, and substitute another person in possession had the effect of a partial partition of the co-owned property which
its enjoyment. Since Salome’s clear intention was to sell merely part of her aliquot entitles the possessor to the definite portion which he occupies. Conformably,
share in Lot 162, in our view no valid objection can be made against it and the sale can petitioners are entitled to the disputed land, having enjoyed uninterrupted possession
be given effect to the foil extent. thereof for a total of 49 years up to the present.

We are not unaware of the principle that a co-owner cannot rightfully dispose of The lower court’s reliance on the doctrine that mere possession cannot defeat the right
a particular portion of a co-owned property prior to partition among all the co- of a holder of a registered Torrens title over property is misplaced, considering that
owners. However, this should not signify that the vendee does not acquire petitioners were deprived of their dominical rights over the said lot through fraud and
anything at all in case a physically segregated area of the co-owned lot is in fact with evident bad faith on the part of Regalado. Failure and intentional omission to
sold to him. Since the co-owner/vendor’s undivided interest could properly to the disclose the fact of actual physical possession by another person during registration
portion which may be allotted to him in the division upon the termination of the proceedings constitutes actual fraud. Likewise, it is fraud to knowingly omit or conceal
co-ownership be the object of the contract of sale between the parties, what the a fact, upon which benefit is obtained to the prejudice of a third person. In this case, we
vendee obtains by virtue of such a sale are the same rights as the vendor had as are convinced that Regalado knew of the fact that he did not have a title to the entire
co-owner, in an ideal share equivalent to the consideration given under their lot and could not, therefore, have validly registered the same in his name alone because
transaction. In other words, the vendee steps into the shoes of the vendor as co- he was aware of petitioners’ possession of the subject portion as well as the sale
owner and acquires a proportionate abstract share in the property held in between Salome and Soledad.
common.
2. YES.
Resultantly, Soledad became a co-owner of Lot 162 as of the year 1940 when the sale That Regalado had notice of the fact that the disputed portion of Lot 162 was under
was made in her favor. It follows that Salome, Consorcia and Alfredo could not have claim of ownership by petitioners and the latter’s predecessor is beyond question.
sold the entire Lot 162 to Jose Regalado, Sr. on April 14, 1948 because at that time, Records show that the particular area subject of this case was mortgaged by Soledad
the ideal shares held by the three co-owners/vendors were equivalent to only 10/16 of and her husband to Jose Regalado, Sr. as early as May 1, 1947 or one year prior to
the undivided property less the aliquot share previously sold by Salome to Soledad. the alienation of the whole lot in favor of the latter. Regalado never questioned the
Based on the principle that “no one can give what he does not have,” Salome, ownership of the lot given by Soledad as security for the P400.00 debt and he must
Consorcia and Alfredo could not legally sell the shares pertaining to Soledad since a have at least known that Soledad bought the subject portion from Salome since he
co-owner cannot alienate more than his share in the co-ownership. We have ruled could not have reasonably accepted the lot as security for the mortgage debt if such
many times that even if a co-owner sells the whole property as his, the sale will were not the case. By accepting the said portion of Lot 162 as security for the mortgage
affect only his own share but not those of the other co-owners who did not obligation, Regalado had in fact recognized Soledad’s ownership of this definite portion
consent to the sale. Since a co-owner is entitled to sell his undivided share, a of Lot 162. Regalado could not have been ignorant of the fact that the disputed portion
sale of the entire property by one co-owner will only transfer the rights of said is being claimed by Soledad and subsequently, by petitioners, since Regalado even
co-owner to the buyer, thereby making the buyer a co-owner of the property. executed a Release of Mortgage on May 4, 1951, three years after the entire property
was supposedly sold to him. It would certainly be illogical for any mortgagee to accept
In this case, Regalado merely became a new co-owner of Lot 162 to the extent of the property as security, purchase the mortgaged property and, thereafter, claim the very
shares which Salome, Consorcia and Alfredo could validly convey. Soledad retained same property as his own while the mortgage was still subsisting.
her rights as co-owner and could validly transfer her share to petitioners in 1951. The
logical effect of the second disposition is to substitute petitioners in the rights of Soledad
Consequently, respondents are estopped from asserting that they own the subject land
in view of the Deed of Mortgage and Discharge of Mortgage executed between
Regalado and petitioners’ predecessor-in-interest. As petitioners correctly contend,
respondents are barred from making this assertion under the equitable principle of
estoppel by deed, whereby a party to a deed and his privies are precluded from
asserting as against the other and his privies any right or title in derogation of the deed,
or from denying the truth of any material fact asserted in it. A perusal of the documents
evidencing the mortgage would readily reveal that Soledad, as mortgagor, had declared
herself absolute owner of the piece of land now being litigated. This declaration of fact
was accepted by Regalado as mortgagee and accordingly, his heirs cannot now be
permitted to deny it.

3. YES.
Although Regalado’s certificate of title became indefeasible after the lapse of one year
from the date of the decree of registration, the attendance of fraud in its issuance
created an implied trust in favor of petitioners and gave them the right to seek
reconveyance of the parcel wrongfully obtained by the former. An action for
reconveyance based on an implied trust ordinarily prescribes in ten years. But when
the right of the true and real owner is recognized, expressly or implicitly such as when
he remains undisturbed in his possession, the said action is imprescriptible, it being in
the nature of a suit for quieting of title. Having established by clear and convincing
evidence that they are the legal owners of the litigated portion included in TCT No.
14566, it is only proper that reconveyance of the property be ordered in favor of
petitioners. The alleged incontrovertibility of Regalado’s title cannot be successfully
invoked by respondents because certificates of title merely confirm or record title
already existing and cannot be used to protect a usurper from the true owner or be
used as a shield for the commission of fraud.

WHEREFORE, the petition is GRANTED. The assailed decision of the Court of Appeals
in CA-G.R. CV No. 30438 is REVERSED and SET ASIDE. The parties are directed to
cause a SURVEY for exact determination of their respective portions in Lot 162-C-6.
Transfer Certificate of Title No. 14566 is declared CANCELLED and the Register of
Deeds of Capiz is ordered to ISSUE a new title in accordance with said survey, upon
finality of this decision.

Costs against respondents.

SO ORDERED.
EXTRAORDINARY DEVELOPMENT CORPORATION, Petitioner, v. HERMINIA F. respondents; and that EDC was not a buyer in good faith because it knew that
SAMSON-BICO AND ELY B. FLESTADO, Respondents. G.R. No. 191090, October respondents were co-owners of the subject property because Herminia informed EDC
13, 2014 of such fact through a letter dated 9 March 2000.

FACTS: Apolonio Ballesteros and Maria Membrebe were husband and wife. They EDC appealed to the Court of Appeals. Court of Appeals partially granted the appeal.
begot two (2) children, namely, Juan M. Ballesteros who married Leonarda Tambongco
and Irenea Ballesteros who married Santiago Samson. Juan and Leonarda begot six The Court of Appeals ruled that respondents were able to establish their co-ownership
(6) children, namely, Leonardo Marcelina, Lydia, Cresencia, Lourdes and Juan Jr., over one-half of the subject property. The appellate court pointed out that the heirs of
while Irenea and Santiago begot two (2) children, namely, Herminia and Merlita, who Juan categorically admitted in their Answer, as well as during the hearing the existence
married Ely D. Flestado. of co-ownership. The appellate court agreed with the trial court’s finding that the heirs
of Juan, as co-owners, could only alienate or convey to EDC their one-half portion of
During his lifetime, Apolonio owned a parcel of land situated at Barangay Pantok, the subject property which may be allotted to them in the division upon the termination
Binangonan, Rizal. When Apolonio and Maria died, the property was inherited by Juan of the co-ownership. Thus, the sale will affect only their share but not those of the other
and Irenea. When the latter died, the heirs of Juan and Irenea became co-owners of co-owners who did not consent to the sale. The appellate court disputed the
the property. submission of EDC that whatever admissions made by the heirs of Juan regarding the
ownership of the subject property is effective only insofar as they are concerned but
On 16 April 2002, the heirs of Juan, without the consent of respondents, the heirs of such do not bind or affect the defenses it raised. The appellate court declared that the
Irenea executed in favor of petitioner EDC a Deed of Absolute Sale covering the subject execution by the heirs of Juan of the Deed of Absolute Sale over the subject property
property. Prior to the sale, respondents claimed that they learned that the property had which they do not exclusively own but is admittedly co-owned by them together with
been the subject of a contract to sell between the heirs of Juan and EDC. On 7 March respondents, was valid only to the extent of the former’s undivided one-half share
2000, respondents wrote to EDC informing it of the existence of co-ownership over the thereof, as they had no title or interest to transfer the other one-half portion which
subject property. EDC wrote back that it will look into the matter and asked respondents pertains to the appellees without the latter’s consent. EDC’s invocation of it being a
to further establish the basis of their claims. buyer in good faith was not considered by the appellate court because the subject
property is an unregistered land and the defense of having purchased the property in
EDC was able to cause the registration of the Deed of Absolute Sale in its name. This
good faith may be availed of only where registered land is involved and the buyer had
prompted respondents to file the Complaint for Annulment of Contract and
relied in good faith on the clear title of the registered owner. The appellate court
Reconveyance of Possession with Damages.
sustained the trial court’s finding that there was no denial of due process as EDC was
In its Answer, EDC alleged that it is a buyer in good faith and for value of the subject given the opportunity to advocate its cause and defend its interest.
property because it was of the honest belief that the heirs of Juan are the only heirs of
However, the appellate court reversed the ruling of the trial court that the Deed of
the late Apolonio.
Absolute Sale is null and void. According to the appellate court, the same is valid with
On the other hand, the heirs of Juan asserted that respondents were aware of and were respect to the transfer of the rights of the co-owners-sellers heirs of Juan over the one-
parties to the contract to sell entered into by them and EDC. The heirs of Juan claimed half portion of the subject property, thereby making EDC a co-owner thereof.
that respondents received their share in the downpayment made by EDC but they were Consequently, the appellate court ordered the heirs of Ballesteros to return to EDC the
both unpaid of the balance on the cost of the land. amount o one-half of the purchase price. The award of moral and exemplary damages,
as well as attorney’s fees, were deleted for lack of legal and factual bases.
RTC ruled in favor of respondents.
Interestingly, it was EDC who pursued this petition and insist that respondents failed to
The Deed of Absolute Sale was declared null and void to the extent of one half of the prove co-ownership presumably to validate in its entirety the Deed of Absolute Sale it
property. [EDC] is ordered to vacate, surrender or reconvey ownership and possession entered into with the heirs of Juan. EDC reiterates its argument that the testimony of
of the parcel of land to [respondents] or the heirs of Apolonio Ballesteros or that they Herminia is insufficient to prove that respondents are entitled to inherit one-half of the
be reinstated to the lawful ownership of one-half (1/2) of the property sold or 14,874 subject property from Apolonio. According to EDC, respondents should have
square meters. established that Irenea is a legitimate child of Apolonio; that Irenea and Juan are the
only legitimate compulsory heirs of Apolonio; that Apolonio predeceased Irenea and
The trial court found that respondents and the heirs of Juan are co-owners of the
Juan; that Hermina and Merlita are the legitimate children of Irenea; and that Irenea
subject property; that at the time of sale, the heirs of Juan did not have the right to sell
predeceased Herminia. EDC also maintains that it is a buyer in good faith and that it
the one half share of the heirs of Irenea; that the sale did not bind the heirs of Irenea;
was respondents who acted in bad faith, thus it prays for damages.
that there was fraud in the execution of the Deed of Absolute Sale when the heirs of
Juan failed to disclose to EDC that one half of the property sold is owned by Issue: Whether or not co-ownership was established
RULING: Yes. contradicted unless it is shown that the admission was made through palpable mistake
or that no such admission was made.
As borne by the records, respondents were able to convincingly establish their co-
ownership over one-half of the subject property. EDC avers that said judicial admission should not bind it because it was an innocent
purchaser in good faith. The Court of Appeals debunked this contention and correctly
Herminia has successfully established her successional rights over the subject property ruled, as follow:
through her clear testimony and admitted by the opposing counsel.
In a contract of sale, it is essential that the seller is the owner of the property
We also took into consideration the admissions made by the heirs of Juan in their he is selling. Under Article 1458 of the Civil Code, the principal obligation
Answer to the Complaint filed by respondents before the trial court that defendants of a seller is to transfer the ownership of the property sold. Also, Article
BALLESTEROS admit the allegations that: 1459 of the Civil Code provides that the thing must be licit and the vendor
must have a right to transfer the ownership thereof at the time it is
[Respondents] together with defendants-Ballesteros and defendant Juan T.
delivered. The execution by appellants Ballesteros of the Deed of
Ballesteros, Jr., are co-owners of a parcel ofl and situated at Barangay Pantok,
Absolute Sale over the subject property which they do not exclusively
Binangonan, Rizal by virtue of succession;
own but is admittedly co-owned by them together with the
[Herminia], defendants Ballesteros and defendant Juan T. Ballesteros are the [respondents], was valid only to the extent of the former’s undivided
Heirs of the late Spouses Apolonio Ballesteros and Maria Membrebe who one-half share thereof, as they had no title or interest to transfer the
were the parents of the late Juan M. Ballesteros and the late Irenea M. other one-half portion which pertains to the [respondents] without the
Ballesteros-Samson. latter’s consent. It is an established principle that no one can give what one
does not have – nemo dat quod non habet. Accordingly, one can sell only
During her lifetime, Irenea M. Ballesteros married Santiago Samson, now what one owns or is authorized to sell, and the buyer can acquire no more
deceased, with whom she had two (2) children, namely: [Herminia] and Merlita than what the seller can transfer legally. Thus, since appellant EDC’s rights
B. Samson over the subject property originated from sellers-appellants Ballesteros, said
corporation merely stepped into the shoes of its sellers and cannot have a
Merlita B. Samson married [respondent] Ely and later died childless and
better right than what its sellers have. Indeed, a spring cannot rise higher than
intestate.
its source.
In his lifetime, Juan M. Ballesteros married Leonarda Tambongco, now
Moreover, EDC was given an ample opportunity to be heard through counsel. The
deceased, with whom she had six (6) children, namely: defendants
essence of due process is the right to be heard. Due process is satisfied when the
Ballesteros and defendant Juan T. Ballesteros, Jr.;
parties are afforded a fair and reasonable opportunity to explain their respective sides
Likewise, during the lifetime of Apolonio Ballesteros, he was the owner of the of the controversy. Thus, when the party seeking due process was in fact given several
parcel of land mentioned in paragraph 8 hereof and the same was declared opportunities to be heard and air his side, but it is by his own fault or choice he
for taxation purposes under his name x x x squanders these chances, then his cry for due process must fail.

Furthermore, Juan testified during the 12 March 2007 hearing that It is apparent that despite numerous resetting of the case for EDC, it failed to appear
respondents are co-owners of the subject property, to wit. because of the absence of its counsel. Therefore, EDC was not deprived of its day in
court and he cannot feign denial of due process.
A party may make judicial admissions in (a) the pleadings, (b) during the trial, either by
verbal or written manifestations or stipulations, or (c) in other stages of the judicial Having established respondents’ co-ownership rights over the subject property, we find
proceeding.23 Sec. 4, Rule 129 of the Revised Rules of Court provides: no error in the appellate court’s ruling sustaining the validity of the Deed of Absolute
Sale but only with respect to the rights of the heirs of Juan over one-half of the property.
Sec. 4. Judicial admissions. – An admission, verbal or written, made by a party
in the course of the proceedings in the same case, does not require proof. The Article 493 of the Civil Code recognizes the absolute right of a co-owner to freely
admission may be contradicted only by showing that it was made through dispose of his pro indiviso share as well as the fruits and other benefits arising
palpable mistake or that no such admission was made. from that share, independently of the other co-owners, thus:

The Answer submitted by the heirs of Juan, as well as the testimony of Juan constitute Art. 493. Each co-owner shall have the full ownership of his part of the fruits
judicial admissions. Well-settled is the rule that a judicial admission conclusively binds and benefits pertaining thereto, and he may therefore alienate, assign or
the party making it. He cannot thereafter take a position contradictory to, or inconsistent mortgage it, and even substitute another person in its enjoyment, except when
with his pleadings. Acts or facts admitted do not require proof and cannot be personal rights are involved. But the effect of the alienation or the mortgage,
with respect to the co-owners, shall be limited to the portion which may be vendor as co-owner and acquires a proportionate abstract share in the property held in
allotted to him in the division upon the termination of the co-ownership. common.

In Spouses Del Campo v. Court of Appeals, we had the occasion to expound the rights We are also in full accord with the appellate court’s order for the heirs of Juan to return
of a co-owner vis-à-vis the vendee, thus: one-half of the purchase price to EDC. There is unjust enrichment when a person
unjustly retains a benefit to the loss of another, or when a person retains money or
x x x Would the sale by a co-owner of a physical portion of an undivided property of another against the fundamental principles of justice, equity and good
property held in common be valid? x x x conscience. Therefore, it is correct for the Court of Appeals to order the heirs of Juan
to return the amount of representing one-half of the purchase price to prevent unjust
On the first issue, it seems plain to us that the trial court concluded that
enrichment at the expense of EDC.
petitioners could not have acquired ownership of the subject land which
originally formed part of Lot 162, on the ground that their alleged right springs
from a void sale transaction between Salome and Soledad. The mere fact that
Salome purportedly transferred a definite portion of the co-owned lot by metes
and bounds to Soledad, however, does not per se render the sale a nullity.
This much is evident under Article 493 of the Civil Code and pertinent
jurisprudence on the matter. More particularly in Lopez vs. Vda. De Cuaycong,
et. al. which we find relevant, the Court, speaking through Mr. Justice Bocobo,
held that:

The fact that the agreement in question purported to sell a concrete portion of
the hacienda does not render the sale void, for it is a well-established principle
that the binding force of a contract must be recognized as far as it is legally
possible to do so. “Quando res non valet ut ago, valeat quantum valere
potest.” (When a thing is of no force as I do it, it shall have as much force as
it can have.)

Applying this principle to the instant case, there can be no doubt that the
transaction entered into by Salome and Soledad could be legally recognized
in its entirety since the object of the sale did not even exceed the ideal shares
held by the former in the co-ownership. As a matter of fact, the deed of sale
executed between the parties expressly stipulated that the portion of Lot 162
sold to Soledad would be taken from Salome’s 4/16 undivided interest in said
lot, which the latter could validly transfer in whole or in part even without the
consent of the other co-owners. Salome’s right to sell part of her undivided
interest in the co-owned property is absolute in accordance with the well-
settled doctrine that a co-owner has full ownership of his pro-indiviso share
and has the right to alienate, assign or mortgage it, and substitute another
person in its enjoyment. Since Salome’s clear intention was to sell merely part
of her aliquot share in Lot 162, in our view no valid objection can be made
against it and the sale can be given effect to the full extent.

We are not unaware of the principle that a co-owner cannot rightfully dispose of a
particular portion of a co-owned property prior to partition among all the co-owners.
However, this should not signify that the vendee does not acquire anything at all in case
a physically segregated area of the co-owned lot is in fact sold to him. Since the co-
owner/vendor’s undivided interest could properly be the object of the contract of sale
between the parties, what the vendee obtains by virtue of such a sale are the same
rights as the vendor had as co-owner, in an ideal share equivalent to the consideration
given under their transaction. In other words, the vendee steps into the shoes of the
VICENTE TORRES, JR., CARLOS VELEZ, AND THE HEIRS OF MARIANO VELEZ, does not mention any boundaries or specific portion. He explained that Jesus permitted
NAMELY: ANITA CHIONG VELEZ, ROBERT OSCAR CHIONG VELEZ, SARAH him to occupy a portion not exceeding 3000 square meters conditioned on the result of
JEAN CHIONG VELEZ AND TED CHIONG VELEZ, Petitioners, v. LORENZO the partition of the co-owners.
LAPINID AND JESUS VELEZ, Respondents. G.R. No. 187987, November 26, 2014
Regarding the forcible entry case, Jesus and Lapinid admitted that such case was filed
FACTS: On 4 February 2006, Vicente V. Torres, Jr., Mariano Velez and Carlos Velez but the same was already dismissed by the Municipal Trial Court. In that decision, it
(petitioners) filed a Complain before RTC praying for the nullification of the sale of real was ruled that the buyers, including Lapinid, were buyers in good faith since a proof of
property by respondent Jesus Velez in favor of Lapinid; the recovery of possession ownership was shown to them by Jesus before buying the property.
and ownership of the property; and the payment of damages.
RTC: dismissed the complaint of petitioners in this wise:
Petitioners alleged in their complaint that they, including Jesus, are co-owners of
several parcels of land including the disputed Lot. No. 4389 located at Cogon, Carcar, CA: affirmed the decision of the trial court.
Cebu. Sometime in 1993, Jesus filed an action for partition of the parcels of land against
It validated the sale and ruled that the compromise agreement did not affect the validity
the petitioners and other co-owners before RTC Cebu City. On 13 August 2001, a
of the sale previously executed by Jesus and Lapinid.
judgment was rendered based on a compromise agreement signed by the parties
wherein they agreed that Jesus, Mariano and Vicente were jointly authorized to sell the Upon appeal before this Court, the petitioners echo the same arguments posited before
said properties and receive the proceeds thereof and distribute them to all the co- the lower courts. They argue that Lapinid, as the successor-in-interest of Jesus, is also
owners. However, the agreement was later amended to exclude Jesus as an authorized bound by the 2001 judgment based on compromise stating that the parcels of land must
seller. Pursuant to their mandate, the petitioners inspected the property and discovered be sold jointly by Jesus, Mariano and Vicente and the proceeds of the sale be divided
that Lapinid was occupying a specific portion of the 3000 square meters of Lot No. 4389 among the co-owners. To further strengthen their contention, they advance the
by virtue of a deed of sale executed by Jesus in favor of Lapinid. It was pointed out by argument that since the portion sold was a definite and specific portion of a co-owned
petitioner that as a consequence of what they discovered, a forcible entry case was property, the entire deed of sale must be declared null and void.
filed against Lapinid.
Issue: Whether or not Jesus, as a co-owner, can validly sell a portion of the property
The petitioners prayed that the deed of sale be declared null and void arguing that the he co-owns in favor of another person.
sale of a definite portion of a co-owned property without notice to the other co-owners
is without force and effect. Further, the complainants prayed for payment of rental fees Ruling: Yes.
from January 2004 or from the time of deprivation of property in addition to attorney’s
Admittedly, Jesus sold an area of land to Lapinid on 9 November 1997.
fees and litigation expenses.
A co-owner has an absolute ownership of his undivided and pro-indiviso share in the
Answering the allegations, Jesus admitted that there was a partition case between him
co-owned property. He has the right to alienate, assign and mortgage it, even to the
and the petitioners filed in 1993 involving several parcels of land including the contested
extent of substituting a third person in its enjoyment provided that no personal rights
Lot No. 4389. However, he insisted that as early as 6 November 1997, a motion was
will be affected. This is evident from the provision of the Civil Code:
signed by the co-owners (including the petitioners) wherein Lot No. 4389 was agreed
to be adjudicated to the co-owners belonging to the group of Jesus and the other lots Art. 493. Each co-owner shall have the full ownership of his part and of the
be divided to the other co-owners belonging to the group of Torres. Jesus further fruits and benefits pertaining thereto, and he may therefore alienate, assign or
alleged that even prior to the partition and motion, several co-owners in his group had mortgage it, and even substitute another person in its enjoyment, except when
already sold their shares to him in various dates of 1985, 1990 and 2004. Thus, when personal rights are involved. But the effect of the alienation or the mortgage,
the motion was filed and signed by the parties on 6 November 1997, his rights as a with respect to the co-owners, shall be limited to the portion which may be
majority co-owner (73%) of Lot No. 4389 became consolidated. Jesus averred that it allotted to him in the division upon the termination of the co-ownership.
was unnecessary to give notice of the sale as the lot was already adjudicated in his
favor. He clarified that he only agreed with the 2001 Compromise Agreement believing A co-owner is an owner of the whole and over the whole he exercises the right
that it only pertained to the remaining parcels of land excluding Lot No. 4389. of dominion, but he is at the same time the owner of a portion which is truly
abstract.18 Hence, his co-owners have no right to enjoin a co-owner who
On his part, Lapinid admitted that a deed of sale was entered into between him and intends to alienate or substitute his abstract portion or substitute a third person
Jesus pertaining to a parcel of land with an area of 3000 square meters. However, he in its enjoyment.
insisted on the validity of sale since Jesus showed him several deeds of sale making
him a majority owner of Lot No. 4389. He further denied that he acquired a specific and In this case, Jesus can validly alienate his co-owned property in favor of Lapinid,
definite portion of the questioned property, citing as evidence the deed of sale which free from any opposition from the co-owners. Lapinid, as a transferee, validly
obtained the same rights of Jesus from the date of the execution of a valid sale. This Court has ruled in many cases that even if a co-owner sells the whole property as
Absent any proof that the sale was not perfected, the validity of sale subsists. In his, the sale will affect only his own share but not those of the other co-owners who did
essence, Lapinid steps into the shoes of Jesus as co-owner of an ideal and not consent to the sale. This is because the sale or other disposition of a co-owner
proportionate share in the property held in common. Thus, from the perfection affects only his undivided share and the transferee gets only what would correspond to
of contract on 9 November 1997, Lapinid eventually became a co-owner of the his grantor in the partition of the thing owned in common.
property.
We find unacceptable the argument that Lapinid must pay rental payments to the other
Even assuming that the petitioners are correct in their allegation that the disposition in co-owners.
favor of Lapinid before partition was a concrete or definite portion, the validity of sale
still prevails. As previously discussed, Lapinid, from the execution of sale, became a co-owner
vested with rights to enjoy the property held in common.
In a catena of decisions, the Supreme Court had repeatedly held that no individual can
claim title to a definite or concrete portion before partition of co-owned property. Each Clearly specified in the Civil Code are the following rights:
co-owner only possesses a right to sell or alienate his ideal share after partition.
Art. 486. Each co-owner may use the thing owned in common, provided he
However, in case he disposes his share before partition, such disposition does not
does so in accordance with the purpose for which it is intended and in such a
make the sale or alienation null and void. What will be affected on the sale is only his
way as not to injure the interest of the co-ownership or prevent the other co-
proportionate share, subject to the results of the partition. The co-owners who did not
owners from using it according to their rights. The purpose of the co-ownership
give their consent to the sale stand to be unaffected by the alienation.
may be changed by agreement, express or implied.
Consequently, whether the disposition involves an abstract or concrete portion of the
Art. 493. Each co-owner shall have the full ownership of his part and of the
co-owned property, the sale remains validly executed.
fruits and benefits pertaining thereto, and he may therefore alienate, assign or
The validity of sale being settled, it follows that the subsequent compromise agreement mortgage it and even substitute another person in its enjoyment, except when
between the other co-owners did not affect the rights of Lapinid as a co-owner. personal rights are involved. But the effect of the alienation or mortgage, with
respect to the co-owners, shall be limited to the portion which may be allotted
Records show that on 13 August 2001, a judgment based on compromise agreement to him in the division upon the termination of the co-ownership.
was rendered with regard to the previous partition case involving the same parties
pertaining to several parcels of land, including the disputed lot. The words of the Affirming these rights, the Court held in Aguilar v. Court of Appeals that:
compromise state that:
x x x Each co-owner of property held pro indiviso exercises his rights over the
COME NOW[,] the parties and to this Honorable Court, most respectfully whole property and may use and enjoy the same with no other limitation than
state that instead of partitioning the properties, subject matter of litigation, that that he shall not injure the interests of his co-owners, the reason being that
they will just sell the properties covered by TCT Nos. 25796, 25797 and 25798 until a division is made, the respective share of each cannot be determined
of the Register of Deeds of the Province of Cebu and divide the proceeds and every co-owner exercises, together with his co-participants joint
among themselves. ownership over the pro indiviso property, in addition to his use and enjoyment
of the same.
That Jesus Velez, Mariano Velez and Vicente Torres, Jr. are currently
authorized to sell said properties, receive the proceeds thereof and distribute From the foregoing, it is absurd to rule that Lapinid, who is already a co-owner, be
them to the parties. ordered to pay rental payments to his other co-owners. Lapinid’s right of enjoyment
over the property owned in common must be respected despite opposition and may not
Be that as it may, the compromise agreement failed to defeat the already accrued right be limited as long he uses the property to the purpose for which it is intended and he
of ownership of Lapinid over the share sold by Jesus. As early as 9 November 1997, does not injure the interest of the co-ownership.
Lapinid already became a co-owner of the property and thus, vested with all the rights
enjoyed by the other co-owners. The judgment based on the compromise agreement,
which is to have the covered properties sold, is valid and effectual provided as it does
not affect the proportionate share of the non-consenting party. Accordingly, when the
compromise agreement was executed without Lapinid’s consent, said agreement could
not have affected his ideal and undivided share. Petitioners cannot sell Lapinid’s share
absent his consent. Nemo dat quod non habet – “no one can give what he does not
have.”
VILMA QUINTOS, represented by her Attorney-in-Fact FIDEL I. QUINTOS, JR.; Bienvenido and Escolastica, the Candelarios accepted from their co-respondents their
FLORENCIA I. DANCEL, represented by her Attorney-in-Fact FLOVY I. DANCEL; share in the subject property as payment. Lastly, respondents sought, by way of
and CATALINO L. IBARRA, Petitioners, vs. PELAGIA I. NICOLAS, NOLI L. counterclaim, the partition of the property.
IBARRA, SANTIAGO L. IBARRA, PEDRO L. IBARRA, DAVID L. IBARRA,
GILBERTO L. IBARRA, HEIRS OF AUGUSTO L. IBARRA, namely CONCHITA R., The quieting of title case was eventually raffled to the same trial court that dismissed
IBARRA, APOLONIO IBARRA, and NARCISO IBARRA, and the spouses RECTO prior Civil Case. During pre-trial, respondents, or defendants a quo, admitted having
CANDELARIO and ROSEMARIE CANDELARIO, Respondents. G.R. No. 210252 filed an action for partition, that petitioners did not participate in the Deed of Adjudication
June 16, 2014 that served as the basis for the issuance of TCT No. 390484, and that the Agreement
of Subdivision that led to the issuance of TCT No. 434304 in favor of respondent
Facts: Petitioners Vilma Quintos, Florencia Dancel, and Catalino Ibarra, and spouses Candelario was falsified. Despite the admissions of respondents, however,
respondents Pelagia Nicolas, Noli Ibarra, Santiago Ibarra, Pedro Ibarra, David Ibarra, the RTC, through its May 27, 2012 Decision, dismissed petitioners’ complaint. The court
Gilberto Ibarra, and the late Augusto Ibarra are siblings. Their parents, Bienvenido and did not find merit in petitioners’ asseverations that they have acquired title over the
Escolastica Ibarra, were the owners of the subject property, a 281 sqm. parcel of land property through acquisitive prescription and noted that there was no document
situated along Quezon Ave., Poblacion C, Camiling, Tarlac, covered by Transfer evidencing that their parents bequeathed to them the subject property. Finding that
Certificate Title (TCT) No. 318717. respondent siblings were entitled to their respective shares in the property as
descendants of Bienvenido and Escolastica Ibarra and as co-heirs of petitioners, the
By 1999, both Bienvenido and Escolastica had already passed away, leaving to their subsequent transfer of their interest in favor of respondent spouses Candelario was
ten (10) children ownership over the subject property. Subsequently, sometime in 2002, then upheld by the trial court.
respondent siblings brought an action for partition against petitioners. RTC dismissed
the case for failure of the parties, as well as their counsels, to appear despite due notice. Aggrieved, petitioners appealed the trial court’s Decision to the CA.

Having failed to secure a favorable decision for partition, respondent siblings instead CA: denying the appeal.
resorted to executing a Deed of Adjudication on September 21, 2004 to transfer the
property in favor of the ten (10) siblings. As a result, TCT No. 318717 was canceled Similar to the trial court, the court a quo found no evidence on record to support
and in lieu thereof, TCT No. 390484 was issued in its place by the Registry of Deeds petitioners’ claim that the subject property was specifically bequeathed by Bienvenido
of Tarlac in the names of the ten (10) heirs of the Ibarra spouses. and Escolastica Ibarra in their favor as their share in their parents’ estate. It also did
not consider petitioners’ possession of the property as one that is in the concept of an
Subsequently, respondent siblings sold their 7/10 undivided share over the property in owner. Ultimately, the appellate court upheld the finding that petitioners and respondent
favor of their co-respondents, the spouses Recto and Rosemarie Candelario. By virtue spouses Candelario co-own the property, 30-70 in favor of the respondent spouses.
of a Deed of Absolute Sale executed in favor of the spouses Candelario and an
Agreement of Subdivision purportedly executed by them and petitioners, TCT No. As regards the issue of partition, the CA added:
390484 was partially canceled and TCT No. 434304 was issued in the name of the
x x x Since it was conceded that the subject lot is now co-owned by the plaintiffs-
Candelarios, covering the 7/10portion.
appellants, (with 3/10 undivided interest) and defendants-appellees Spouses
On June 1, 2009, petitioners filed a complaint for Quieting of Title and Damages against Candelarios (with 7/10 undivided interest) and considering that plaintiffs-appellants had
respondents wherein they alleged that during their parents’ lifetime, the couple already constructed a 3-storey building at the back portion of the property, then
distributed their real and personal properties in favor of their ten (10) children. Upon partition, in accordance with the subdivision plan undertaken by defendants-appellants
distribution, petitioners alleged that they received the subject property and the house [sic] spouses, is in order.
constructed thereon as their share. They likewise averred that they have been in Issue:
adverse, open, continuous, and uninterrupted possession of the property for over four
(4) decades and are, thus, entitled to equitable title thereto. They also deny any 1. Whether or not the petitioners were able to prove ownership over the property- NO.
participation in the execution of the aforementioned Deed of Adjudication dated
September 21, 2004 and the Agreement of Subdivision. Respondents countered that 2. Whether or not the respondents’ counterclaim for partition is already barred by laches
petitioners’ cause of action was already barred by estoppel when sometime in 2006, or res judicata; and
one of petitioners offered to buy the 7/10 undivided share of the respondent siblings.
3. Whether or not the CA was correct in approving the subdivision agreement as basis
They point out that this is an admission on the part of petitioners that the property is not
for the partition of the property.
entirely theirs. In addition, they claimed that Bienvenido and Escolastica Ibarra
mortgaged the property but because of financial constraints, respondent spouses
Candelario had to redeem the property in their behalf. Not having been repaid by
Ruling: 2. The counterclaim for partition is not barred by prior judgment

1. Petitioners were not able to prove equitable title or ownership over the This brings us to the issue of partition as raised by respondents in their counterclaim.
property In their answer to the counterclaim, petitioners countered that the action for partition
has already been barred by res judicata.
Quieting of title is a common law remedy for the removal of any cloud, doubt, or
uncertainty affecting title to real property. For an action to quiet title to prosper, two The doctrine of res judicata provides that the judgment in a first case is final as to the
indispensable requisites must concur, namely: claim or demand in controversy, between the parties and those privy with them, not
only as to every matter which was offered and received to sustain or defeat the claim
(1) the plaintiff or complainant has a legal or equitable title to or interest in the or demand, but as to any other admissible matter which must have been offered for
real property subject of the action; and that purpose and all matters that could have been adjudged in that case.19 It precludes
parties from relitigating issues actually litigated and determined by a prior and final
(2) the deed, claim, encumbrance, or proceeding claimed to be casting cloud
judgment. As held in Yusingco v. Ong Hing Lian:
on the title must be shown to be in fact invalid or inoperative despite its prima
facie appearance of validity or efficacy. It is a rule pervading every well-regulated system of jurisprudence, and is put
upon two grounds embodied in various maxims of the common law; the one,
In the case at bar, the CA correctly observed that petitioners’ cause of action must
public policy and necessity, which makes it to the interest of the st ate
necessarily fail mainly in view of the absence of the first requisite.
that there should be an end to litigation — republicae ut sit finis litium; the
At the outset, it must be emphasized that the determination of whether or not petitioners other, the hardship on the individual that he should be vexed twice for the
sufficiently proved their claim of ownership or equitable title is substantially a factual same cause — nemo debet bis vexari et eadem causa. A contrary doctrine
issue that is generally improper for Us to delve into. Section 1, Rule 45 of the Rules of would subject the public peace and quiet to the will and neglect of individuals
Court explicitly states that the petition for review on certiorari "shall raise only questions and prefer the gratitude identification of a litigious disposition on the part of
of law, which must be distinctly set forth." In appeals by certiorari, therefore, only suitors to the preservation of the public tranquility and happiness.
questions of law may be raised, because this Court is not a trier of facts and does not
The rationale for this principle is that a party should not be vexed twice concerning the
normally undertake the re-examination of the evidence presented by the contending
same cause. Indeed, res judicata is a fundamental concept in the organization of every
parties during the trial. Although there are exceptions to this general rule as eloquently
jural society, for not only does it ward off endless litigation, it ensures the stability of
enunciated in jurisprudence, none of the circumstances calling for their application
judgment and guards against inconsistent decisions on the same set of facts.
obtains in the case at bar. Thus, We are constrained to respect and uphold the findings
of fact arrived at by both the RTC and the CA. There is res judicata when the following requisites are present:
In any event, a perusal of the records would readily show that petitioners, as aptly (1) the formal judgment or order must be final;
observed by the courts below, indeed, failed to substantiate their claim. Their alleged
open, continuous, exclusive, and uninterrupted possession of the subject property is (2) it must be a judgment or order on the merits, that is, it was rendered after a
belied by the fact that respondent siblings, in 2005, entered into a Contract of Lease consideration of the evidence or stipulations submitted by the parties at the trial of the
with the Avico Lending Investor Co. over the subject lot without any objection from the case;
petitioners.Petitioners’ inability to offer evidence tending to prove that Bienvenido and
Escolastica Ibarra transferred the ownership over the property in favor of petitioners is (3) it must have been rendered by a court having jurisdiction over the subject matter
likewise fatal to the latter’s claim. On the contrary, on May 28, 1998, Escolastica Ibarra and the parties; and
executed a Deed of Sale covering half of the subject property in favor of all her 10
(4) there must be, between the first and second actions, identity of parties, of subject
children, not in favor of petitioners alone.
matter and of cause of action.
The cardinal rule is that bare allegation of title does not suffice. The burden of proof is
In the case at bar, respondent siblings admit that they filed an action for partition
on the plaintiff to establish his or her case by preponderance of evidence.18
docketed as Civil Case No. 02-52, which the RTC dismissed through an Order dated
Regrettably, petitioners, as such plaintiff, in this case failed to discharge the said burden
March 22, 2004 for the failure of the parties to attend the scheduled hearings.
imposed upon them in proving legal or equitable title over the parcel of land in issue.
Respondents likewise admitted that since they no longer appealed the dismissal, the
As such, there is no reason to disturb the finding of the RTC that all 10 siblings inherited
ruling attained finality. Moreover, it cannot be disputed that the subject property in Civil
the subject property from Bienvenido and Escolastica Ibarra, and after the respondent
Case No. 02-52 and in the present controversy are one and the same, and that in both
siblings sold their aliquot share to the spouses Candelario, petitioners and respondent
cases, respondents raise the same action for partition. And lastly, although respondent
spouses became co-owners of the same.
spouses Candelario were not party-litigants in the earlier case for partition, there is
identity of parties not only when the parties in the case are the same, but also between This is not to say, however, that the action for partition will never be barred by res
those in privity with them, such as between their successors-in-interest.25 judicata. There can still be res judicata in partition cases concerning the same parties
and the same subject matter once the respective shares of the co-owners have been
With all the other elements present, what is left to be determined now is whether or not determined with finality by a competent court with jurisdiction or if the court determines
the dismissal of Civil case No. 02-52 operated as a dismissal on the merits that would that partition is improper for co-ownership does not or no longer exists.
complete the requirements of res judicata.
So it was that in Rizal v. Naredo,29 We ruled in the following wise:
Truly, We have had the occasion to rule that dismissal with prejudice under the above-
cited rule amply satisfies one of the elements of res judicata. It is, thus, understandable Article 484 of the New Civil Code provides that there is co-ownership
why petitioners would allege res judicata to bolster their claim. However, dismissal with whenever the ownership of an undivided thing or right belongs to different
prejudice under Rule 17, Sec. 3 of the Rules of Court cannot defeat the right of a co- persons. Thus, on the one hand, a co-owner of an undivided parcel of land is
owner to ask for partition at any time, provided that there is no actual adjudication of an owner of the whole, and over the whole he exercises the right of dominion,
ownership of shares yet. Pertinent hereto is Article 494 of the Civil Code, which reads: but he is at the same time the owner of a portion which is truly abstract. On
the other hand, there is no co-ownership when the different portions owned
Article 494. No co-owner shall be obliged to remain in the co-ownership. Each by different people are already concretely determined and separately
co-owner may demand at any time the partition of the thing owned in common, identifiable, even if not yet technically described.
insofar as his share is concerned.
Pursuant to Article 494 of the Civil Code, no co-owner is obliged to remain in
Nevertheless, an agreement to keep the thing undivided for a certain period the co-ownership, and his proper remedy is an action for partition under Rule
of time, not exceeding ten years, shall be valid. This term may be extended 69 of the Rules of Court, which he may bring at anytime in so far as his share
by a new agreement. is concerned. Article 1079 of the Civil Code defines partition as the separation,
division and assignment of a thing held in common among those to whom it
A donor or testator may prohibit partition for a period which shall not exceed
may belong. It has been held that the fact that the agreement of partition lacks
twenty years. Neither shall there be any partition when it is prohibited by law.
the technical description of the parties’ respective portions or that the subject
No prescription shall run in favor of a co-owner or co-heir against his co-
property was then still embraced by the same certificate of title could not
owners or co-heirs so long as he expressly or impliedly recognizes the co-
legally prevent a partition, where the different portions allotted to each were
ownership. (emphasis supplied)
determined and became separately identifiable.
From the above-quoted provision, it can be gleaned that the law generally does not
The partition of Lot No. 252 was the result of the approved Compromise
favor the retention of co-ownership as a property relation, and is interested instead in
Agreement in Civil Case No. 36-C, which was immediately final and executory.
ascertaining the co-owners’ specific shares so as to prevent the allocation of portions
Absent any showing that said Compromise Agreement was vitiated by fraud,
to remain perpetually in limbo. Thus, the law provides that each co-owner may demand
mistake or duress, the court cannot set aside a judgment based on
at any time the partition of the thing owned in common.
compromise. It is axiomatic that a compromise agreement once approved by
Between dismissal with prejudice under Rule 17, Sec. 3 and the right granted to the court settles the rights of the parties and has the force of res judicata. It
co-owners under Art. 494 of the Civil Code, the latter must prevail. To construe cannot be disturbed except on the ground of vice of consent or forgery.
otherwise would diminish the substantive right of a co-owner through the promulgation
Of equal significance is the fact that the compromise judgment in Civil Case
of procedural rules. Such a construction is not sanctioned by the principle, which is too
No. 36-C settled as well the question of which specific portions of Lot No. 252
well settled to require citation, that a substantive law cannot be amended by a
accrued to the parties separately as their proportionate shares therein.
procedural rule. This further finds support in Art. 496 of the New Civil Code, viz:
Through their subdivision survey plan, marked as Annex "A" of the
Article 496.Partition may be made by agreement between the parties or by Compromise Agreement and made an integral part thereof, the parties
judicial proceedings.1âwphi1 Partition shall be governed by the Rules of Court segregated and separately assigned to themselves distinct portions of Lot No.
insofar as they are consistent with this Code. 252. The partition was immediately executory, having been accomplished and
completed on December 1, 1971 when judgment was rendered approving the
Thus, for the Rules to be consistent with statutory provisions, We hold that Art. 494, as same. The CA was correct when it stated that no co-ownership exist when the
cited, is an exception to Rule 17, Sec. 3 of the Rules of Court to the effect that even if different portions owned by different people are already concretely determined
the order of dismissal for failure to prosecute is silent on whether or not it is with and separately identifiable, even if not yet technically described. (emphasis
prejudice, it shall be deemed to be without prejudice. supplied)
In the quoted case, We have held that res judicata applied because after the acceptable terms with petitioners. It, therefore, lacked the essential requisite of
parties executed a compromise agreement that was duly approved by the court, consent. Thus, to approve the agreement in spite of this fact would be tantamount to
the different portions of the owners have already been ascertained. Thus, there allowing respondent spouses to divide unilaterally the property among the co-owners
was no longer a co-ownership and there was nothing left to partition. This is in based on their own whims and caprices. Such a result could not be countenanced.
contrast with the case at bar wherein the co-ownership, as determined by the trial
court, is still subsisting 30-70 in favor of respondent spouses Candelario. To rectify this with dispatch, the case must be remanded to the court of origin, which
Consequently, there is no legal bar preventing herein respondents from praying for shall proceed to partition the property in accordance with the procedure outlined in Rule
the partition of the property through counterclaim. 69 of the Rules of Court.

3. The counterclaim for partition is not barred by laches

We now proceed to petitioners’ second line of attack. According to petitioners, the claim
for partition is already barred by laches since by 1999, both Bienvenido and Escolastica
Ibarra had already died and yet the respondent siblings only belatedly filed the action
for partition, Civil Case No. 02-52, in 2002. And since laches has allegedly already set
in against respondent siblings, so too should respondent spouses Candelario be barred
from claiming the same for they could not have acquired a better right than their
predecessors-in-interest.

The argument fails to persuade.

Laches is the failure or neglect, for an unreasonable and unexplained length of time, to
do that which––by the exercise of due diligence––could or should have been done
earlier. It is the negligence or omission to assert a right within a reasonable period,
warranting the presumption that the party entitled to assert it has either abandoned or
declined to assert it. The principle is a creation of equity which, as such, is applied not
really to penalize neglect or sleeping upon one’s right, but rather to avoid recognizing
a right when to do so would result in a clearly inequitable situation. As an equitable
defense, laches does not concern itself with the character of the petitioners’ title, but
only with whether or not by reason of the respondents’ long inaction or inexcusable
neglect, they should be barred from asserting this claim at all, because to allow them
to do so would be inequitable and unjust to petitioners.

As correctly appreciated by the lower courts, respondents cannot be said to have


neglected to assert their right over the subject property. They cannot be considered to
have abandoned their right given that they filed an action for partition sometime in 2002,
even though it was later dismissed. Furthermore, the fact that respondent siblings
entered into a Contract of Lease with Avico Lending Investor Co. over the subject
property is evidence that they are exercising rights of ownership over the same.

The CA erred in approving the Agreement for Subdivision

There is merit, however, in petitioners’ contention that the CA erred in approving the
proposal for partition submitted by respondent spouses. Art. 496, as earlier cited,
provides that partition shall either be by agreement of the parties or in accordance with
the Rules of Court. In this case, the Agreement of Subdivision allegedly executed by
respondent spouses Candelario and petitioners cannot serve as basis for partition, for,
as stated in the pre-trial order, herein respondents admitted that the agreement was a
falsity and that petitioners never took part in preparing the same. The "agreement" was
crafted without any consultation whatsoever or any attempt to arrive at mutually
HEIRS OF QUIRICO SERASPI AND PURIFICACION R. SERASPI, Petitioners, v. The Court of Appeals, while ruling that petitioners were able to establish the identity of
COURT OF APPEALS AND SIMEON RECASA, Respondents. G.R. No. 135602. the property as well as the credibility of their title ¾ the elements required to prove one’s
April 28, 2000. claim for recovery of property 2 ¾ nonetheless held that the action was barred by
prescription. Citing Arradaza v. Court of Appeals, it held that an action for recovery of
Marcelino Recasa was the owner of two parcels of land described as follows: title or possession of real property or an interest therein can only be brought within ten
(10) years after the cause of action has accrued. Since the action for recovery of
PARCEL I: A parcel of cocal land located at Barangay Lapnag, Banga, Aklan, with an
possession and ownership was filed by petitioners only on April 12, 1987, i.e., thirteen
area of 770 square meters,
(13) years after their predecessor-in-interest had been allegedly deprived of the
PARCEL II: A parcel of cocal land with an area of 3,648 square meters, more or less, possession of the property by private respondent, it was held that the action had
located in Barangay Lapnag, Banga, Aklan; prescribed.

During his lifetime, Marcelino contracted three (3) marriages. At the time of his death Arradaza involves acquisitive, not extinctive, prescription. What is more, the facts in
in 1943, he had fifteen (15) children from his three marriages. In 1948, his intestate that case arose before the effectivity of the Civil Code. Accordingly, what was applied
estate was partitioned into three parts by his heirs, each part corresponding to the share was the Code of Civil Procedure which provides that title by prescription is acquired
of the heirs in each marriage. after ten (10) years, in whatever manner possession may have been commenced or
continued, and regardless of good faith or with just title. On the other hand, what is
In the same year, Patronicio Recasa, representing the heirs of the first marriage, sold involved here is extinctive prescription, and the applicable law is Art. 1141 of the Civil
the share of the heirs in the estate to Dominador Recasa, an heir of the second Code which provides:
marriage. On June 15, 1950, Dominador, representing the heirs of the second
marriage, in turn sold the share of the heirs to Quirico and Purificacion Seraspi whose Real actions over immovables prescribe after thirty years.
heirs are the present petitioners. Included in this sale was the property sold by
This provision is without prejudice to what is established for the acquisition of
Patronicio to Dominador.
ownership and other real rights by prescription.
In 1958, the Seraspis obtained a loan from the Kalibo Rural Bank, Inc. (KRBI) on the
The question, therefore, is whether private respondent has acquired the
security of the lands in question to finance improvements on the lands. However, they
ownership of the two lands by prescription. On this point, the Civil Code
failed to pay the loan for which reason the mortgage was foreclosed and the lands were
provides:
sold to KRBI as the highest bidder. Subsequently, the lands were sold by KRBI to
Manuel Rata, brother-in-law of Quirico Seraspi. It appears that Rata, as owner of the ARTICLE 1117. Acquisitive prescription of dominion and other real rights may
property, allowed Quirico Seraspi to administer the property. be ordinary or extraordinary.
In 1974, private respondent Simeon Recasa, Marcelino’s child by his third wife, taking Ordinary acquisitive prescription requires possession of things in good faith
advantage of the illness of Quirico Seraspi, who had been paralyzed due to a stroke, and with just title for the time fixed by law.
forcibly entered the lands in question and took possession thereof.
ARTICLE 1134. Ownership and other real rights over immovable property are
In 1983, the Seraspis purchased the lands from Manuel Rata and afterwards filed a acquired by ordinary prescription through possession of ten years.
complaint against Simeon Recasa for recovery of possession of the lands.
ARTICLE 1137. Ownership and other real rights over immovables also
TC: ruled in favor of the Seraspis, stating that they had acquired the property through prescribe through uninterrupted adverse possession thereof for thirty years,
a sale and acquisitive prescription. without need of title or of good faith.
CA: reversed on the ground that the action of the Seraspis was barred by the statute Thus, acquisitive prescription of dominion and other real rights may be ordinary or
of limitations. extraordinary, depending on whether the property is possessed in good faith and with
just title for the time fixed by law. 4 Private respondent contends that he acquired the
Issues:
ownership of the questioned property by ordinary prescription through adverse
(1) whether petitioners’ action is barred by extinctive prescription; and possession for ten (10) years.

(2) whether private respondent Simeon Recasa acquired ownership of the properties The contention has no merit, because he has neither just title nor good faith. As Art.
in question through acquisitive prescription. 1129 provides:

Ruling: NO.
For the purposes of prescription, there is just title when the adverse claimant came into sale or assignment of the property, and the transferee can maintain such action against
possession of the property through one of the modes recognized by law for the the wrongdoer.
acquisition of ownership or other real rights, but the grantor was not the owner or could
not transmit any right.

In the case at bar, private respondent did not acquire possession of the property
through any of the modes recognized by the Civil Code, to wit: (1) occupation, (2)
intellectual creation, (3) law, (4) donation, (5) succession, (6) tradition in consequence
of certain contracts, and (7) prescription.

Private respondent could not have acquired ownership over the property through
occupation since, under Art. 714 of the Civil Code, the ownership of a piece of land
cannot be acquired by occupation. Nor can he base his ownership on succession for
the property was not part of those distributed to the heirs of the third marriage, to which
private respondent belongs. It must be remembered that in the partition of the intestate
estate of Marcelino Recasa, the properties were divided into three parts, each part
being reserved for each group of heirs belonging to one of the three marriages
Marcelino entered into. Since the contested parcels of land were adjudicated to the
heirs of the first and second marriages, it follows that private respondent, as heir of the
third marriage, has no right over the parcels of land. While, as heir to the intestate estate
of his father, private respondent was co-owner of all of his father’s properties, such co-
ownership rights were effectively dissolved by the partition agreed upon by the heirs of
Marcelino Recasa.

Neither can private respondent claim good faith in his favor. Good faith consists in the
reasonable belief that the person from whom the possessor received the thing was its
owner but could not transmit the ownership thereof. Private respondent entered the
property without the consent of the previous owner. For all intents and purposes, he is
a mere usurper.

Like private respondent, petitioners have not acquired the property through any of the
modes recognized by law for the acquisition of ownership. The basis of petitioners’
claim of ownership is the contract of sale they had with Rata, but this by itself is
insufficient to make them owners of the property. For while a contract of sale is
perfected by the meeting of minds upon the thing which is the object of the contract and
upon the price, the ownership of the thing sold is not transferred to the vendee until
actual or constructive delivery of the property. Hence, the maxim non nudis pactis, sed
traditione dominia dominica rerum transferuntur (not mere agreements but tradition
transfers the ownership of things).

Consequently, petitioners are not the owners of the property since it has not been
delivered to them. At the time they bought the property from Rata in 1983, the property
was in the possession of private Respondent.

However, this does not give private respondent a right to remain in possession of the
property. Petitioners’ title to the property prevails over private respondents’ possession
in fact but without basis in law. As held in Waite v. Peterson, 9 when the property
belonging to a person is unlawfully taken by another, the former has the right of action
against the latter for the recovery of the property. Such right may be transferred by the
DOMINGO R. CATAPUSAN, MINELIO R. CATAPUSAN, and FILOMENO R. ownership. The court must initially settle the issue of ownership, the first stage
CATAPUSAN, petitioners, vs. THE COURT OF APPEALS, VICENTE CATAPUSAN, in an action for partition. Needless to state, an action for partition will not lie if
JR., CIPRIANO CATAPUSAN, et.al, respondents. G.R. No. 109262 November the claimant has no rightful interest over the subject property. In fact, Section
21, 1996 1 of Rule 69 requires the party filing the action to state in his complaint the
"nature and extent of his title" to the real estate. Until and unless the issue of
FACTS: Bonifacio Catapusan was first married to Narcissa Tanjuatco, the only ownership is definitely resolved, it would be premature to effect a partition of
surviving heir of Dominga Piguing. 2 They had four (4) children namely, Felix, Vicente, the properties. Hence, on the first issue we rule in the affirmative.
Benicio and Loreto. 3 Narcissa died in 1910. In 1927, Bonifacio married Paula Reyes
and out of their wedlock petitioners Domingo, Minelio and Filomeno Catapusan were 2. Anent the second and third issues, it is a basic rule of evidence that the party
born. Bonifacio died in 1940. 4 Felix, Vicente and Benicio, Bonifacio's sons from the making an allegation has the burden of proving it by preponderance of
first marriage, died before the institution of this case, survived by their respective evidence.
widows and children, respondents herein.
In this case, petitioners' evidence of their father's (Bonifacio) ownership of the Wawa
The petitioners filed on June 11, 1974, an action for partition of the Wawa lot, which lot are the tax declarations of the adjacent lot owners and the testimonies of some
they allegedly co-own with their half-brothers and half-sisters. Petitioners contend that witnesses who merely saw Bonifacio working on the lot. On the other hand,
the said lot belongs to their father Bonifacio and should therefore be partitioned among respondents presented tax declarations which indicated that the same lot is owned by
the heirs of the first and second marriages. 6 In support thereof, they presented the tax their predecessors-in-interest, the children of the first marriage, evidence which carry
declarations of the Wawa lot's four (4) adjacent lot owners. These four tax declarations more weight as they constitute proof of respondents' ownership of the land in their
state that each of them bounds on one side the Wawa lot declared in the name of possession. The statement in the neighboring lot owners' tax declarations is not a
Bonifacio. Stated differently, the petitioner's proof of Bonifacio's ownership of the Wawa conclusive proof that Bonifacio owned the surrounded lot. In fact, petitioners cannot
lot are the tax declarations of the adjoining lot owners which noted that they each border show any tax receipts or declarations of their ownership over the same lot. Although
on one side the Wawa lot declared in the name of Bonifacio. tax declarations and receipts are not direct proofs of ownership, yet when accompanied
by proof of actual possession for the required period, they become strong evidence to
In their answer with counterclaim, respondents asserted that the Wawa lot was
support the claim of ownership thru acquisitive prescription. The possession
originally owned by Dominga and inherited by Narcissa as her paraphernal property.
contemplated as foundation for prescriptive right must be one under claim of title or
Upon Narcissa's death, the Wawa lot passed to her four children who are the
adverse to or in concept of owner. Possession by tolerance, as in the case of
predecessor-in-interest of respondents. These children possessed and occupied the
petitioners, is not the kind of possession that may lead to title by prescription. It is the
Wawa lot and secured tax declarations thereon in their names. Respondents likewise
respondents' open, continuous, adverse and uninterrupted possession far beyond the
alleged that they had been in open, continuous and uninterrupted possession of the
30 year extraordinary period for acquisitive prescription, coupled with the tax
said lot for more than 50 years when the suit was filed in 1974. 10 They also invoke
declarations of their predecessors-in-interest, that constitutes a superior weight of
laches and prescription against petitioner's action. In response, petitioners argue that
evidence that clinched their claim. Moreover, petitioners bare and unsubstantiated
their action had not lapsed since respondents repudiated the co-ownership only in
allegation that respondents' tax declarations were fraudulently issued is insufficient to
1968. 11 They also questioned the respondents' lack of documentary proof (like "titulo
sustain the imputation of fraud considering that good faith is always presumed. Besides,
real") with regard to Dominga and Narcissa's title, as the two lived during the Spanish
respondents' tax declarations are deemed regularly issued. Being an action involving
era.
property, the petitioners must rely on the strength of their own title and not on the
TC: dismissed the complaint, declared there respondents as the true and lawful owners weakness of the respondents' claim.
of the Wawa lot
In any event, the second and third issues pertain to factual findings of the courts below.
CA: affirmed It is a settled doctrine that factual findings of the lower court when supported by
substantial evidence on the record is not usually reviewed by the Supreme Court,
Issue: especially when it is affirmed by the Court of Appeals, as in this case. No cogent
evidence appears from the records of this case for us to apply the above doctrine
(1) whether an action for partition includes the question of ownership; -YES
differently. No essential facts were overlooked by the courts below, which if considered,
(2) whether Bonifacio had title to the Wawa lot, and- NO
may produce a different outcome. Besides, the credence of the evidence and the
(3) whether petitioner's action is barred by laches and/or prescription.-
assessment of the weight and evidentiary value of the testimonies presented are best
Ruling:
appreciated by the trial court judge having observed that elusive and incommunicable
1. In actions for partition, the court cannot properly issue an order to divide the evidence of the witness' deportment on the stand.
property, unless it first makes a determination as to the existence of co-
ZOSIMA VERDAD, petitioner, vs. THE HON. COURT OF APPEALS, SOCORRO C. right to the property is not because she rightfully can claim heirship in Macaria's
ROSALES, AURORA ROSALES, NAPOLEON ROSALES, ANTONIO ROSALES, estate but that she is a legal heir of her husband, David Rosales, part of whose
FLORENDA ROSALES, ELENA ROSALES AND VIRGINIA ROSALES, estate is a share in his mother's inheritance.
respondents. G.R. No. 109972 April 29, 1996
David Rosales, incontrovertibly, survived his mother's death. When Macaria died on 08
FACTS: The petitioner, Zosima Verdad, is the purchaser of a 248-square meter March 1956 her estate passed on to her surviving children, among them David Rosales,
residential lot (identified to be Lot No. 529, Ts-65 of the Butuan Cadastre, located along who thereupon became co-owners of the property. When David Rosales himself later
Magallanes Street, now Marcos M. Calo St., Butuan City). Private respondent, Socorro died, his own estate, which included his undivided interest over the property inherited
Cordero Vda. de Rosales, seeks to exercise a right of legal redemption over the subject from Macaria, passed on to his widow Socorro and her co-heirs pursuant to the law on
property and traces her title to the late Macaria Atega, her mother-in-law, who died succession.
intestate on 08 March 1956.
Art. 995. In the absence of legitimate descendants and ascendants, and
During her lifetime, Macaria contracted two marriages: the first with Angel Burdeos and illegitimate children and their descendants, whether legitimate or illegitimate,
the second, following the latter's death, with Canuto Rosales. At the time of her own the surviving spouse shall inherit the entire estate, without prejudice to the
death, Macaria was survived by her son Ramon A. Burdeos and her grandchild (by her rights of brothers and sisters, nephews and nieces, should there be any, under
daughter Felicidad A. Burdeos) Estela Lozada of the first marriage and her children of article 1001.
the second marriage, namely, David Rosales, Justo Rosales, Romulo Rosales, and
Aurora Rosales. Art. 1001. Should brothers and sisters or their children survive with the widow
or widower, the latter shall be entitled to one-half of the inheritance and the
Socorro Rosales is the widow of David Rosales who himself, some time after Macaria's brothers and sisters or their children to the other half.
death, died intestate without an issue.
Socorro and herein private respondents, along with the co-heirs of David Rosales,
In an instrument, dated 14 June 1982, the heirs of Ramon Burdeos, namely, his widow thereupon became co-owners of the property that originally descended from Macaria.
Manuela Legaspi Burdeos and children Felicidad and Ramon, Jr., sold to petitioner
Zosima Verdad (their interest on) the disputed lot supposedly for the price of When their interest in the property was sold by the Burdeos heirs to petitioner, a right
P55,460.00. In a duly notarized deed of sale, dated 14 November 1982, it would of redemption arose in favor of private respondents; thus:
appear, however, that the lot was sold for only P23,000.00. Petitioner explained that
Art. 1619. Legal redemption is the right to be subrogated, upon the same
the second deed was intended merely to save on the tax on capital gains.
terms and conditions stipulated in the contract, in the place of one who
Socorro discovered the sale on 30 March 1987 while she was at the City Treasurer's acquires a thing by purchase or dation in payment, or by any other transaction
Office. On 31 March 1987, she sought the intervention of the Lupong Tagapayapa of whereby ownership is transmitted by onerous title.
Barangay 9, Princess Urduja, for the redemption of the property. She tendered the sum
Art. 1620. A co-owner of a thing may exercise the right of redemption in case
of P23,000.00 to Zosima. The latter refused to accept the amount for being much less
the shares of all the other co-owners or of any of them, are sold to a third
than the lot's current value of P80,000.00. No settlement having been reached before
person. If the price of the alienation is grossly excessive, the redemptioner
the Lupong Tagapayapa, private respondents, on 16 October 1987, initiated against
shall pay only a reasonable one.
petitioner an action for "Legal Redemption with Preliminary Injunction" before the
Regional Trial Court of Butuan City. We hold that the right of redemption was timely exercised by private
respondents. Concededly, no written notice of the sale was given by the Burdeos
TC: private respondents' right to redeem the property had already lapsed.
heirs (vendors) to the co-owners required under Article 1623 of the Civil Code —
CA: Reversed RTC-declaring plaintiff-appellant, Socorro C. Rosales, entitled to
Art. 1623. The right of legal pre-emption or redemption shall not be exercised
redeem the inheritance rights (Art. 1088, NCC) or pro indiviso share (Art. 1620, NCC)
except within thirty days from the notice in writing by the prospective vendor,
of the Heirs of Ramon Burdeos.
or by the vendor, as the case may be. The deed of safe shall not be recorded
Issue: W/N Socorro C. Rosales is capacitated to redeem the property, she being in the Registry of Property, unless accompanied by an affidavit of the vendor
merely the spouse of David Rosales, a son of Macaria, and not being a co-heir herself that he has given written notice thereof to all possible redemptioners.
in the intestate estate of Macaria.
Hence, the thirty-day period of redemption had yet to commence when private
RULING: YES..It is true that Socorro, a daughter-in-law (or, for that matter, a mere respondent Rosales sought to exercise the right of redemption on 31 March 1987, a
relative by affinity), is not an intestate heir of her parents-in-law, however, Socorro's day after she discovered the sale from the Office of the City Treasurer of Butuan City,
or when the case was initiated, on 16 October 1987, before the trial court.
The written notice of sale is mandatory. This Court has long established the rule that
notwithstanding actual knowledge of a co-owner, the latter is still entitled to a written
notice from the selling co-owner in order to remove all uncertainties about the sale, its
terms and conditions, as well as its efficacy and status.

Even in Alonzo vs. Intermediate Appellate Court, relied upon by petitioner in contending
that actual knowledge should be an equivalent to a written notice of sale, the Court
made it clear that it was not reversing the prevailing jurisprudence; said the Court:

We realize that in arriving at our conclusion today, we are deviating from the
strict letter of the law, which the respondent court understandably applied
pursuant to existing jurisprudence. The said court acted properly as it had no
competence to reverse the doctrines laid down by this Court in the above-
cited cases. In fact, and this should be clearly stressed, we ourselves are not
abandoning the De Conejero and Buttle doctrines. What we are doing simply
is adopting an exception to the general rule, in view of the peculiar
circumstances of this case.

In Alonzo, the right of legal redemption was invoked several years, not just
days or months, after the consummation of the contracts of sale. The
complaint for legal redemption itself was there filed more than thirteen years
after the sales were concluded.

Relative to the question posed by petitioner on private respondents' tender of


payment, it is enough that we quote, with approval, the appellate court; viz.:

In contrast, records dearly show that an amount was offered, as required in


Sempio vs. Del Rosario, 44 Phil. 1 and Daza vs. Tomacruz, 58 Phil. 414, by
the redemptioner-appellant during the barangay conciliation proceedings
(Answer, par. 8) but was flatly rejected by the appellee, not on the ground that
it was not the purchase price (though it appeared on the face of the deed of
sale, Exh. "J-1"), nor that it was offered as partial payment thereof, but rather
that it was

All given, we find no error in the appellate court's finding that private respondents are
entitled to the redemption of the subject property.
ANDREA TABUSO and RENATO BISMORTE, petitioners, vs. COURT OF The trial court likewise gave credit to the testimony of Atty. Jose Gonzales, private
APPEALS and the HEIRS OF ESTEBAN ABAD represented by Nemesio Abad and respondents' counsel who had been presented by petitioners as their own witness. He
Ana Abad Paghubasan, respondents. [G.R. No. 108558. June 21, 2001] testified that the land in question, which was adjacent to the land he himself possessed,
had been in the possession of Esteban Abad's heirs, herein private respondents. The
FACTS
trial court also took note of the various tax declarations covering the property, indicating
This case involves declaration of ownership filed before the RTC of Naval, Leyte, [in] that it was owned by private respondents.
Biliran, Leyte, of an unregistered parcel of land at Antipolo, Naval, Leyte with an
area of 3,267 square meters.
CA: upheld the findings of the trial court. After a careful review of the records, [w]e
PRIVATE RESPONDENTS’ evidence consists of the following:
agree with the trial court that the preponderance of evidence supports the claim
"a) A tax declaration No. 3705 (Exh. A) in the name of Ignacio Montes for the year 1912. of ownership of defendants-appellees.
However, the land taxes thereon for the years 1944 to 1947 were paid only in 1981
ISSUE: Who owns the property in dispute?
(Exh. F and series).
RULING: The totality of the evidence presented leans heavily in favor of herein private
"b) Plaintiff Andrea Tabuso claims to be the owner as successor in interest
respondents.
(granddaughter) of one Andrea Elaba, daughter of Maria Montes and Borja Elaba,
Maria Montes appears to be a sister of Ignacio Montes, in whose name the tax
declaration for the property in question was issued for the year 1912 (Exh. A).
"c) The property in question has been in the possession of the defendants (heirs of
Main Issue:
Esteban Abad), although the house standing thereon appears to have been
Ownership of the Property
constructed by Marcelo Tabuso, father of plaintiff Andrea Tabuso.
We agree with the findings of the Court of Appeals that for a period of more than 60
years, private respondents have been able to establish that they are the owners
PETITIONERS tend to establish the following: of the lot; and that for said period, they have been in open, continuous and
uninterrupted possession of the same.
"a) The land in question originally owned by Maria Montes was donated to Isabel Elaba
through an ancient document executed on September 24, 1923 (Exh. F). Isabel in turn Both the trial and the appellate courts were likewise correct in giving weight to the
sold the land to Esteban Abad on May 5, 1948 (Exh. 4). testimony of Atty. Jose Gonzales. He testified that being, the owner of the adjacent
land, he had personal knowledge of the simple fact that the land in question was owned
"b) The original tax declaration in the name of Ignacio Montes (Exh. A) was superseded
by private respondents, who were in actual, open and continuous possession thereof.
by Tax Declaration Nos. 6422 and 1450 both in the name of Isabel Elaba (Exh. 6-D; 6-
Significantly, while he was private respondents' counsel, he was presented by
E)[;] Declaration No. 1450 for the year 1948 was superseded by Tax Declaration No.
petitioners themselves. Having done so, they are bound by his testimony, even if it is
6959 for 1960 (Exh. 6-C) in the name of Esteban Abad; and the latter was superseded
hostile.
in 1969 by Tax Declaration No. 1661 (Exh. 6-B) in the name of Esteban Abad. In 1974
a new tax declaration No. 19 (Exh. 6-A) was issued in the name of Esteban Abad with The only substantial argument of petitioners supporting their claim of ownership
Nemesio Abad and his co-heirs as administrators. The last tax declaration No. 22 (Exh. is their construction of a small house (barong-barong) on the property, as
6) for 1982 was in the name of Esteban Abad. The land taxes due thereon for the years acknowledged in private respondents' letter, which reads:
1947 to 1982 were paid by Isabel Elaba[,] Esteban Abad and Nemesio Abad (Exhs. 7
"Notice to Vacate
to 7-W).
To: Mr. & Mrs. Renato Bismorte
"c) The land in question is tenanted by one Valentin Poblete in accordance with a lease
Barangay Calumpang
contract executed by defendant Nemesio [Abad], one of the heirs and co-owners of the
Naval, Leyte
land.

Greetings:
RTC concluded that there was abundant proof of private respondents' ownership
of the lot in question as against the scanty evidence offered by petitioners. And even You are advised to vacate the area/lot where your 'Barong-Barong House'
if the latter had built a house thereon, such action was only tolerated by private [was] temporarily constructed for we, the lawful owners, shall have to use
respondents, who had originally allowed one Marcelo Tabuso (father of Petitioner it. You are given three (3) months grace period upon receipt thereof within
Andrea Tabuso), to construct a house on the same lot. Besides, Petitioner Tabuso is which to transfer or completely vacate the area/lot.
not a compulsory heir of Ignacio Montes, from whom she claims to have inherited the
[Should there be f]ailure to comply [with] this notice or advise [,] an ejectment
lot, subject of this litigation. In addition, the tax declaration in his name has long been
proceeding shall be instituted or filed against you before the proper court.
revised.
Hence, compliance is hereby desired.
hand, the entire lot that was adjudicated measures 11,927 square meters. Petitioners'
contention deserves scant consideration, because they have not substantiated, by any
(Signed) Mr. NEMESIO E. ABAD
means whatsoever, their claim to any part of the disputed land. Hence, they are not
Co-Owner
entitled to ownership thereof.
Besides, what defines a piece of land is not the numerical data indicated as its
Obviously, the claim of private respondents that they are the owners of the land is area, but the boundaries or "metes and bounds" specified in its description as
supported by the above letter, in which they were asking petitioners to vacate the enclosing the land and indicating its limits.
property. Moreover, considering its size, which is 11,927 square meters as found
WHEREFORE, the petition is hereby DISMISSED and the assailed Decision and
by the court-appointed commissioner, the fact that petitioners' house is only
Resolution AFFIRMED. Costs against petitioners. SO ORDERED.
a barong-barong or make-shift shanty lends support to private respondents'
claim that the former's presence on the property was merely tolerated.
It must be stressed "that possession and ownership are distinct legal concepts.
• Ownership exists when a thing pertaining to one person is completely subjected
to his will in a manner not prohibited by law and consistent with the rights of
others.

- Ownership confers certain rights to the owner, one of which is the right to dispose
of the thing by way of sale.
• Possession is defined as the holding of a thing or the enjoyment of a right.
- Literally, to possess means to actually and physically occupy a thing with or
without right.

- Possession may be had in one of two ways:


1. Possession in the concept of an owner - Possessors in the concept of
owners may be the owners themselves or those who claim to be so.
2. Possession of a holder - those who possess as mere holders
acknowledge in another a superior right which he believes to be
ownership, whether his belief be right or wrong." 13
In this case, the evidence shows that the occupation of the property by
petitioners is not in the concept of owners, because their stay is merely tolerated.
This finding is bolstered by the fact that Petitioner Andrea Tabuso is the daughter of
Marcelo Tabuso, who was merely allowed by the previous owner, Esteban Abad, to
construct a small house on the lot. An owner's act of allowing another to occupy
his house, rent-free[,] does not create a permanent and indefeasible right of
possession in the latter's favor."
Lastly, the claim of petitioners that private respondents are not in actual possession of
the land is unsubstantiated. Besides, it is not necessary that the latter actually stay
on the property in order to prove ownership of the same. As found by both the trial
and the appellate courts, since the acquisition of the subject property by private
respondents, they had religiously paid the taxes due thereon. Further, one of the co-
owners executed a lease contract over it in favor of a tenant. These acts are clearly
consistent with ownership.

Size of Lot Immaterial To Private Respondents' Claim of Ownership


PETITIONERS argue that private respondents own only 3,267 square meters of the
questioned lot. This is the area that appears on their Tax Declarations. On the other
PAUL P. GABRIEL, JR., IRENEO C. CALWAG, THOMAS L. TINGGA-AN, and the because such defense was a collateral attack which was prohibited under P.D. No.
Heirs of JULIET B. PULKERA, Petitioners, vs. CARMELING 1529, otherwise known as the Property Registration Decree. Thus, it could not inquire
CRISOLOGO, Respondent. [G.R. No. 204626. June 9, 2014] into the intrinsic validity of Crisologo’s titles.

FACTS RTC reversed and set aside the decision of the MTCC. It was of the view that
petitioners’ assertion of the TCTs’ invalidity was not a collateral attack. The titles
Records show that Carmeling Crisologo (Crisologo), represented by her attorney-in-
of Crisologo were products of illegal proceedings nullified by this Court. She also
fact, Pedro Isican (Isican), filed her complaint for Recovery of Possession and/or
failed to comply with the conditions set forth in P.D. No. 1271. Accordingly, the
Ownership with Damages against Juliet B. Pulkera, Paul P. Gabriel, Ireneo C.
titles were void and the same could not be a legal basis for Crisologo to justify the
Calwag, and Thomas L. Tingga-an (petitioners) before the MTCC.
eviction of petitioners from the subject premises. Having been nullified, these
CRISOLOGO alleged, among others, that she was the registered owner of two certificates of title ceased to be the best proof of ownership.
parcels of land with a total area of approximately 2,000 square meters, described
CA rendered the assailed decision, setting aside the RTC decision and reinstating that
in, and covered by, two (2) certificates of title – Transfer Certificate of Title (TCT)Nos.
of the MTCC.
T-13935 and T-13936;
The CA held that Crisologo was entitled to the possession of the subject parcels
• that sometime in 2006, she discovered that petitioners unlawfully entered, of land. It explained that her possession was established when she acquired the
occupied her properties by stealth, by force and without her prior consent
same by sale sometime in 1967 and when the certificates of title covering the
and knowledge, and constructed their houses thereon; that upon discovery of
properties were subsequently issued. It added that her payment of realty taxes due
their illegal occupation, her daughter, Atty. Carmelita Crisologo, and Isican
on the said properties since 1969 further strengthened her claim of possession.
personally went to the properties and verbally demanded that petitioners vacate
Moreover, her appointment of Isican as administrator of the subject properties and her
the premises and remove their structures thereon; that the petitioners begged
offer to sell the lots to the petitioners showed that she had control over the same.
and promised to buy the said properties for 3,500.00 per square meter; that
she gave petitioners time to produce the said amount, but they reneged on their ISSUE who between petitioners and respondent Crisologo have a better right of
promise to buy them; that petitioners refused to vacate the subject properties possession over the subject parcels of land.
despite several demands; that the petitioners knew full well that the subject
RULING: Crisologo has a better right of possession over the subject parcels of land.
premises they were occupying were titled properties but they insisted on unlawfully
holding the same; and that she was unlawfully dispossessed and displaced from Accion Publiciana: its nature and purpose
the subject properties due to petitioners’ illegal occupation.
Also known as accion plenaria de posesion, accion publiciana is an ordinary civil
PETITIONERS countered that the titles of Crisologo were products of Civil proceeding to determine the better right of possession of realty independently of title. It
Registration Case No. 1, Record 211, which were declared void by the Supreme refers to an ejectment suit filed after the expiration of one year from the accrual
Court in Republic v. Marcos;7 that the said case was later enacted into law, Presidential of the cause of action or from the unlawful withholding of possession of the
Decree (P.D.)No. 1271, entitled "An Act Nullifying Decrees of Registration and realty.
Certificates of Title within the Baguio Town site Reservation Case No.1, GLRO Record
The objective of the plaintiffs in accion publiciana is to recover possession only,
No. 211, pursuant to Act No. 931, as amended, but Considering as Valid Certain Titles not ownership. When parties, however, raise the issue of ownership, the court may
of Lands that are Alienable and Disposable Under Certain Conditions and For Other
pass upon the issue to determine who between the parties has the right to possess the
Purposes" which took effect on December 22, 1977; that Crisologo failed to comply
property. This adjudication, nonetheless, is not a final and binding determination of the
with the conditions provided in Section 1 of P.D. No. 1271 for the validation of
issue of ownership; it is only for the purpose of resolving the issue of possession, where
said titles, hence, the titles were void; that petitioners had been in open, actual,
the issue of ownership is inseparably linked to the issue of possession. The adjudication
exclusive, notorious, uninterrupted, and continuous possession of the subject
of the issue of ownership, being provisional, is not a bar to an action between the same
land, in good faith; and that Crisologo was never in prior possession and had no valid
parties involving title to the property. The adjudication, in short, is not conclusive on the
title over the subject land.8
issue of ownership.12
CRISOLOGO prayed that she be declared in prior actual possession of the properties
MTCC rendered a decision in favor of Crisologo, It ruled that Crisologo was the in dispute and that petitioners vacate the same and demolish their houses therein. She
registered owner of the subject parcels of land, who, as such, had declared these alleged, among others, that she was the registered owner of the subject parcels of land
properties for taxation purposes since 1969 and regularly paid the realty taxes thereon. and that petitioners unlawfully entered her properties by stealth, force and without her
It stated that with Crisologo being the owner, petitioners were illegally occupying the prior consent and knowledge. Clearly, she primarily wanted to recover possession of
land. the subject parcels of land from petitioners. Hence, the case is an accion publiciana.
The MTCC added that petitioners could not question Crisologo’s titles over the
subject parcels of land in an ordinary civil action for recovery of possession
PETITIONERS mainly argue that Crisologo’s titles on the subject properties are void Property Registration Decree, provides that a certificate of title cannot be the subject of
and that they have been in open, actual, exclusive, notorious, uninterrupted and a collateral attack. Thus:
continuous possession over the subject properties in good faith.
SEC. 48. Certificate not subject to collateral attack. – A certificate of title shall
not be subject to collateral attack. It cannot be altered, modified, or canceled
except in a direct proceeding in accordance with law.
The nullity of the decrees of registration and certificates of titles in Section 1 of
P.D. No. 1271 is not absolute Registration of land under the Torrens system, aside from perfecting the title and
rendering it indefeasible after the lapse of the period allowed by law, also renders
Although Section 1 of P.D. No. 1271 invalidated decrees of registration and certificates
the title immune from collateral attack. A collateral attack transpires when, in another
of title within the Baguio Town site Reservation Case No. 1, GLRO Record No. 211,
action to obtain a different relief and as an incident of the present action, an attack is
the nullity, however, is not that sweeping. The said provision expressly states that "all
made against the judgment granting the title.
certificates of titles issued on or before July 31, 1973 shall be considered valid
and the lands covered by them shall be deemed to have been conveyed in fee This manner of attack is to be distinguished from a direct attack against a judgment
simple to the registered owners" upon: granting the title, through an action whose main objective is to annul, set aside, or enjoin
the enforcement of such judgment if not yet implemented, or to seek recovery if the
1) showing proof that the land covered by the subject title is not within any
property titled under the judgment had been disposed of. To permit a collateral attack
government, public or quasi-public reservation, forest, military or otherwise, as
on respondents-plaintiffs' title is to water down the integrity and guaranteed legal
certified by appropriating government agencies; and
indefeasibility of a Torrens title.
2) compliance by the titleholder with the payment to the Republic of the Philippines of
The petitioners-defendants' attack on the validity of respondents-plaintiffs' title,
the correct assessed value of the land within the required period.
by claiming that fraud attended its acquisition, is a collateral attack on the title.
In the case at bench, the records show that the subject parcels of land were It is an attack incidental to their quest to defend their possession of the properties in an
registered on August 24, 1967. The titles are, thus, considered valid although subject "accion publiciana," not in a direct action whose main objective is to impugn the validity
to the conditions set. But whether or not Crisologo complied with the said of the judgment granting the title. This is the attack that possession of a Torrens
conditions would not matter because, this would be a collateral attack on her Title specifically guards against; hence, we cannot entertain, much less accord credit
registered titles, as would be discussed later. to, the petitioners-defendants' claim of fraud to impugn the validity of the respondents-
plaintiffs' title to their property.
At any rate, petitioners, as private individuals, are not the proper parties to
question the status of the respondent’s registered titles. Section 6 of P.D. No.
127114 expressly states that the "Solicitor General shall institute such actions or suits
As the lawful possessor. the respondent has the right to eject the petitioners
as may be necessary to recover possession of lands covered by all void titles not
validated under this Decree." The foremost relevant issue that needs to be determined here is simply possession,
not ownership.
The testimonial and documentary evidence on record prove that Crisologo has a
The respondent’s certificates of title give her the better right to possess the
preferred claim of possession over that of petitioners. It cannot be denied that she
subject parcels of land
bought the subject properties from the previous owner in 1967, which was why the
It is settled that a Torrens title is evidence of indefeasible title to property in favor transfer certificates of title were subsequently issued in her name. Records further
of the person in whose name the title appears. It is conclusive evidence with respect show that she has been paying the realty taxes on the said properties since 1969.
to the ownership of the land described therein. It is also settled that the titleholder is She likewise appointed Isican as administrator of the disputed lands. More
entitled to all the attributes of ownership of the property, including possession. importantly, there is no question that she offered to sell to petitioners the portions
The "age-old rule is that the person who has a Torrens title over a land is entitled of the subject properties occupied by them. Hence, she deserves to be respected
to possession thereof." and restored to her lawful possession as provided in Article 539 of the New Civil Code.20
The records show that TCT No. T-1393517 and TCT No. T-1393618 bear the name WHEREFORE, the petition is DENIED.
of Carmeling P. Crisologo, as the registered owner. Petitioners do not dispute
the fact that she has a Torrens title over the subject parcels of land.

The respondent’s Torrens certificates of title are immune from a collateral


attack.
As a holder of a Torrens certificate of title, the law protects Crisologo from a
collateral attack on the same. Section 48 of P.D. No. 1529, otherwise known as the
FERNANDA MENDOZA CEQUEÑA and RUPERTA MENDOZA which are evidence of the best kind of circumstance proving the claim of the title of
LIRIO, petitioners, vs. HONORATA MENDOZA BOLANTE, respondent. [G.R. No. ownership and enjoys the presumption of preferred possessor.
137944. April 6, 2000]

RULING: CA did not err.


Tax receipts and declarations are prima facie proofs of ownership or possession
Preference of Possession
of the property for which such taxes have been paid. Coupled with proof of actual
possession of the property, they may become the basis of a claim for ownership. By The CA ruled that the respondent was the preferred possessor under Article 538
acquisitive prescription, possession in the concept of owner — public, adverse, of the Civil Code because she was in notorious, actual, exclusive and continuous
peaceful and uninterrupted — may be converted to ownership. On the other hand, mere possession of the land since 1985. Petitioners dispute this ruling. They contend
possession and occupation of land cannot ripen into ownership. that she came into possession through force and violence, contrary to Article 536 of the
Civil Code.
We concede that despite their dispossession in 1985, the petitioners did not lose legal
FACTS
possession because possession cannot be acquired through force or violence. To all
The Petition herein refers to a parcel of land situated in Barangay Bangad, Binangonan, intents and purposes, a possessor, even if physically ousted, is still deemed the legal
Province of Rizal, having an area of 1,728 square meters and covered by Tax possessor. Indeed, anyone who can prove prior possession, regardless of its
Declaration No. 26-0027. character, may recover such possession.
Prior to 1954, the land was originally declared for taxation purposes in the name of However, possession by the petitioners does not prevail over that of the
Sinforoso Mendoza, father of [respondent] and married to Eduarda Apiado. Sinforoso respondent. Possession by the petitioner before 1985 was not exclusive, as the
died in 1930. respondent also acquired it before 1985. The records show that the petitioners'
father and brother, as well as the respondent and her mother were simultaneously in
PETITIONERS were the daughters of Margarito Mendoza. On the basis of an
adverse possession of the land.
affidavit, the tax declaration in the name of Sinforoso Mendoza of the contested
lot was cancelled and subsequently declared in the name of Margarito Mendoza. Before 1985, the subject land was occupied and cultivated by the respondent's father
Margarito and Sinforoso are brothers. (Sinforoso), who was the brother of petitioners' father (Margarito), as evidenced by Tax
Declaration No. 26425. When Sinforoso died in 1930, Margarito took possession
[Respondent] is the present occupant of the land. Earlier, on October 15, 1975,
of the land and cultivated it with his son Miguel. At the same time, respondent
[respondent] and Miguel Mendoza, another brother of [petitioners], during the cadastral
and her mother continued residing on the lot.
survey had a dispute on [the] ownership of the land.
When respondent came of age in 1948, she paid realty taxes for the years 1932-
1948. 16 Margarito declared the lot for taxation in his name in 1953 17 and paid its realty
TRIAL COURT rendered its judgment in favor of the PETITIONERS taxes beginning 1952. 18 When he died, Miguel continued cultivating the land. As found
by the CA, the respondent and her mother were living on the land, which was
CA reversed the trial court because the genuineness and the due execution of the
being tilled by Miguel until 1985 when he was physically ousted by the
affidavit allegedly signed by the respondent and her mother had not been
respondent.
sufficiently established. The notary public or anyone else who had witnessed the
execution of the affidavit was not presented. No expert testimony or competent witness Based on Article 538 of the Civil Code, the respondent is the preferred possessor
ever attested to the genuineness of the questioned signatures. because, benefiting from her father's tax declaration of the subject lot since 1926,
she has been in possession thereof for a longer period. On the other hand,
• The affidavit was insufficient to overcome the denial of respondent and her
petitioners' father acquired joint possession only in 1952.
mother. The former testified that the latter, never having attended school, could
neither read nor write. Respondent also said that she had never been called Possession of Better Right
"Leonor," which was how she was referred to in the affidavit.
PETITIONER challenge the CA ruling that "actual and physical coupled with the
Moreover, the appellate court held that the probative value of petitioners' tax exclusive and continuous possession [by respondent] of the land since 1985" proved
receipts and declarations paled in comparison with respondent's proof of her ownership of the disputed land.
ownership of the disputed parcel. Actual, physical, exclusive and continuous
possession by respondent since 1985 indeed gave her a better title under Article 538
of the Civil Code. RESPONDENT argues that she was legally presumed to possess the subject land with
a just title since she possessed it in the concept of owner. Under Article 541 of the
Hence, this Petition.
Code, she could not be obliged to show or prove such title.
ISSUE: W/N CA erred in holding that respondent has been in actual and physical
The respondent's contention is untenable.
possession, coupled with exclusive and continuous possession of the land since 1985,
The presumption in Article 541 of the Civil Code is merely disputable; it prevails
until the contrary is proven. 20 That is, one who is disturbed in one's possession
shall, under this provision, be restored thereto by the means established by law.
Article 538 settles only the question of possession, and possession is different
from ownership. Ownership in this case should be established in one of the ways
provided by law.
To settle the issue of ownership, we need to determine who between the claimants has
proven acquisitive prescription. 22
Ownership of immovable property is acquired by ordinary prescription through
possession for ten years. Being the sole heir of her father, respondent showed
through his tax receipt that she had been in possession of the land for more than ten
years since 1932. When her father died in 1930, she continued to reside there with her
mother. When she got married, she and her husband engaged in kaingin inside the
disputed lot for their livelihood. 24
Respondent's possession was not disturbed until 1953 when the petitioners'
father claimed the land. But by then, her possession, which was in the concept
of owner — public, peaceful, and uninterrupted 25 — had already ripened into
ownership.
Furthermore she herself, after her father's demise, declared and paid realty taxes for
the disputed land. Tax receipts and declarations of ownership for taxation, when
coupled with proof of actual possession of the property, can be the basis of a
claim for ownership through prescription.
In contrast, the petitioners, despite thirty-two years of farming the subject land,
did not acquire ownership. It is settled that ownership cannot be acquired by mere
occupation. Unless coupled with the element of hostility toward the true
owner, occupation and use, however long, will not confer title by prescription or
adverse possession. Moreover, the petitioners cannot claim that their possession was
public, peaceful and uninterrupted. Although their father and brother arguably acquired
ownership through extraordinary prescription because of their adverse possession for
thirty-two years (1953-1985), this supposed ownership cannot extend to the entire
disputed lot, but must be limited to the portion that they actually farmed.
We cannot sustain the petitioners' contention that their ownership of the disputed land
was established before the trial court through the series of tax declarations and receipts
issued in the name of Margarito Mendoza. Such documents prove that the holder has
a claim of title over the property. Aside from manifesting a sincere desire to obtain title
thereto, they announce the holder's adverse claim against the state and other
interested parties. 30
However, tax declarations and receipts are not conclusive evidence of
ownership. 31 At most, they constitute mere prima facie proof of ownership or
possession of the property for which taxes have been paid. In the absence of
actual public and adverse possession, the declaration of the land for tax
purposes does not prove ownership. In sum, the petitioners' claim of ownership of
the whole parcel has no legal basis.
WHEREFORE, the Petition is DENIED and the assailed Decision and Resolution
AFFIRMED. Costs against petitioners. SO ORDERED.
SUBIC BAY LEGEND RESORTS AND CASINOS, INC., Petitioner, vs. BERNARD although he was quite aware of the consequences of a crime such as direct assault
C. FERNANDEZ, Respondent. [G.R. No. 193426. September 29, 2014] because he had previously been convicted thereof. About two weeks later, Deoven
executed a retraction in Baguio City where he took up his engineering course.
On July 1, 1997, RESPONDENT filed Civil Case for recovery of sum of money with
FACTS
damages against petitioner, on the premise that on June 13, 1997, he went to
Petitioner Subic Bay Legend Resorts and Casinos, Inc., a duly organized and Legenda with his brothers Ludwin and Deoven; that he handed over Legenda casino
existing corporation operating under Philippine laws, operates the Legenda Hotel and chips worth US$6,000.00, which belonged to him, to his brothers for the latter to
Casino (Legenda) located in the Subic Bay Freeport Zone in Zambales. use at the casino; that petitioner accosted his brothers and unduly and illegally
confiscated his casino chips equivalent to US$5,900.00; and that petitioner refused
On the other hand, respondent Bernard C. Fernandez is the plaintiff in Civil Case No.
and continues to refuse to return the same to him despite demand. His
237-0-97 prosecuted against petitioner in Olongapo RTC.
Complaint8 prayed for the return of the casino chips and an award of ₱50,000.00 moral
damages, ₱50,000.00 exemplary damages, ₱30,000.00 attorney's fees, ₱20,000.00
At around eleven o'clock in the evening of 6 June 1997, the appellee’s brother[,] litigation expenses, and costs.
Ludwin Fernandez[,] visited the Legenda Hotel and Casino owned and operated
by the appellant6 and located along the Waterfront Road, Subic Bay Freep011 Zone.
PETITIONER alleged that right after Ludwin and Deoven's transactions with the
Legenda had strategically installed several closedcircuit television (CCTV) cameras as
part of security measures required by its business. The monitors revealed that Legenda cashier were frozen on June 13, 1997, they voluntarily agreed to proceed to
the Legenda security office upon invitation, where Ludwin voluntarily informed
Ludwin changed $5,000.00 worth of chips into smaller denominations. Legenda
security officers that it was a certain Michael Cabrera (Cabrera) - a Legenda table
admitted in its brief that its surveillance staff paid close attention to Ludwin
inspector at the time - who gave him the casino chips for encashment, taught
simply because it was "unusual" for a Filipino to play using dollar-denominated
him how to play baccarat and thereafter encash the chips, and rewarded him with
chips. After Ludwin won $200.00 in a game of baccarat, he redeemed the value of
Pl,000.00 for every $1,000.00 he encashed; that Ludwin pointed to a picture of
chips worth $7,200.00. A review of the CCTV recordings showed that the incident was
Cabrera in a photo album of casino employees shown to him; that Ludwin and Deoven
not the first time Ludwin visited the Casino, as he had also been there on 5 June 1997.
were then brought to the IIO SBMA, where they reiterated their statements made at the
An operation was launched by Legenda to zero-in on Ludwin whose picture was Legenda security office; that they volunteered to testify against Cabrera; that
furnished its security section. Thus, unbeknownst to him, he was already closely respondent himself admitted that it was Cabrera who gave him the casino chips; that
watched on 13 June 1997 when he went with another brother, Deoven[,] to the Ludwin and Deoven voluntarily executed a joint affidavit before the Olongapo
casino at around the same time or at 11: 17 p.m. After playing (and losing $100.00) City Prosecutor's Office, which they subsequently recanted; that respondent had
only one round of baccarat, the siblings had their chips encashed at two separate no cause of action since the confiscated casino chips worth US$5,900.00 were stolen
windows. Since the cashiers were apprised of a supposed irregularity, they from it, and thus it has the right to retain them. By way of counterclaim, petitioner sought
"froze" the transaction. an award of P 1 million moral damages, ₱1 million exemplary damages, and P.5 million
Shortly thereafter, Legenda's internal security officers accosted Ludwin and attorney's fees and litigation expenses.
Deoven and ordered them to return the cash and they complied without ado
because they were being pulled away.
RTC: evidence preponderates in favor of the respondent, judgment is rendered against
The two were eventually escorted to private rooms where they were separately the petitioner.
interrogated about the source of the chips they brought. They were held for about
seven hours until the wee hours of the morning, without food or sleep. The There is no dispute that the subject chips were in the possession of the respondent. He
ultimatum was simple: they confess that the chips were given by a certain claims he got hold of them as payment for car services he rendered to a Chinese
employee, Michael Cabrera, or they would not be released from questioning. The individual. petitioner however, contends that said chips were stolen from the casino and
it is the lawful owner of the same.
same line of questioning confronted them when they were later turned-over for blotter
preparation to the Intelligence and Investigation Office of the Subic Bay Metropolitan The onus fell on petitioner to prove that the casino chips were stolen. The proof
Authority (IIO SBMA). adduced however, is wanting. The statements of Deoven and Ludwin C.
Fernandez, confessing to the source of the chips were recanted hence, have little
Finally, the brothers succumbed to Legenda's instruction to execute a joint
probative value.
statement implicating Cabrera as the illegal source of the chips. Due to hunger
pangs and fatigue, they did not disown the statement even when they subscribed the CA affirmed the trial court's May 17, 2006 Decision. CA held that, applying Article 559
same before the prosecutor in whose office they were [later] brought. On the other of the Civil Code, respondent had the legal presumption of title to or ownership
hand, they signed for basically the san1e reason a document purporting to show that of the casino chips. This conclusion springs from respondent's admission during trial
they were "released to [their] brother's custody in good condition." At the time, Deoven that the chips represented payment by a Chinese customer for services he rendered to
was about 21 years old, in his second year of engineering studies and was not familiar the latter in his car shop.
with the so-called "estafa" with which the security personnel threatened to sue him for;
ISSUE: W/N CA erred in affirming the ruling of the lower court? CANNOT apply; the presumption that the chips were exchanged for value
remains.
RULING: No. CA is correct.
WHEREFORE, the Petition is DENIED. The assailed April 27, 2010 Decision and
PETITIONER’S underlying theory is that the subject casino chips were in fact stolen by
August 24, 2010 Resolution of the Court of Appeals in CA-G.R. CV No. 91758 are
its employee Cabrera, then handed over to respondent's brothers, Ludwin and Deoven,
AFFIRMED. SO ORDERED.
for encashment at the casino; that Ludwin and Deoven played at the casino only for
show and to conceal their true intention, which is to encash the chips; that respondent's
claim that he owned the chips, as they were given to him in payment of services he
rendered to a Chinese client, is false.
Besides, a question of fact cannot properly be raised in a petition for review on
certiorari. Moreover, if petitioner should stick to its theory that Cabrera stole the
subject casino chips, then its failure to file a criminal case against the latter -
including Ludwin and Deoven for that matter - up to this point certainly does not
help to convince the Court of its position, especially considering that the
supposed stolen chips represent a fairly large amount of money. Indeed, for
purposes of this proceeding, there appears to be no evidence on record - other than
mere allegations and suppositions - that Cabrera stole the casino chips in question;
such conclusion came unilaterally from petitioner, and for it to use the same as
foundation to the claim that Ludwin, Deoven and respondent are dealing in stolen chips
is clearly irregular and unfair.
Thus, there should be no basis to suppose that the casino chips found in Ludwin's and
Deoven's possession were stolen; petitioner acted arbitrarily in confiscating the same
without basis. Their Joint Affidavit - which was later recanted - does not even bear such
fact; it merely states that the chips came from Cabrera. If it cannot be proved, in the
first place, that Cabrera stole these chips, then there is no more reason to
suppose that Ludwin and Deoven were dealing in or possessed stolen goods;
unless the independent fact that Cabrera stole the chips can be proved, it cannot
be said that they must be confiscated when found to be in Ludwin's and Deoven's
possession.
The fact that Ludwin and Deoven appear to be indecisive as to who gave them the
casino chips does not help petitioner at all. It cannot lead to the conclusion that Cabrera
stole the chips and then gave them to the two; as earlier stated, petitioner had to prove
this fact apart from Ludwin's and Deoven's claims, no matter how incredible they may
seem.
Though casino chips do not constitute legal tender, there is no law which
prohibits their use or trade outside of the casino which issues them. In any case,
it is not unusual – nor is it unlikely – that respondent could be paid by his Chinese
client at the former' s car shop with the casino chips in question; said
transaction, if not common, is nonetheless not unlawful. These chips are paid for
anyway; petitioner would not have parted with the same if their corresponding
representative equivalent - in legal tender, goodwill, or otherwise – was not received by
it in return or exchange.
Given this premise - that casino chips are considered to have been exchanged with
their corresponding representative value - it is with more reason that this Court should
require petitioner to prove convincingly and persuasively that the chips it confiscated
from Ludwin and Deoven were indeed stolen from it; if so, any Tom, Dick or Harry in
possession of genuine casino chips is presumed to have paid for their representative
value in exchange therefor. If petitioner cannot prove its loss, then Article 559
PILAR DEVELOPMENT CORPORATION, Petitioner, vs. RAMON DUMADAG, RULING: We deny.
EMMA BACABAC, RONALDO NAVARRO, JIMMY PAGDALIAN, PAY DELOS
An easement or servitude is a real right on another's property, corporeal and
SANTOS, ARMANDO TRILLOS, FELICISIMO TRILLOS, ARCANGEL FLORES,
immovable, whereby the owner of the latter must refrain from doing or allowing
EDDIE MARTIN, PRESILLA LAYOG, CONRADO CAGUYONG, GINA GONZALES,
somebody else to do or something to be done on his or her property, for the benefit of
ARLENE PEDROSA, JOCELYN ABELINO, ROQUE VILLARAZA, ROLANDO
another person or tenement; it is jus in re aliena, inseparable from the estate to
VILLARAZA, CAMILO GENOVE, NILDA ROAYANA, SUSAN ROAYANA,
which it actively or passively belongs, indivisible, perpetual, and a continuing
JUANCHO PANGANIBAN, BONG DE GUZMAN, ARNOLD ENVERSO, DONNA
property right, unless extinguished by causes provided by law.
DELA RAZA, EMELYN HAGNAYA, FREDDIE DE LEON, RONILLO DE LEON,
MARIO MARTINEZ, and PRECY LOPEZ, Respondents. [G.R. No. 194336. March 11, The Code defines easement as an encumbrance imposed upon an immovable for the
2013] benefit of another immovable belonging to a different owner or for the benefit of a
community, or of one or more persons to whom the encumbered estate does not
belong.
FACTS
On July 1, 2002, PETITIONER filed a Complaint for accion publiciana with damages
Two kinds of easement according to source:
against respondents for allegedly building their shanties, without its knowledge
and consent, in its 5,613-square-meter property located at Daisy Road, Phase V, Pilar 1. by law (Legal Easement or compulsory easement): an easement by necessity
Village Subdivision, Almanza, Las Piñas City. It claims that said parcel of land, which constituted by law has for its object either public use or the interest of private
is duly registered in its name under Transfer Certificate of Title No. 481436 of the persons; or
Register of Deeds for the Province of Rizal, was designated as an open space of Pilar 2. by will of the owners (Voluntary easement)
Village Subdivision intended for village recreational facilities and amenities for
subdivision residents.
RESPONDENTS denied the material allegations of the Complaint and briefly asserted While Article 630 of the Code provides for the general rule that "the owner of the
that it is the local government, not petitioner, which has jurisdiction and authority over servient estate retains the ownership of the portion on which the easement is
them. established, and may use the same in such a manner as not to affect the exercise
of the easement," Article 635 thereof is specific in saying that "all matters concerning
easements established for public or communal use shall be governed by the special
TRIAL COURT dismissed petitioner’s complaint, finding that the land being occupied laws and regulations relating thereto, and, in the absence thereof, by the provisions of
by respondents are situated on the sloping area going down and leading towards this Title Title VII on Easements or Servitudes."
the Mahabang Ilog Creek, and within the three-meter legal easement; thus,
In the case at bar, the applicability of DENR A.O. No. 99-21 dated June 11, 1999, which
considered as public property and part of public dominion under Article 502 of the
superseded DENR A.O. No. 97-0519 dated March 6, 1997 and prescribed the revised
New Civil Code (Code), which could not be owned by petitioner.
guidelines in the implementation of the pertinent provisions of Republic Act (R.A.) No.
The trial court opined that respondents have a better right to possess the occupied 1273 and Presidential Decree (P.D.) Nos. 705 and 1067, cannot be doubted. Inter alia,
lot, since they are in an area reserved for public easement purposes and that only it was issued to further the government’s program of biodiversity preservation. Aside
the local government of Las Piñas City could institute an action for recovery of from Section 2.1 above-quoted, Section 2.3 of which further mandates:
possession or ownership.
2.3 Survey of Titled Lands:
2.3.1 Administratively Titled Lands:
CA said that petitioner could not close its eyes or ignore the fact, which is glaring in its
The provisions of item 2.1.a and 2.1.b shall be observed as the above.
own title, that the 3-meter strip was indeed reserved for public easement. By relying
However, when these lands are to be subdivided, consolidated or
on the TCT, it is then estopped from claiming ownership and enforcing its
consolidated-subdivided, the strip of three (3) meters which falls within
supposed right. Unlike the trial court, however, the CA noted that the proper party
urban areas shall be demarcated and marked on the plan for easement
entitled to seek recovery of possession of the contested portion is not the City of Las and bank protection.
Piñas, but the Republic of the Philippines, through the Office of the Solicitor General
(OSG), pursuant to Section 10111 of Commonwealth Act (C.A.) No. 141 (otherwise The purpose of these strips of land shall be noted in the technical description
known as The Public Land Act). and annotated in the title.
2.3.3 Complex Subdivision or Consolidation Subdivision Surveys for
Housing/Residential, Commercial or Industrial Purposes:
ISSUE: W/N the petitioner is entitled to its lawful possession, hence, the proper party
to file an action for recovery of possession against respondents conformably with When titled lands are subdivided or consolidated-subdivided into lots for
Articles 42813 and 53914 of Code. residential, commercial or industrial purposes the segregation of the three (3)
meter wide strip along the banks of rivers or streams shall be observed and be dispatch the eviction, demolition, and relocation of respondents and any other
made part of the open space requirement pursuant to P.D. 1216. persons similarly situated in order to give flesh to one of the avowed policies of
R.A. 7279, which is to reduce urban dysfunctions, particularly those that
The strip shall be preserved and shall not be subject to subsequent subdivision.
adversely affect public health, safety, and ecology.
Certainly, in the case of residential subdivisions, the allocation of the 3-meter
Indeed, as one of the basic human needs, housing is a matter of state concern as it
strip along the banks of a stream, like the Mahabang Ilog Creek in this case, is
directly and significantly affects the general welfare.
required and shall be considered as forming part of the open space requirement
pursuant to P.D. 1216 dated October 14, 1977. Said law is explicit: open spaces are WHEREFORE, the petition is DENIED. The March 5, 2010 Decision and October 29,
"for public use and are, therefore, beyond the commerce of men" and that "[the] areas 2010 Resolution of the Court of Appeals in CA-G.R. CV No. 90254, which affirmed the
reserved for parks, playgrounds and recreational use shall be non-alienable public May 30, 2007 Decision of the Las Piñas RTC, Branch 197, dismissing petitioner's
lands, and non-buildable." complaint, is hereby AFFIRMED. SO ORDERED.
Running in same vein is P.D. 1067 or The Water Code of the Philippines21 which
provides:
Art. 51. The banks of rivers and streams and the shores of the seas and lakes
throughout their entire length and within a zone of three (3) meters in urban
areas, twenty (20) meters in agricultural areas and forty (40) meters in forest
areas, along their margins, are subject to the easement of public use in the
interest of recreation, navigation, floatage, fishing and salvage. No person shall
be allowed to stay in this zone longer than what is necessary for recreation,
navigation, floatage, fishing or salvage or to build structures of any kind.
(Underscoring supplied)
Thus, the above prove that petitioner’s right of ownership and possession has
been limited by law with respect to the 3-meter strip/zone along the banks of
Mahabang Ilog Creek.
Despite this, the Court cannot agree with the trial court’s opinion, as to which the CA
did not pass upon, that respondents have a better right to possess the subject
portion of the land because they are occupying an area reserved for public
easement purposes.
Similar to petitioner, respondents have no right or title over it precisely because it is
public land. Likewise, we repeatedly held that squatters have no possessory rights
over the land intruded upon. The length of time that they may have physically
occupied the land is immaterial; they are deemed to have entered the same in
bad faith, such that the nature of their possession is presumed to have retained
the same character throughout their occupancy.
—————- Might be considered immaterial
Both the Republic of the Philippines, through the OSG and the local government
of Las Piñas City, may file an action depending on the purpose sought to be
achieved. The former shall be responsible in case of action for reversion under C.A.
141, while the latter may also bring an action to enforce the relevant provisions of
Republic Act No. 7279 (otherwise known as the Urban Development and Housing Act
of 1992).24 Under R.A. 7279, which was enacted to uplift the living conditions in the
poorer sections of the communities in urban areas and was envisioned to be the
antidote to the pernicious problem of squatting in the metropolis,25 all local government
units (LGUs) are mandated to evict and demolish persons or entities occupying danger
areas such as esteros, railroad tracks, garbage dumps, riverbanks, shorelines,
waterways, and other public places such as sidewalks, roads, parks, and playgrounds.
Yet all is not lost for petitioner. It may properly file an action for mandamus to
compel the local government of Las Piñas City to enforce with reasonable
ALICIA B. REYES, Petitioner, vs. SPOUSES FRANCISCO S. VALENTIN and was respondents’ predecessor-in-interest. Dominador was also her mother’s brother
ANATALIA RAMOS, Respondents. [G.R. No. 194488. February 11, 2015] and caretaker of properties.16
FACTS Only 500 square meters were given to Dominador. Part of the 1,500 square meters was
intended as a right of way. Dominador was tasked to prepare the documents. But,
On March 28, 2006, petitioner Alicia B. Reyes, through Dolores B. Cinco, filed a
instead of limiting the conveyance to himself to 500 square meters of the
Complaint before RTC Maloles, Bulacan, for easement of right of way against
property, he conveyed the whole 1,500 square meters, including that which was
respondents, Spouses Francisco S. Valentin and Anatalia Ramos.
supposed to be the access to the barangay road. Petitioner’s mother only learned
PETITIONER alleged that she was the registered owner of a 450-square-meter parcel about what Dominador did when a meeting was called in 1989 regarding the
of land in Barangay Malibong Bata, Pandi, Bulacan, designated as Lot No. 3-B-12 and implementation of the Comprehensive Agrarian Reform Program.18 She did not cause
covered by TCT No. T-343642-(M).7 The property used to be a portion of Lot No. 3-B the recovery of her title because at that time, the Register of Deeds of Bulacan was
and was surrounded by estates belonging to other persons. Petitioner also alleged razed by fire, causing the destruction of the documents covering the subject properties.
that respondents’ 1,500-square-meter property surrounded her property, and Dominador was also her brother, whom she presumed would give her a right of way to
that it was the only adequate outlet from her property to the highway. A 113- the main road. Instead of giving way, however, he closed the passage, causing
square-meter portion of respondents’ property was also the "point least petitioner’s property’s isolation.19 Despite demands and willingness to pay the amount,
prejudicial to the [respondents]." The easement sought was the vacant portion near respondents refused to accede to petitioner’s claims.20
the boundary of respondents’ other lot.
RESPONDENT contended that the isolation of petitioner’s property was due to her
mother’s own act of subdividing the property among her children without regard
to the pendency of an agrarian case between her and her tenants. The property
chosen by petitioner as easement was also the most burdensome for
respondents.23Respondents pointed to an open space that connected petitioner’s
property to another public road.24
TRIAL COURT issued its Decision,28 dismissing the Complaint for easement of right of
way
The trial court found that petitioner’s proposed right of way was not the least
onerous to the servient estate of respondents. It noted that the proposed right of
way would passthrough improvements, such as respondents’ garage, garden,
and grotto. The trial court also noted the existence of an irrigation canal that limited
access to the public road.33 However, the trial court pointed out that "[o]ther than the
existing irrigation canal, no permanent improvements/structures can be seen
standing on the subject rice land.” Moreover, the nearby landowner was able to
construct a bridge to connect a property to the public road.35 Hence, "[t]he way
through the irrigation canal would . . . appear to be the shortest and easiest way
to reach the barangay road."
CA affirmed in toto the Regional Trial Court’s Decision.38 It found no reversible error in
the trial court’s decision to dismiss petitioner’s complaint.
ISSUE: whether petitioner has the compulsory easement of right of way over
respondents’ property.
Figure 1. Drawing showing the location of petitioner’s and respondents’ properties in
relation to the proposed easement. Petitioner’s property is located on the leftmost part
of the drawing. Respondents’ property and the proposed 113-square-meter easement RULING: The petition has no merit.
are located on the drawing’s right side that contains petitioner’s property. Barangay
Malibong Bata Road can be seen on the rightmost part of the drawing. I
Petitioner insisted that her property was not isolated because of her own acts. When The issue of ownership is irrelevant
her mother gave the property to her as part of her inheritance, there was no intention to the case; filing of a complaint for
for the property to have no outlet. easement is a recognition of the
servient property owner’s rights
According to petitioner, her and respondents’ lots were previously owned by her
mother. Respondents’ lot was given to Dominador Ramos (Dominador) who allegedly Petitioner points out that respondents’ property was previously owned by her mother.
She alleged that her uncle who was her mother’s caretaker of property fraudulently
caused the titling of the whole 1,500-square-meter property instead of just the 500- Mere convenience for the dominant estate is not what is required by law as the
square-meter portion under his name.57 basis of setting up a compulsory easement. Even in the face of necessity, if it can
be satisfied without imposing the easement, the same should not be imposed.
These allegations are relevant only if we are determining the issue of the property’s
ownership. However, this is not an issue in this case. Petitioner does not question the Based on the Ocular Inspection Report, petitioner’s property had another outlet
ownership or the registration of respondents’ title over the property. We are limited to to the highway. In between her property and the highway or road, however, is an
the issue of petitioner’s easement rights. On that matter, petitioner’s act of filing a irrigation canal, which can be traversed by constructing a bridge, similar to what
Complaint for easement of right of way is an acknowledgement that the property was done by the owners of the nearby properties.
is owned by respondents. It is tantamount to a waiver of whatever right or claim
There is, therefore, no need to utilize respondents’ property to serve petitioner’s
of ownership petitioner had over the property.
needs. Another adequate exit exists. Petitioner can use this outlet to access the public
II roads.
Petitioner failed to satisfy the Civil The outlet referred to in the Ocular Inspection Report may be longer and more
Code requirements for the grant of inconvenient to petitioner because she will have to traverse other properties and
easement rights construct a bridge over the irrigation canal before she can reach the road.
However, these reasons will not justify the imposition of an easement on
The acts of petitioner’s predecessor-in-interest necessarily affect petitioner’s
respondents’ property because her convenience is not the gauge in determining
rights over the property. One of the requirements for the grant of an easement of
whether to impose an easement of right of way over another’s property.
right of way is that the isolation of the property is not due to the acts of the
dominant estate’s owners.
Petitioner alleged that it was her uncle, Dominador, who caused the isolation of her Petitioner also failed to satisfy the requirement of "least prejudicial to the
property through his act of appropriating for himself the whole property entrusted to him servient estate."
by her mother. Moreover, he closed the passage from petitioner’s property to the public
Article 650 of the Civil Code provides that in determining the existence of an easement
road.
of right of way, the requirement of "least prejudic[e] to the servient estate" trumps
On the other hand, respondents alleged that the isolation was due to the acts of "distance [between] the dominant estate [and the] public highway." "Distance" is
petitioner’s predecessor-in-interest. She allegedly subdivided the property in favor considered only insofar as it is consistent to the requirement of "least prejudice."
of her children, including petitioner, without regard to the pending dispute over the
This court had already affirmed the preferred status of the requirement of "least
property. If the latter is true, petitioner could not claim any right to compulsory
prejudice" over distance of the dominant estate to the public highway.
easement even if it was not she who caused the property’s isolation. Petitioner
is bound by her predecessor-in-interest’s act of causing the isolation of her T]he court is not bound to establish what is the shortest distance; a longer way
property. may be adopted to avoid injury to the servient estate, such as when there are
constructions or walls which can be avoided by a round about way, or to secure the
Assuming, however, that petitioner or her mother did not cause the isolation of
interest of the dominant owner, such as when the shortest distance would place the
petitioner’s property, petitioner still cannot be granted the easement of right of
way on a dangerous decline.
way over the proposed portion of respondents’ property. This is because she
failed to satisfy the requirements for an easement of right of way under the Civil The criterion of least prejudice to the servient estate must prevail over the
Code. criterion of shortest distance although this is a matter of judicial appreciation.
Articles 649 and 650 of the Civil Code provide the requisites of an easement of right of While shortest distance may ordinarily imply least prejudice, it is not always so
way: please see codal provisions as when there are permanent structures obstructing the shortest distance; while
on the other hand, the longest distance may be free of obstructions and the easiest
or most convenient to pass through.
We agree with the Regional Trial Court’s and the Court of Appeals’ findings that
In other words, where the easement may be established on any of several
petitioner failed to establish:
tenements surrounding the dominant estate, the one where the way is shortest
1. that there was no adequate outlet to the public highway and and will cause the least damage should be chosen. However, as elsewhere stated,
if these two (2) circumstances do not concur in a single tenement, the way which
2. that the proposed easement was the least prejudicial to respondents’ estate.
will cause the least damage should be used, even if it will not be the shortest.
There is an adequate exit to a public highway.
Petitioner would have permanent structures — such as the garage, garden, and grotto
The convenience of the dominant estate’s owner is not the basis for granting an already installed on respondent’s property — destroyed to accommodate her preferred
easement of right of way, especially if the owner’s needs may be satisfied without location for the right of way.
imposing the easement.
The cost of having to destroy these structures, coupled with the fact that there is an
available outlet that can be utilized for the right of way, negates a claim that
respondents’ property is the point least prejudicial to the servient estate.
An easement is a limitation on the owner’s right to use his or her property for the
benefit of another. By imposing an easement on a property, its owner will have
to forego using it for whatever purpose he or she deems most beneficial. Least
prejudice, therefore, is about the suffering of the servient estate. Its value is not
determined solely by the price of the property, but also by the value of the owner’s
foregone opportunity for use, resulting from the limitations imposed by the easement.69
Imposing an easement on the part of respondents’ property for petitioner’s benefit
would cost respondents not only the value of the property but also the value of
respondents’ opportunity to use the property as a garage or a garden with a grotto.
Petitioner may use another outlet, which may provide longer access from her property
to the public highway, but is free from obstructions. The four-meter wide irrigation canal
may be traversed upon construction of a bridge. As noted by the trial court:
A neighboring land owner was able to construct a short concrete bridge wide enough
even for vehicles to pass through the irrigation canal from his property to the barangay
road. The Court sees no reason why plaintiff could not do the same and why it would
not be allowed if carried in accordance with the requirements set by NIA.70
It is true that an easement of right of way may be granted even if the construction
of the bridge was allowed. However, in determining if there is an adequate outlet
or if the choice of easement location is least prejudicial to the servient estate,
this court cannot disregard the possibility of constructing a bridge over the four-
meter-wide canal. This court must consider all the circumstances of the case in
determining whether petitioner was able to show the existence of all the conditions for
the easement of right of way.
The Regional Trial Court and the Court of Appeals also considered the aspect of
necessity for an easement in determining petitioner’s rights.
The trial court found that there is still no necessity for an easement of right of
way because petitioner’s property is among the lots that are presently being
tenanted by Dominador and Filomena Ramos’ children. Petitioner is yet to use her
property. The Complaint for easement was found to have been filed merely "for
future purposes."75 Thus, according to the Court of Appeals, "[a]dmittedly, there is no
immediate and imperative need for the construction of a right of way as the dominant
estate and its surrounding properties remain as agricultural lands under tenancy."76
The aspect of necessity may not be specifically included in the requisites for the
grant of compulsory easement under the Civil Code. However, this goes into the
question of "least prejudice." An easement of right of way imposes a burden on a
property and limits the property owner’s use of that property. The limitation imposed
on a property owner’s rights is aggravated by an apparent lack of necessity for
which his or her property will be burdened.

WHEREFORE, the Court of Appeals Decision promulgated on August 12, 2010 and its
Resolution promulgated on October 28, 2010 are AFFIRMED. SO ORDERED.

S-ar putea să vă placă și